100% found this document useful (5 votes)
8K views362 pages

Selective Test Handbook

This document provides an introduction to creative writing for students preparing for writing tests. It defines creative writing as original works that use techniques like narrative storytelling. Narratives typically have characters, plot, setting, conflict, and resolution. The document offers tips for developing ideas, such as brainstorming topics of interest, considering the audience, and using imagination. The overall purpose is to help students understand creative writing fundamentals and generate ideas that could score well on exams.

Uploaded by

Mani Krish
Copyright
© © All Rights Reserved
We take content rights seriously. If you suspect this is your content, claim it here.
Available Formats
Download as PDF, TXT or read online on Scribd
100% found this document useful (5 votes)
8K views362 pages

Selective Test Handbook

This document provides an introduction to creative writing for students preparing for writing tests. It defines creative writing as original works that use techniques like narrative storytelling. Narratives typically have characters, plot, setting, conflict, and resolution. The document offers tips for developing ideas, such as brainstorming topics of interest, considering the audience, and using imagination. The overall purpose is to help students understand creative writing fundamentals and generate ideas that could score well on exams.

Uploaded by

Mani Krish
Copyright
© © All Rights Reserved
We take content rights seriously. If you suspect this is your content, claim it here.
Available Formats
Download as PDF, TXT or read online on Scribd
You are on page 1/ 362

A free book for students from

grade 2 - grade 6

SELECTIVE TEST
HANDBOOK

FOR PARENTS IN NSW


FEATURING PRACTICE
QUESTIONS, TEST INSIGHTS,
AND SUPPLEMENTARY
MATERIALS WE THINK ARE
BENEFICIAL FOR SELECTIVE
TEST PREPARATION.
Dear Reader,

We are pleased to present our handbook, a


compilation of notes and materials we have gathered
over the years, focusing on various aspects of
selective test preparation. This resource aims to offer
an integrated approach to aid those preparing for the
selective test.

While this handbook provides a comprehensive


collection of our materials, it's essential to
understand that success in the selective test involves
various factors, including individual effort,
understanding, and practice. Our intention is to give
you a resource to supplement your preparation, not
to replace all other forms of study or guarantee
specific outcomes.

We encourage you to use this guide in conjunction


with other resources and personalised study plans
tailored to your needs. Always remember, each
student's journey is unique, and while our handbook
can be a valuable tool, individual results can vary
based on numerous factors.

We hope this resource proves beneficial as you


navigate your preparation.

Warm regards,

Scholarly
TABLE OF CONTENTS

Unit 1 - Introduction

rly
1.1 Definition of creative writing and why it is important
1.2 Definition of narrative and its elements (characters,
plot, setting, conflict, resolution)
1.3 Tips for coming up with ideas for stories that will score
well on writing tests
IV

Unit 2 - Pre-writing techniques VIII


2.1 Brainstorming and expanding your ideas
la
2.2 Outlining to ensure a clear and logical structure
2.3 Free writing to explore different ideas and approaches

Unit 3 - Characters XIII


3.1 Developing memorable and unique characters
ho

3.2 Creating character profiles to help bring your


characters to life
3.3 Using character development to drive the plot and
keep readers engaged

Unit 4 - Plot and conflict XVII


4.1 Crafting a compelling plot that is appropriate for the
test format
Sc

4.2 Adding conflict to create tension and interest


4.3 Resolving conflict in a satisfying way that demonstrates
your understanding of the theme or prompt

ii
TABLE OF CONTENTS
XXI Unit 5 - Setting
5.1 Importance of setting in a story
5.2 Using setting to enhance character and plot
5.3 Creating vivid and immersive settings that are
appropriate for the test format

XXIII
rly Unit 6 - Writing and Revising
6.1 Tips for writing a first draft that is engaging and follows
the test format
6.2 The revision process and how to make your writing
more polished and effective
6.3 Editing and proofreading techniques to ensure your
writing is error-free and easy to understand
la
XXVII Unit 7 - Vocabulary and style
7.1 Tips for using more complex vocabulary to impress the
reader
7.2 Techniques for adding style and flair to your writing
7.3 Examples of successful writing that uses complex
ho

vocabulary and style effectively

XXX Unit 8 - Conclusion


8.1 Recap of key points covered in the book
8.2 Encouragement to continue practicing and improving
creative writing skills
8.3 Suggestions for finding additional resources and
Sc

support for creative writing and preparing for writing tests

XXXI Unit 9 - Practice test

iii
1 Unit 1
Introduction
1.1 Definition of Creative
writing and why is it important
CREATIVEWRITING

Creative writing refers to the art of crafting a written work that is


original and imaginative. It can take many forms, including fiction,

rly
poetry, and non-fiction, and often involves the use of literary devices
such as figurative language and storytelling techniques. Creative
writing is important because it allows people to express themselves in
a unique and personal way, and it can also be a powerful form of
communication that can engage and inspire readers.
For example, a personal essay about overcoming a fear of public
speaking could be considered creative writing because it is the
writer's unique and original reflection on a personal experience.
A poem about the beauty of a sunset could also be considered
la
creative writing because it is an original work of art that uses
figurative language and imagery to convey the writer's emotions and
observations.

1.2 Definition of Narrative and


its elements (Characters, plot,
ho

setting, conflict, resolution)


Narrative refers to a type of writing that tells a story, and it is one of
the most common forms of creative writing. A narrative typically has
five key elements: characters, plot, setting, conflict, and resolution.

Characters are the people or creatures that populate a story, and


they can be central or minor.
Sc

Plot refers to the sequence of events that take place in the story,
including the problem or conflict that the characters face and the
steps they take to resolve it.

IV
01
Setting refers to the time and place where the story takes place, and

CREATIVE WRITING
it can have a significant impact on the characters and plot.

Conflict refers to the problem or struggles that the characters face,


and it is often what drives the plot forward.

Resolution is the outcome of the conflict, and it can be either positive


or negative.

Example:

rly
Image 1.1.1
la
in the story "The Three Little Pigs," the characters are the three little
pigs and the big bad wolf. The plot involves the pigs building homes
and the wolf trying to blow them down. The setting is a forest or
countryside, and the conflict is the wolf's attempt to eat the pigs. The
resolution is the pigs' victory over the wolf when they build a house
made of bricks that the wolf cannot blow down.
ho

1.3 Tips for coming up with ideas


for stories that will score well on
writing tests
To come up with ideas for stories that will score well on writing tests,
Sc

you can try the following tips:

V
01
TIP 1: Start by brainstorming ideas that interest you and that
CREATIVE WRITING
Descriptive you feel passionate about. This can help you write about a
Language topic that you are genuinely invested in and that you will
enjoy exploring.

For example, if you love animals, you could brainstorm ideas


for stories about animals, such as a tale about a clever fox
outwitting a group of hunters or a story about a lost kitten
finding its way home.

TIP 2: Consider your audience and what they might enjoy


Audience reading about. This can help you choose a topic that will be
appealing and relevant to the people who will be reading

rly
your story.

For example, if you are writing for a younger audience, you


might consider ideas for stories that feature adventurous or
imaginative themes, such as a story about a group of friends
who go on a treasure hunt or a tale about a magical land
filled with talking animals.

TIP 3: Use your imagination to come up with unique and


la
Imagination original ideas. This can help you create a story that stands
out and that captures the reader's attention.

For example, you could try combining two seemingly


unrelated ideas to create a unique and unexpected story,
such as a romance between a mermaid and a robot or a
ho

tale about a group of pirates who are also food critics.


Inspiration
TIP 4: Look for inspiration in the world around you, including
events in the news, people you meet, and your own life
experiences. This can help you find ideas for stories

Writing
exercises TIP 5: Use prompts or writing exercises to help get your
Sc

creative juices flowing.

VI
01
Example: NARRATIVE WRITING PROMPT

CREATIVE WRITING
You are a detective who has been called to investigate a
strange case. When you arrive at the scene, you find a locked
door with a note attached to it. The note reads: "I didn't do it.
You'll never find the real culprit. - A guilty person."
As you try to unlock the door, you realize that it is locked from
the inside. You hear a faint noise coming from inside the room.
What do you do next?

Example: ESSAY

As a detective, I was no stranger to strange cases, but this one

rly
was particularly perplexing. When I arrived at the scene and
laid eyes on the locked door with the enigmatic note affixed to
it, I knew I had my work cut out for me.
I scrutinised the note with a hawk's eye, attempting to glean any
clues from its cryptic message. "I didn't do it," it said. "You'll
never find the real culprit." Who was this guilt-ridden person, and
what had they perpetrated?
I made the decision to attempt to unlock the door. I retrieved my
lock-picking kit and got to work. It was a formidable lock, but I
was resolute to get inside and discover what was happening. As
la
I worked, I heard a faint noise emanating from inside the room. It
sounded like something shuffling or rustling around.
I had to act swiftly. I couldn't leave whatever was inside the
room locked up indefinitely, and I needed to find out what was
occurring. I redoubled my efforts and finally succeeded in
unlocking the door. I gingerly swung it open, my gun at the ready
ho

in case of any danger. What I saw inside the room was


astounding. There was a young woman huddled in the corner, her
attire tattered and her face covered in grime and bruises. She
appeared terrified and relieved to see me.
"Thank goodness you're here," she said, sobbing. "I've been
locked in here for days. I didn't do it, I swear. Please, you have to
help me."
I knew I had to get to the bottom of the truth. I comforted the
young woman and promised to do everything in my power to
Sc

assist her. And with her aid, I was able to track down the real
culprit and bring them to justice. It was a long and arduous case,
but in the end, I was able to solve it and help the innocent victim.
That's what being a detective is all about.

VII
2PRE WRITING TECHNIQUES
Unit 2
Pre-writing techniques
In this chapter, we will be exploring a variety of techniques that can
help you generate ideas and plan your writing. We will start by
looking at brainstorming and expanding our ideas, which can be a
helpful way to come up with a list of potential topics or ideas for a
story. We will then move on to outlining, which can help you organize
your thoughts and ensure that your story has a clear and logical

rly
structure. Finally, we will discuss free writing, which is a technique that
involves writing freely and continuously without worrying about
grammar, structure, or other formalities. These pre-writing techniques
can be helpful tools to help you get started with your writing and set
yourself up for success. So, let's get started!

2.1 Brainstorming and expanding


your ideas
la
Brainstorming is the process of coming up with as many ideas as
possible without censoring or evaluating them. This can be a helpful
way to generate a list of potential topics or ideas for a story. To
brainstorm, you can simply start by jotting down any thoughts or ideas
that come to mind, or you can use prompts or writing exercises to
ho

stimulate your thinking.

For example, you might start by brainstorming ideas for a story about
a young woman who discovers she has magical powers. Some
potential ideas that you might come up with include:

She discovers her powers while trying to save a friend from danger

She is part of a secret society of magic users who have been


Sc

hiding their powers from the world

She is the only one in her family who has powers, and she must
figure out how to use them without revealing her secret

VIII
02

PRE WRITING TECNIQUES


Once you have a list of ideas, you can then expand upon them by
asking yourself questions and adding more details. For example, you
might ask yourself:

she
d oes
How t o us
e What is t
he
n
r eac
t lear m? source of
sh e the her
d oes scovers powers?
How she di
h e n o w ers?
w p
has
she

rly
la
2.2 Outlining to ensure a clear
and logical structure
Once you have a general idea of the story you want to tell, it can be
helpful to create an outline to help you organize your thoughts and
ho

ensure that your story has a clear and logical structure.

An outline is a way of breaking down your story into smaller chunks


and organizing them in a way that makes sense. It can help you
figure out the sequence of events and make sure that your story has a
beginning, middle, and end.

For example, you might create an outline for your story about a young
Sc

woman who discovers she has magical powers like this:

IX
02
PRE WRITING TECHNIQUES INTRODUCTION

Image 2.2.1

Introduce the main character and her normal life


Set up the problem or conflict that she will face

RISING ACTION

rly
Image 2.2.2

Describe the moment when the main character discovers


la
she has magical powers
Show how she reacts to this discovery
Describe how she learns to use her powers

CLIMAX
ho

Image 2.2.3

Show the main character facing a crisis or turning point in her


Sc

journey
Describe how she uses her powers to overcome this challenge

X
02

PRE WRITING TECNIQUES


FALLING ACTION

Image 2.2.4

Show the aftermath of the climax and how the main


character's life changes as a result

RESOLUTION
rly
Image 2.2.5

Describe how the main character's life is different now


la
that she has discovered her powers
Show how she has grown or changed as a result of her
journey

2.3 Free writing to explore


ho

different ideas and approaches


Free writing is a technique that involves writing freely and
continuously without worrying about grammar, structure, or other
formalities. It can be a helpful way to explore different ideas and
approaches and to get your creative juices flowing.
Sc

XI
02

To free write:

Set a timer for a certain amount of time (such as 10-15 minutes)

start writing whatever comes to mind


PRE WRITING TECHNIQUES

Use prompts or writing exercises to help stimulate your

Or you can simply start with a blank page and see what emerges.

rly
For example, you might try free writing about a character you are
developing for a story. You might start by writing a few sentences
describing the character and then continue writing whatever comes to
mind, without worrying about grammar or structure. You might come up
with something like this:
la
Jenny was a curious and adventurous young
woman with a wild streak. She had always
been drawn to the unknown and had a
particular fascination with magic and the
supernatural. When she discovered that she
ho

had magical powers of her own, she was


thrilled and terrified at the same time. She
had no idea how to control her powers or
what she was supposed to do with them. But
she was determined to learn and to use
them for good.
Sc

XII
Unit 3
Characters
3

CHARACTERS
Welcome to chapter 3: Characters

In this chapter, we will be exploring the important role of characters in


creative writing. Characters are the people or creatures that populate
a story, and they can be central or minor. In order to create
memorable and unique characters, it is important to give them distinct

rly
personalities, flaws, and desires. We will discuss how to develop
memorable and unique characters, as well as how to create character
profiles to help bring your characters to life. We will also look at the
role of character development in driving plot and keeping readers
engaged. By the end of this chapter, you will have a better
understanding of how to create compelling and well-rounded
characters for your stories. Let's get started!
la
3.1 Developing memorable and
unique characters
One of the key elements of creative writing is creating memorable and
unique characters.
ho

In order to create memorable characters, you should give them distinct


personalities, flaws, and desires. You should also consider their
appearance, background, and any other details that will help bring
them to life.

For example, if you are writing a story about a detective, you might
create a character named Jack who is smart, witty, and determined,
but also stubborn and sometimes arrogant. You might give him a scar
Sc

above his left eyebrow that he got when he was younger, and you
might give him a troubled relationship with his family.

XIII
03
3.2 Creating character profiles to
help bring your characters to life
One way to help develop your characters more fully is to create
character profiles. A character profile is a detailed description of a
character that includes information about their appearance,
personality, background, and other relevant details. Creating character
profiles can help you get to know your characters better and can also
help you stay consistent with their portrayal throughout your story.

For example, you might create a character profile for Jack like this:

rly Name: Jack

Age: 35

Appearance: Tall and lean, with


short dark hair and piercing blue
eyes. He has a scar above his left
eyebrow and is usually impeccably
la
dressed in a suit
CHARACTERS

Personality: Jack is smart, witty, and


determined, but he can also be stubborn
and sometimes arrogant. He is fiercely
independent and doesn't like to ask for
ho

help, but he is also fiercely loyal to those he


cares about.

Background: Jack grew up in a rough


neighborhood and had to fight for
everything he had. He worked his way up
through the ranks to become a detective,
and he is fiercely proud of his
accomplishments. He has a troubled
Sc

relationship with his family and doesn't see


them often.

XIV
03
3.3 Using character development to

CHARACTERS
drive plot and keep readers engaged
Character development refers to the changes that a character
undergoes over the course of a story. This can include changes in their
personality, beliefs, or circumstances. Using character development
can help keep readers engaged and can also help drive the plot of
your story.
For example, in your story about Jack the detective:

You might show him growing and changing as he works on


different cases and experiences different challenges.

rly
You might show him learning to be more open to help and to trust
others, or you might show him struggling with difficult decisions
and learning from his mistakes. By showing Jack's character
development, you can create a more compelling and engaging
story.

You might show him struggling with a difficult case that pushes him
to his limits.
la
You might show him initially being stubborn and refusing to ask for
help, but then eventually learning to trust his colleagues and
accept their assistance. This could be a significant moment of
character development for Jack, and it could also help drive the
plot of the story forward.
ho

Example: WRITING PROMPT

Write a 500-word essay about a detective who becomes


obsessed with solving a cold case. Describe the detective's
character and how their determination and tenacity drive them
to solve the case. Use the character development to drive the
plot and show how the detective's obsession with the case
changes them over time. How do they feel when they finally
solve the case, and how does it change their perspective on
Sc

their job? Use descriptive language and figurative devices such


as personification and metaphor to bring the story to life.

XV
03
Example: ESSAY

Jack was a detective with a fierce determination to solve


every case he was assigned to. He was known for his tenacity
and his ability to see a case through to the end, no matter how
long it took. So when he was given the cold case of a missing
person, he knew he had to do everything in his power to crack it.
Despite the months that had passed since the crime had been
committed, Jack was convinced that there were still clues to be
found. He spent long hours poring over the evidence and
following leads, trying to piece together the puzzle. He was like a
dog with a bone, unwilling to let go until he had found the truth.

rly
As he worked on the case, Jack began to feel a connection to the
missing person. He imagined what it would be like to be in their
shoes, to be lost and alone in the world. He felt a sense of
responsibility to bring them home, to give them the closure and
the peace they deserved. As the days turned into weeks and the
weeks turned into months, Jack's obsession with the case grew.
He worked late into the night, ignoring the aches and pains that
wracked his body. He was driven by a single-minded purpose
and an unyielding will. And then, finally, it happened. As he was
la
sifting through a pile of old documents, he saw something that
CHARACTERS

caught his eye. It was a small detail, something that seemed


insignificant at first, but as he looked closer, he realised that it
was the key to the case.

He felt a surge of triumph and elation as he pieced together the


ho

clues and saw the puzzle fall into place. He had done it. He had
solved the case and brought the missing person home. He had
given them the closure and the peace they deserved, and he had
brought a little bit of light into the darkness. As he closed the
case file and stood up from his desk, he felt a sense of pride and
accomplishment wash over him. He had done something
worthwhile, something that mattered. He had made a difference,
and that was all that mattered.From that day on, Jack became
known as the detective who never gave up. He was respected
Sc

and admired by his colleagues, and he was proud of the work he


did. He knew that he had a calling, and a purpose in life, and he
was determined to see it through to the end.

XVI
Unit 4
Plot and Conflict
4.1 Crafting a compelling plot that is appropriate for
4

PLOT AND CONFLICT


the test fromat
Plot is the sequence of events that make up a story. It is the way that
the story unfolds and the order in which the events take place. A good
plot is essential for a successful story, as it provides structure and helps
keep the reader engaged.

EXPOSITION

RISING ACTION
rly
There are several elements that make up a plot, including:

This is the introduction to the story, where the


setting, characters, and conflict are introduced.

This is the part of the story where the conflict or


problem begins to escalate and the stakes
become higher.

This is the turning point of the story, where the


CLIMAX conflict or problem reaches its peak and the
la
outcome is uncertain.

This is the part of the story where the conflict or


FALLING ACTION problem begins to resolve itself and the
characters work towards a resolution.
This is the part of the story where the conflict or
RESOLUTION problem is fully resolved and the story comes to
ho

an end.

4.2 Adding conflict to create tension and interest


Conflict is an essential Person vs. person
element of a good plot. This is a conflict between two characters,
It is the problem or such as a fight or a disagreement.
obstacle that the
characters must
Sc

overcome in order to
reach their goal.
Conflict can take many
forms, including:

XVII
04
Person vs. self

This is a conflict within a character, such as a struggle with their own


emotions or desires.

Person vs. nature

rly
PLOT AND CONFLICT

This is a conflict between a character and a natural force, such as a


storm or a wild animal.

Person vs. society


la
This is a conflict between a character and the society or culture they
live in, such as a character fighting against discrimination or injustice.
4.3 Resolving conflict is an important part of a story's resolution or
the way that the story ends. Students should think about how to
ho

resolve conflicts in a way that is satisfying and makes sense within the
context of their story.

Example: WRITING PROMPT

Write a creative story about a group of animal friends who embark on


an adventure through an enchanted forest. Describe the wonders of
the forest and the challenges that the animal friends encounter along
the way. Use descriptive language and figurative devices such as
Sc

personification and metaphor to bring the story to life. How do the


animal friends handle the challenges they face, and how does their
adventure change them? How do they feel when it is time for them to
leave the forest and return home?

XVIII
04
Example: ESSAY

As the animal friends ventured through the enchanted forest, they

PLOT AND CONFLICT


were greeted by its many wonders. The trees seemed to soar
towards the sky, their branches extended like benevolent arms
embracing the travellers. The flowers that dotted the forest floor
swayed in the breeze, their petals undulating to an imperceptible
melody.
But as they progressed further into the forest, they encountered a
malevolent presence blocking their path. The malevolent old bear
stood before them, his beady eyes narrowed in disdain. He
appeared to detest their presence, his growls reverberating
through the ground like thunder.

"What do we do now?" asked the timid rabbit, her voice shaking


with fear.
rly
"We can't turn back now," said the kind-hearted deer, trying to
reassure her friends. "We've come too far and faced too many
challenges. We just have to find a way to get past this bear."
"But how?" asked the mischievous squirrel, eyeing the bear warily.
"He looks pretty tough."
"We'll have to use our brains," said the clever fox, his eyes glinting
with determination. "We each have our own unique skills and
talents. If we use them together, we might be able to distract the
la
bear and sneak past him while he's distracted."

The animal friends nodded, their determination renewed. They


huddled together and brainstormed ideas. The rabbit could use
her speed to dart past the bear's legs, the squirrel could climb up
a tree and throw acorns at the bear to distract him, and the deer
ho

could use her soothing voice to try and calm the bear down.
"Are you ready?" asked the fox, his gaze moving from one animal
friend to the next.
"As ready as we'll ever be," said the rabbit, taking a deep breath.

The animal friends nodded, their nerves steadying as they


prepared to put their plan into action. They took a deep breath
and stepped forward, determination etched on their faces.
The rabbit zigzagged past the bear's legs, the squirrel pelted him
Sc

with acorns, and the deer spoke soothing words to him in her
gentle voice. The bear roared and swatted at them, but they were
quick and agile, and they managed to get past him and continue
on their journey.

XIX
04
As they travelled further into the forest, the animal friends couldn't
help but feel a sense of accomplishment. They had faced a
difficult challenge and come up with a creative solution. They
knew that they could handle whatever else the forest might throw
their way.

The forest itself seemed to recognise their bravery, its atmosphere


shifting from menacing to welcoming. The trees rustled in approval,
their leaves shimmering like jewels in the sunlight. The flowers that
dotted the forest floor nodded their heads in admiration, their
petals unfurling like flags of victory. The animal friends were filled
with a sense of wonder as they explored the forest. It was as if they
were walking through a dream, a place where anything was

rly
possible. They climbed up tree trunks that seemed to stretch up to
PLOT AND CONFLICT

the stars, they chased butterflies that sparkled like diamonds, and
they splashed through streams that glimmered like silver.
But as the sun began to set and it was time for the animal friends
to return home, a sense of melancholy washed over them. They
knew that their magical adventure was coming to an end, and that
they would have to bid farewell to the enchanted forest and all its
wonders.

"I don't want to leave," said the rabbit, her eyes filling with tears.
la
"This place is so beautiful and magical. I never want to leave."
"I know how you feel," said the deer, patting her friend on the
back. "But we can't stay here forever. We have to return to our
own homes and lives."
"But we can always come back," said the squirrel, his eyes
twinkling with mischief. "We can make this our special place, and
ho

return here anytime we need to escape from the real world."

The animal friends nodded, their spirits lifting at the thought. They
knew that they would always cherish the memories of their
adventure and that they would always hold a special place in their
hearts for the enchanted forest and all its inhabitants. Finally, after
what felt like an eternity, they reached the edge of the forest and
stepped out into the bright sunlight. As they looked back one last
time, the forest seemed to shimmer and glow, as if bidding them a
Sc

fond farewell. The animal friends knew that they would always
carry the magic of the forest with them, and that they would return
again one day to explore its many wonders.

XX
5.1 Importance of setting in a story
Unit 5
Setting
5

SETTING
Setting is the time and place in which a story takes place. It can be a
real place or a fictional one, and it should be described in a way that
is immersive and helps to set the mood for the story. Setting can also
be used to enhance character and plot. For example, a character's
actions and decisions might be influenced by the setting, and the
setting can help to create a sense of atmosphere or tension.

TIME PERIOD
rly
There are several elements that make up a setting, including:
This is the time in which the story takes place, such
as the present day, the past, or the future.

This is the place in which the story takes place,


LOCATION such as a city, a countryside, or a different planet.

This is the mood or feeling that the setting


ATMOSPHERE creates, such as gloomy, hopeful, or mysterious.
la
This is the culture, customs, and values of the
SOCIAL CONTEXT
society in which the story takes place.

It is important for writers to consider the setting carefully when


crafting a story, as it can help to create a sense of immersion and
believability. Vivid and immersive descriptions of the setting can help
ho

to transport the reader into the world of the story and make it feel
more real.

Example: WRITING PROMPT

Write a creative essay set in the 1920s about a young woman who
dreams of becoming a jazz singer and the challenges she faces
along the way. Make sure to incorporate the setting of the 1920s
Sc

into the story, and use it to enhance the character and plot. Use
at least 5 instances of personification and 10 instances of
metaphor to add flair to your writing.

XXI
05
Example: ESSAY

As the night sky twinkled with the glow of the city's neon lights,
Alice stood on stage, her heart thrumming with exhilaration. She
was about to make her debut at the most popular club in the city,
and she was determined to make it a performance to remember.

The band struck up a lively beat, and Alice let out a joyous laugh
as she began to sing. Her voice was like a fragrant flower, soft and
velvety, and it seemed to envelop the listeners in a warm embrace.
The audience erupted into cheers and applause, mesmerised by
her charisma and talent.

rly
But as the night progressed, Alice encountered more and more
challenges. She had to fend off greedy agents who tried to steal
her spotlight and spiteful rivals who hoped to see her fail. She also
had to confront the expectations and prejudices of her society, as
she fought to break free from the confines of traditional roles.

However, Alice was tough and courageous, and she refused to let
these obstacles stand in her way. She sang with every fiber of her
being, her voice soaring like a celestial being, and she inspired
la
others to do the same.

"She's got something special," the club owner exclaimed, his eyes
sparkling with admiration.
"I've never seen anyone like her before."
ho

"Absolutely," the band leader agreed, a smile spreading across his


face.
SETTING

"She's a true star, destined for greatness." And greatness is exactly


what Alice achieved. She became a household name, with her
music played on the radio and her face splashed across headlines.
She inspired others to chase their dreams and showed the world
that anything was possible with hard work and determination.
Sc

XXII
Unit 6
Writing and Revising
6

WRITING AND REVISING


In this chapter, we will explore the importance of the writing and
revision process in crafting a successful and engaging piece of
creative writing. We will discuss various techniques and strategies for
writing a strong first draft and revising it to make it more polished and
effective. We will also cover editing and proofreading techniques to
ensure that your writing is error-free and easy to understand. By the

writing the best it can be.

Writing a first draft


rly
end of this chapter, you should have a better understanding of how to
approach the writing and revision process, and how to make your

Writing a first draft is the initial stage of the writing process, where
you get all of your ideas down on paper. It's important to just focus on
getting your ideas out and not worry too much about grammar,
spelling, and structure at this stage. You can always fix those things
later during the revision process.
la
Here are the recommended steps of how you might go about writing a
first draft:

Step 1: Plan your story. Take some time to brainstorm ideas and plot
out the basic structure of your story. Consider the characters, setting,
conflict, and resolution.
ho

Step 2: Start writing. Set a timer for 30 minutes and start writing. Don't
worry about grammar or spelling at this stage, just focus on getting
your ideas down.

Step 3: Keep writing. When the timer goes off, take a short break and
then set it for another 30 minutes. Continue this process until you have
a complete first draft.
Sc

Step 4: Edit and revise. Once you have a complete first draft, set it
aside for a day or two and then come back to it with fresh eyes. Use
the revision process to improve the clarity and flow of your writing,
and to make sure it is as engaging and effective as possible.

XXIII
06
By following these steps, you can write a strong first draft that you can
then revise and polish to create a final product that you are proud of.

Tips for writing a strong first draft

Start with an outline or a plan to help you organize your


thoughts and ideas.
Don't worry about perfection – just focus on getting your ideas
down on paper.

Don't be afraid to revise and change things as you go.


WRITING AND REVISING

Use transitional words and phrases to help connect your ideas


and create a logical flow.

Revising the draft

rly
Revising a writing draft is an important step in the writing process. It
allows you to make improvements to your work and ensure that it is the
best it can be. When revising a writing draft, it is important to consider
the following:
Content: Is the content of your writing clear and well-organized? Do
you have a clear purpose for writing? Are you using evidence to
support your ideas?
la
Structure: Does your writing have a logical structure? Do your
paragraphs flow smoothly from one to the next? Do you have a strong
introduction and conclusion?

Word choice: Are you using precise and descriptive language? Are
you avoiding overused or cliche words? Are you using a variety of
words to convey your ideas?
ho

Grammar and mechanics: Are you using proper grammar and


punctuation? Have you checked for spelling mistakes?

Tone and voice: Is the tone of your writing appropriate for your
audience and purpose? Do you have a strong voice that comes
through in your writing?
To revise your writing draft, you can start by reading it over and taking
Sc

note of any areas that need improvement. Then, go through your


writing and make the necessary changes. You may want to ask
someone else to read your writing and give feedback as well. It can
also be helpful to take a break from your writing and come back to it
with fresh eyes.

XXIV
06
Remember, revising your writing draft is a crucial step in the writing

WRITING AND REVISING


process. It takes time and effort, but it is worth it to create the best
possible piece of writing. So, it is always better to revise your writing
draft before submitting it.

Example: WRITING PROMPT

Write a creative story about a group of friends who embark on a


journey to find a magical treasure. The treasure is said to grant
wishes to those who find it, and the friends are determined to be
the ones to uncover it. Along their journey, they face challenges and
obstacles, but they work together and never give up. Will they be
able to find the magical treasure and have their wishes granted?

Example: ESSAY

rly
It was a tumultuous and tempestuous night when a group of friends set
out on a journey to find a magical treasure that granted wishes to
those who found it. As they trudged through the dense forest, the wind
howled through the trees and the rain pounded against the leaves.
"Are you sure we're going the right way?" asked Sarah, the group's
skeptic.
"This storm doesn't seem to be letting up and I'm starting to get a
foreboding feeling about this."
The friends emerged from the trees into a clearing and saw the
la
treasure lying before them, glowing in the moonlight. It was a chest,
intricately carved and adorned with jewels. "We did it!" exclaimed
Alex, rushing towards the treasure. But as he reached for it, a bolt of
lightning struck a nearby tree, causing it to fall and block their path.
They soon realised it wasn't an obstacle blocking their path - it was a
dragon, guarding the treasure. As the dragon breathed fire, the
ho

friends scattered, trying to come up with a plan. Emily remembered a


legend she had heard about dragons being afraid of music, and
suggested they try playing their instruments to calm it down. The
friends began to play and sing, the music carrying across the clearing.
The dragon paused, its flames flickering. Slowly, it began to sway to
the rhythm of the music.
As the dragon calmed down, the friends were able to approach the
treasure and claim their prize. They made their wishes and the
treasure granted them, filling them with joy and gratitude. As they
Sc

made their way back home, the storm had passed and the sun was
shining. The friends knew that this journey had changed them and that
they had learned that anything was possible if they worked together
and never gave up. The friends were overjoyed and grateful for the
magical experience they had shared together.

XXV
7VOCABULARY AND STYLE
Unit 7
Vocabulary and style
Vocabulary is an important part of language skills and plays a crucial
role in effective communication. It is the collection of words that a
person knows and uses in their writing and speaking. Strong
vocabulary skills can help you understand and express complex ideas,
and can also make your writing more interesting and engaging. Using
more complex vocabulary in your writing can help you impress your

rly
reader and make your writing more engaging. Here are some tips and
strategies for incorporating more advanced words into your writing:

7.1 tips for using complex vocabulary to impress


readers
1. Read widely: Reading exposes you to new words and helps you
learn their meanings in context. As you encounter new words, make a
note of them and try to use them in your own writing.

2. Use a thesaurus: A thesaurus is a useful tool for finding synonyms, or


la
words with similar meanings, to use in your writing. This can help you
vary your vocabulary and make your writing more interesting.
Wordhippo.com, Vocabulary.com

3. Choose specific words: Instead of using general words, try to


choose more specific, precise words that convey your meaning more
ho

effectively.

4. Use figurative language: Figurative language, such as metaphors


and similes, can help you convey your ideas in a more vivid and
imaginative way. By using figurative language, you can also show off
your vocabulary skills and impress your reader.

For example, instead of using general language: “The sun was shining
brightly and made the grass look green”
Sc

You could use figurative language (metaphor): The sun was a glowing
orb, making the grass look like a sea of emeralds.

Same idea, expressed differently

XXVI
07
5. Avoid overusing complex words: While using more complex
vocabulary can be impressive, it is important not to overdo it. If you
use too many advanced words, it can be confusing for your reader
and take away from your message.
By following these tips and strategies, you can effectively incorporate
more complex vocabulary into your writing and impress your reader. It
takes time and effort to expand your vocabulary, but it is a worthwhile
investment that can improve the effectiveness and impact of your
writing.
Example: WRITING PROMPT

VOCABULARY AND STYLE


Writing prompt: Write a creative scene in which you describe a
moment where you are sitting by a fireplace on a cold winter night.
In your writing, use descriptive language to bring the setting and

rly
atmosphere to life and create a vivid picture for the reader. You
may also include dialogue and any other relevant details that add
to the scene.

Example: ESSAY

I ensconced myself by the fireplace on a frigid winter night,


swaddled in a comfortable blanket, the heat of the flames
percolating into my bones. The fire undulated and flickered, its light
casting ghostly shadows on the walls. Outside, the wind howled, the
la
trees bending and oscillating like ethereal figures.
I flicked a glance at the clock, noting the tardy hour. I should have
retired hours ago, but something about the salubrious crackle of the
fire kept me affixed to the spot. I couldn't shake the feeling that I
was intended to be here, imbibing the warmth and solace of the
flames.
ho

As I sat mired in contemplation, the sound of the door creaking open


arrested my attention. My heart skipped a beat as a figure emerged
from the inky darkness, their visage obscured by the shadows.
"Who's there?" I called out, my voice trembling.
"It's just me," came the reply, and I exhaled a sigh of relief as my
friend stepped into the light.
"You startled me," I said, laughing nervously. "What are you doing
here so late?" "I couldn't slumber," my friend replied, taking a seat
next to me. "The wind was keeping me awake. I surmised I'd come
Sc

see if you were still up and savouring the fireplace."


I gestured to the spot next to me. "As you can see, I am. Want to join
me?" My friend nodded and settled in, pulling their own blanket
around them. We sat in amicable silence, the fire providing a warm
and cosy atmosphere.

XXVII
07
7.2 Techniques for adding style and flair
to your writing
Adding style and flair to your writing can help make it more engaging
and interesting for your reader. Here are some techniques you can use
to add style and flair to your writing:
1. Use descriptive language: Use descriptive words and phrases to
paint a picture in the reader's mind and make your writing more vivid.
VOCABULARY AND STYLE

Instead of saying "the tree was tall," you could say "the tree soared
into the sky, its branches reaching towards the clouds."

2. Vary your sentence structure: Instead of using the same sentence


structure over and over, mix it up to keep your writing interesting. Try

rly
using short and long sentences, as well as complex and compound
sentences.

3. Use rhetorical devices: Rhetorical devices, such as repetition and


rhetorical questions, can add emphasis and impact to your writing. By
using these techniques effectively, you can add style and flair to your
writing.

4. Show, don't tell: Instead of just telling the reader what happened,
la
try to show them through your writing. Use descriptive language and
imagery to bring your writing to life and engage the reader.

By using these techniques, you can add style and flair to your writing
and make it more engaging and interesting for your reader. Remember,
it takes practice to develop your own writing style, so don't be afraid
ho

to experiment and try new things.

Example: WRITING PROMPT

Write an essay in which you describe a city that you visit. In your
writing, use descriptive language to bring the setting to life and
create a vivid picture for the reader. You may also include
dialogue and any other relevant details that add to the scene. As
you describe the city, explore the themes of discovery, alienation,
Sc

and danger. How do these themes affect your experience in the


city? What do you learn about the city and its inhabitants? How
does your time in the city change you?

XXVIII
07
Example: ESSAY

As I stood at the edge of the lost city, I couldn't believe my eyes. The
city was a sprawling metropolis, its streets and buildings stretching
out as far as the eye could see. The sun glinted off the skyscrapers,
the light reflecting off the glass and steel. The air was alive with the
sounds of the city, the honking of horns, the chatter of pedestrians,
the hum of traffic. It was like nothing I had ever seen before, a city
of endless possibility and opportunity.
But as I stepped into the city, I couldn't shake the feeling that

VOCABULARY AND STYLE


something was off. The people seemed distant and disconnected,
their eyes glazed over and unseeing. The air was thick with the
scent of pollution, the stench almost overwhelming. The city was

rly
beautiful, but there was a darkness lurking beneath its surface, a
malevolence that I couldn't quite put my finger on.

As I wandered through the city, I couldn't help but feel a sense of


alienation. I was an outsider here, a stranger in a strange land. I
didn't belong in the lost city, and it was clear that the city didn't
want me either. But I couldn't turn back now, not when I was so close
to discovering the truth about the lost city.

So, I pushed on, my feet taking me deeper into the heart of the city.
la
I followed the whispers and rumors, the clues that led me to the
truth. And as I peered into the shadows, I saw it. The lost city was a
place of darkness and deceit, a city built on lies and corruption. It
was a city of shadows and secrets, a place where no one was safe.

I knew then that I had to leave the lost city, to escape before it was
ho

too late. I turned and ran, my heart racing with fear and
desperation. I don't know how I made it out of the city alive, but I
did. And as I left the lost city behind, I vowed never to return. The
secrets of the city would stay hidden, buried beneath its streets and
buildings.

But even as I left the lost city behind, I knew that I would never
forget it. The city had left its mark on me, a scar that would stay with
me forever. The lost city was a place of danger and darkness, a
Sc

place that I had been lucky to escape. And as I made my way home,
I knew that I had been changed by my time in the lost city. The
secrets and horrors that I had witnessed there would stay with me
always, a reminder of the dangers that lurked in the shadows.

XXIX
8
CONCLUSION
Unit 8
Conclusion

rly
la
Over the course of this book, we have covered a variety of topics and
techniques that can help you improve your writing skills and unleash
your creativity. We have explored the importance of reading,
brainstorming, and planning as well as the use of descriptive
language, figurative language, and dialogue to bring your writing to
life.
ho

It's important to remember that writing is a skill that takes time and
practise to develop. The more you write, the more comfortable you will
become with expressing your ideas and finding your voice. So, don't
be afraid to experiment and try new things. Whether you're writing a
short story, a poem, or a personal essay, the key is to have fun and
enjoy the process.

We hope that you have found this textbook helpful and that it has
Sc

inspired you to keep writing and exploring your creativity. Remember,


the possibilities are endless when it comes to creative writing, and the
only limit is your imagination. So, pick up your pen and get writing!

XXX
Unit 9
Pratice test
9
Topic 1: Write an essay in which you describe a journey or adventure that
you undertake. In your writing, use descriptive language to bring the
setting and atmosphere to life and create a vivid picture for the reader.
You may also include dialogue and any other relevant details that add to
the scene. As you describe your journey, explore the themes of survival,
determination, and self-discovery. How do these themes affect your

rly
experience on the journey? What do you learn about yourself and the
world around you? How does your journey change you?

Topic 2: Write an essay in which you describe a mysterious and magical


place that you discover. In your writing, use descriptive language to bring
the setting and atmosphere to life and create a vivid picture for the
reader. You may also include dialogue and any other relevant details
la
that add to the scene. As you describe the place, explore the themes of
adventure, discovery, and imagination. How do these themes affect your
experience in the mysterious place? What do you learn about the place
and its inhabitants? How does your time in the mysterious place change
you?
ho

Topic 3: Write an essay in which you describe a strange and incredible


dream that you have. In your writing, use descriptive language to bring
the setting and atmosphere to life and create a vivid picture for the
reader. You may also include dialogue and any other relevant details
that add to the scene. As you describe the dream, explore the themes of
imagination, creativity, and the power of the mind. How do these themes
affect your experience in the dream? What do you learn about yourself
Sc

and your unconscious mind? How does the dream change you?

XXXI
09

Topic 4: Write an essay in which you describe a journey through time and
space. In your writing, use descriptive language to bring the setting and
atmosphere to life and create a vivid picture for the reader. You may
also include dialogue and any other relevant details that add to the
scene. As you describe your journey, explore the themes of adventure,
exploration, and the unknown. How do these themes affect your
experience on the journey? What do you learn about the past, present,
and future? How does your journey through time and space change you?

Topic 5: Write an essay in which you describe a fantastical and

rly
incredible creature that you encounter. In your writing, use descriptive
language to bring the creature to life and create a vivid picture for the
reader. You may also include dialogue and any other relevant details
that add to the scene. As you describe the creature, explore the themes
of imagination, creativity, and the power of the mind. How do these
themes affect your experience with the creature? What do you learn
about the creature and its world? How does your encounter with the
creature change you?
la
Topic 6: Write a creative essay about a new student who has just moved
to a new school. In your essay, include instances of personification,
metaphor, and dialogue that increases suspense.Your essay should be
approximately 500 words.
ho

Topic 7: Write a 500-word creative writing piece set in a haunted


mansion. Include at least 5 instances of personification and 10 instances
of metaphor.

Topic 8: Write a narrative essay about a magical moment you


Sc

experienced in the city at night. Use extensive emotional vocabulary,


metaphors, and personification to bring the moment to life, and include
dialogue to increase the sense of drama and excitement.

XXXII
09

Topic 9: You have been chosen to be the first human to visit a distant
planet, a place that is completely unknown to you. As you step off the
spaceship and onto the alien planet, you are struck by the strangeness of
your surroundings. The air is thin and dry, the sky a deep shade of purple.
The ground is rocky and barren, the only life coming from strange and
otherworldly plants. You soon realise that you are not alone on the
planet, that there are other beings living there. Write a story about your
adventures on the alien planet.

Topic 10: Imagine you are a young detective trying to solve a string of
burglaries in your neighbourhood. Write a short story about your

rly
investigation, including at least 3 instances of personification and 5
instances of metaphor. Be sure to include dialogue with suspects and
clues that lead to the eventual capture of the thief.
la
ho
Sc

XXXIII
ESSAY EXAMPLES
Topic 1:

As the sun began to set on the horizon, I found myself in a desperate situation. I had been
stranded at sea for days, my small boat tossed and turned by the turbulent waves. The old,
wretched man had seen better days, his wooden planks worn and weathered by the relentless
saltwater. My supplies of food and water had run out, and my energy was all but depleted.
The sea around me was a chaotic blend of conflicting emotions, its surface a tapestry of
constantly shifting shades. At times it was as smooth as glass, its waters as clear and still as a
diamond. At other times it was a cauldron of fury, its surface frothing and seething like a
boiling pot of jade.
My senses were overwhelmed by the sights, smells, and sounds of the sea. The salty tang of the
ocean filled my nostrils, the taste of saltwater on my tongue. I heard the crashing of the waves
against the boat, the howling of the wind through the sails. I saw the endless expanse of water

rly
before me, the sparkling surface broken only by the occasional cresting wave. I felt the rough,
splintered wood of the boat beneath me, the spray of the saltwater on my skin.
As the nightfall approached, my thoughts turned to my home and the life I had left behind. I
thought about the warm, cosy embrace of my bed, the savory aroma of my mother's cooking,
the joyous sounds of my friends. I realised with a pang of longing that I might never experience
those things again.
But as the hours ticked by, I began to feel a glimmer of hope. I remembered the stories my
father had told me about brave sailors who had faced unimaginable challenges and emerged
victorious. I knew that I had to hold onto that hope, to believe that I too could find a way to
survive.
As the dawn broke and the sun began to rise, I found myself filled with a renewed sense of
la
determination. I knew that I had to do something, that I couldn't just sit there and wait for
rescue. I had to take matters into my own hands.
So, I set about repairing the old, wretched man as best I could, patching up the holes and
strengthening the sails. I collected rainwater in my bucket and caught fish with my bare hands.
I navigated by the stars, using the constellations to guide me towards land.
And as the days passed and my energy returned, I began to feel a sense of pride and
accomplishment. I was no longer a helpless victim, but a survivor. I was no longer adrift at sea,
ho

but in control of my own destiny.


Finally, after what felt like an eternity, I saw land on the horizon. I felt a surge of joy and relief
as I realised that I had made it, that I had survived the turbulent sea and all its dangers.
As I sailed into the harbour, I was struck by the beauty of the scene before me. The water was
a deep, rich blue, like the sapphire of a king's crown. The buildings lining the shore were a riot
of colours, their roofs a mosaic of red, orange, and yellow. The air was filled with the sounds of
life, the chatter of people, the clanging of bells.
My heart swelled with emotion as I stepped off the old, wretched man and onto solid ground. I
had made it home, against all odds. I had faced my fears and my doubts, and I had emerged
Sc

stronger for it.


As I was reunited with my loved ones, I knew that I would never forget my journey. I would
always carry the memories of the sea with me, the lessons I had learned and the strength I had
found. And I knew that I would always be grateful to the old, wretched man, the faithful
companion who had carried me through the stormy waters and brought me safely home.
ESSAY EXAMPLES
Topic 2:

As I walked through the dense jungle, the trees seemed to tower above me like
sentinels, their leaves rustling in the breeze like whispers. The ground was a verdant
carpet, the grass a lush green and the flowers colourful jewels. The air was alive
with the sounds of the jungle, the chirping of birds like a choir, the buzzing of insects
like a symphony. It was a place of wonder and beauty, unlike anything I had ever
seen before.
But as I ventured deeper into the jungle, a sense of unease grew within me. The
animals seemed skittish, their gazes darting around as if they were being watched.
The air was thick with the scent of danger, the stench almost suffocating. The jungle
was breathtaking, but there was a malevolence lurking beneath its surface, a
darkness that I couldn't shake.

rly
"This place gives me the creeps," I muttered to myself, trying to shake off the feeling
of dread. "I have a bad feeling about this."
As I walked through the jungle, I felt a sense of isolation. I was an outsider here, a
stranger in a foreign land. The jungle didn't want me, and I didn't belong. But I
couldn't turn back now, not when I was so close to discovering the truth.
So I pushed on, my feet taking me deeper into the heart of the jungle. I followed the
whispers and rumors, the clues that led me to the truth. And as I peered into the
shadows, I saw it. The jungle was a place of darkness and deceit, a place where
secrets and lies lurked. It was a place of danger, where no one was safe.
la
I knew then that I had to leave the jungle, to escape before it was too late. I turned
and ran, my heart pounding with fear and desperation. I don't know how I made it
out alive, but I did. And as I left the jungle behind, I vowed never to return. The
secrets of the jungle would stay hidden, buried beneath its trees and vines.
But even as I left the jungle behind, I knew that I would never forget it. The jungle
had left its mark on me, a scar that would stay with me forever. The jungle was a
ho

place of danger and darkness, a place that I had been lucky to escape. And as I
made my way home, I knew that I had been changed by my time in the jungle. The
secrets and horrors that I had witnessed there would stay with me always, a
reminder of the dangers that lurked in the shadows.
I clutched my chest, panting and sweating as I stumbled out of the jungle. "I can't
believe I made it out alive," I gasped, collapsing to the ground. "I never want to go
back there again."
But even as I vowed never to return, a part of me knew that the jungle would always
have a hold on me. The memories of my time there would stay with me forever, a
Sc

reminder of the strength and resilience that I had found within myself. The jungle
may have been a place of danger and darkness, but it had also been a place of
growth and self-discovery. And for that, I would always be grateful.
ESSAY EXAMPLES
Topic 3:

As I stood atop the peak, the gusts of wind tore through my locks and the rays of the sun
pounded down on my countenance. The vista before me was nothing short of
breathtaking, the panorama unfurling in all its splendour. Beneath me, the valley was a
verdant expanse, the trees a tapestry of jade and the flowers a mosaic of hues. The sky
was an azure abyss, the clouds a canvas of cotton.
But as I marvelled at the beauty before me, something peculiar occurred. The earth
beneath me began to tremble and shake, the mountain quivering and the sky turning
ashen. I heard a deafening rumble, the noise reverberating throughout the valley. And
then, without warning, I was falling.
I tumbled through the air, the gusts of wind rushing past me as I plummeted towards the
ground. I shut my eyes, bracing myself for the collision. But it never came. Instead, I
found myself floating, weightless and liberated.

imagination.
rly
I opened my eyes, and that's when I saw it. The world around me was a blur of colours, a
prism of light and sound. I was in a place that was indescribable, a place that was pure

"Where am I?" I muttered to myself, my voice echoing through the dreamscape.


"You are in the realm of dreams," a soft, ethereal voice replied.
I turned to see a being floating before me, its body a swirl of colours and its eyes two
orbs of light.
"Who are you?" I asked, my voice filled with wonder.
"I am the keeper of this world," the being replied. "I am the one who guides you through
la
the dreamscape."
As I conversed with the keeper, I learned more about the strange and incredible world
that I had found myself in. I discovered that the dreamscape was a place of endless
possibility, a place where anything was possible. I saw how the unconscious mind could
shape and influence the world around us, how it could create and destroy. I saw the
beauty and the darkness that lurked within us, the light and the shadows.
ho

Through my interactions with the keeper, I began to understand the themes of


imagination, creativity, and the power of the mind. I saw how they were all connected,
how they all influenced one another. I saw how they all played a role in my experience
in the dream.
As I reflected on these themes, I realised that they had affected me in ways that I could
never have imagined. My imagination had been awakened, my creativity ignited. I had
gained a new perspective on the world, a deeper understanding of myself and my
unconscious mind.
"I have to go back now," I said, a sense of longing in my voice.
Sc

"You can return whenever you wish," the keeper replied. "The dreamscape will always
be here, waiting for you."
And with that, I felt myself being pulled back into the waking world. As I left the dream
behind, I knew that I had been changed by it. I carried the lessons and the insights that I
had gained with me, a part of me forever. The dream had touched me in a way that
nothing else ever had, and I knew that I would always be grateful for the experience.
ESSAY EXAMPLES
Topic 4:

As I stepped through the portal, the world around me was transformed. I was no
longer in the present, but in a time and place that was beyond my wildest
dreams. The air was filled with the sounds of a bygone era, the clashing of
swords and the clanging of armour. The ground was rocky and rough, the earth
beneath me ancient and unchanged.
I looked around, taking in the sights and sounds of the past. I saw the towering
castles, their walls adorned with banners and their gates guarded by knights. I
saw the sprawling cities, their streets teeming with life and their markets
bustling with activity. I saw the lush forests, their trees reaching towards the sky

rly
and their streams gurgling with life.
But as I marvelled at the wonders before me, something peculiar occurred. I
felt a tug on my sleeve, a gentle pull. I turned to see a being standing before
me, its body a blur of colours and its eyes two orbs of light.
"Come with me," the being said, its voice as gentle as a breeze.
Without hesitation, I followed the being through the portal and into the
unknown. As we journeyed through time and space, I encountered all manner of
strange and wondrous things. I saw the rise and fall of empires, the birth and
death of civilisations. I saw the march of progress, the evolution of humanity.
la
Through my travels, I explored the themes of adventure, exploration, and the
unknown. I saw how they were all connected, how they all influenced one
another. I saw how they all played a role in my journey through time and space.
As I reflected on these themes, I realised that they had affected me in ways
that I could never have imagined. My curiosity had been piqued, my sense of
wonder ignited. I had gained a new perspective on the world, a deeper
understanding of the past, present, and future.
ho
Sc
ESSAY EXAMPLES
Topic 5:

As I walked through the enchanted forest, I couldn't believe my eyes. Before me


stood a creature unlike any I had ever seen. It was a majestic beast, its body a
tapestry of shimmering scales and shimmering feathers. Its wings were a blur of
colour, their tips touched with gold. Its eyes were two pools of liquid silver, their
depths filled with knowledge and mystery.
I was awestruck by the creature's beauty and grace. I stood there, mesmerised
by its presence. I couldn't help but feel a sense of awe and wonder, as if I was
in the presence of something truly magical.
"Greetings, human," the creature said, its voice like a symphony of sound. "What
brings you to my forest?"
rly
I stumbled over my words, my tongue tied in knots. I had never spoken to a
creature like this before, and I wasn't sure how to respond.
"I-I'm just exploring," I stammered. "I didn't mean to intrude."
The creature chuckled, its laughter like the tinkling of bells.
"You are not intruding," it said. "You are welcome here. My name is Kael, and I
am the guardian of this forest."
As Kael and I spoke, I explored the themes of imagination, creativity, and the
power of the mind. I saw how they were all connected, how they all influenced
la
one another. I saw how they all played a role in my encounter with the
fantastical creature.
Through my conversation with Kael, I learned about its world and its ways. I
learned about the magic that flowed through the forest, the secrets that were
hidden within its trees. I learned about the creatures that lived within its
bounds, their dreams and their fears.
As our conversation came to an end, I realised that my encounter with Kael had
ho

changed me. I had gained a new perspective on the world, a deeper


understanding of the magical and the fantastical. I had discovered a sense of
wonder that I had never known before.
As I said goodbye to Kael and continued on my journey, I knew that I would
always treasure the memory of my encounter with the fantastical creature. It
was a moment that would stay with me always, a moment that had opened my
eyes to the endless possibilities of the world.
Sc
ESSAY EXAMPLES
Topic 6:

As the sun rose on the first day of school, the new student felt a sense of
foreboding wash over her. She had just moved to a new town, and she didn't
know anyone at her new school.

As she walked up the grand staircase and through the ornate front doors of
the school, the smell of freshly cut grass and new textbooks filled her nostrils.
The marble floor was cool and smooth beneath her feet, the chandeliers
above her sparkling in the morning light. The sound of students' laughter and
excited chatter filled the air.

rly
The new student felt a pang of apprehension as she looked around at the
sea of unfamiliar faces. She felt small and out of place, like a flower in a
field of weeds. She had always struggled to fit in, and she feared that this
new school would be no different.

As she made her way to her first class, the new student couldn't help but
compare her old school to this one. Her old school had been quaint and
homely, with warm teachers and friendly students. This new school was grand
la
and imposing, with soaring ceilings and opulent decor.

"Don't be nervous, dear," said a kind-faced woman as she passed by. "We're
all one big family here at Westfield Prep. You'll fit right in in no time."

The new student tried to force a smile, but inside she was filled with doubt.
ho

How could she possibly fit in with all these wealthy, sophisticated students?
She didn't belong here, she was sure of it.

As she took her seat in her first class, the new student felt her anxiety reach a
fever pitch. She was surrounded by perfectly coiffed and dressed classmates,
their designer bags and shoes a stark contrast to her own worn backpack
and sneakers. The teacher, a no-nonsense woman with a sharp bob and red
lipstick, began to speak, but the new student couldn't focus on her words. She
was too consumed by her own thoughts, her own feelings of inadequacy.
Sc
She couldn't help but wonder if she would ever find her place in this new school.
Would she make friends, or would she always be an outsider? Would she excel
academically, or would she struggle to keep up with the expectations of her
peers?

As the day went on, the new student found herself struggling to keep up with the
fast-paced, competitive environment of Westfield Prep. She was constantly
feeling overwhelmed, her confidence shattered by the constant comparisons to
her successful classmates.

But as the final bell of the day rang, the new student couldn't help but feel a
sense of relief. She had survived her first day at a new school, but she knew that
there were many more challenges ahead.

rly
As she walked home, the new student couldn't shake the feeling of not
belonging. She knew that she had to find a way to make this new school her
own, to find her place in the world. But she also knew that it wouldn't be easy,
that it would take time and effort.

But as the sun set on the horizon, the new student felt a glimmer of hope. She
knew that she had the strength and determination to overcome her challenges,
to find her place in this new school. She vowed to work harder, to push herself
out of her comfort zone and try new things.
la
As she walked through the front doors of her home, the new student knew that
she had a long road ahead of her. But she was ready for the journey, ready to
face whatever challenges came her way. She was determined to find her place
at Westfield Prep, to prove to herself and to everyone else that she belonged.
ho
Sc
ESSAY EXAMPLES
Topic 7:

As I walked up the winding drive to the haunted mansion, I couldn't shake the
feeling that I was being watched. The mansion loomed above me, its walls and
windows dark and foreboding. The air was thick with the scent of decay and
rot, the stench almost overpowering. The ground was slippery beneath my feet,
the leaves and branches giving way with each step. I could hear the sound of
rustling in the underbrush, the noise like a whispered warning.
I knew I shouldn't be there, that I was trespassing on sacred ground. But I
couldn't resist the pull of the mansion, the mystery and secrets it held. I had to
know what had happened there, what had caused the mansion to be

heart of the mansion. rly


abandoned and left to rot. So, I pushed on, my feet taking me closer to the

As I approached the front door, I hesitated. The door was old and weathered,
the wood rotted and peeling. The handle was cold and slick beneath my hand,
the metal rusted and corroded. But I couldn't turn back now, not when I was so
close to discovering the truth about the mansion. So, I pushed the door open
and stepped inside.
la
The inside of the mansion was even more decrepit than the outside, the floors
creaky and the walls peeling. The furniture was covered in cobwebs, the once
elegant pieces now dusty and forgotten. The air was thick with the smell of
decay and neglect, the stench almost overpowering. As I walked through the
empty rooms, I couldn't shake the feeling that I was being watched. I could
feel eyes on me, the presence of the ghosts that haunted the mansion
palpable.
ho

I knew I shouldn't be there, that I was trespassing on sacred ground. But I


couldn't resist the pull of the haunted mansion, the mystery and secrets it held.
I had to know what had happened there, what had caused the mansion to be
abandoned and left to rot. So, I pushed on, my feet taking me deeper into the
heart of the mansion.

As I descended into the basement, the feeling of unease intensified. The air
Sc

was cold and damp, the walls covered in mold. The sound of dripping water
echoed through the room, the sound eerie and unsettling. I couldn't shake the
feeling that I was being watched, that something was lurking in the shadows. I
tried to turn back, to retrace my steps, but it was too late. I was trapped, a
prisoner in the haunted mansion.
As I stood in the dark and musty basement, I couldn't help but wonder what
secrets the mansion held. What had happened there, to cause it to be
abandoned and left to rot? What were the stories of the people who had
once called it home? I could feel the weight of the mansion's history pressing
down on me, the ghosts of the past still clinging to the walls.

But as I stood there, shivering and alone, I realised that I wasn't alone. There
was something else in the basement with me, something dark and malevolent. I
could feel its presence, could sense its malevolent gaze upon me. My heart
raced as I turned to face it, my mind racing with fear.

I don't know how long I stood there, locked in a battle of wills with the dark
presence. Time seemed to stand still as I faced off against it, my fear and
determination a match for its malevolence. In the end, I don't know who won
the battle. All I know is that when I finally stumbled out of the haunted mansion,
I was a changed person. The secrets of the mansion had left their mark on me,

rly
and I knew that I would never

Topic 8:

As I stood on the rooftop, the city stretched out before me in all its sparkling
glory. The night sky was alive with the magic of fireworks, the bursts of colour
and light illuminating the darkness. The air was thick with the scent of
gunpowder and smoke, the stench almost overpowering. The ground was hard
la
and cold beneath my feet, the concrete unforgiving. " Have you ever seen
anything so beautiful?" I heard a voice beside me say. I turned to see a young
boy, his eyes wide with wonder as he watched the fireworks.

"Never," I replied, my own eyes fixed on the display. "It's like a dream come
true." As the fireworks reached their crescendo, the sky was alive with colour.
ho

The sparks rained down like a shower of diamonds, the light almost blinding.
The explosions boomed and echoed, the sound almost deafening. And as the
final firework burst in the sky, I couldn't help but feel a sense of awe and
wonder. The magic of the fireworks had touched me, had left its mark on my
soul. "Wow," the boy breathed, a look of pure amazement on his face. "That
was amazing. I've never seen anything like it."

"It was something special," I agreed, a smile spreading across my face.


"Something that I'll never forget." As the last sparks faded from the sky, I
Sc

turned and began my descent from the rooftop. The city was eerily quiet, the
magic of the fireworks a distant memory. But as I drifted off to sleep that
night, the memory of the spectacular display stayed with me. The fireworks
were a reminder of the magic and mystery that existed in the world, a
reminder that anything was possible. And as I drifted into a peaceful slumber, I
knew that I would always hold the magic of the fireworks close to my heart.
ESSAY EXAMPLES
Topic 9:

As I stepped off the spaceship and onto the alien planet, I couldn't believe my eyes. The
air was thin and dry, the sky a deep shade of purple. The ground was rocky and barren,
the only life coming from strange and otherworldly plants. It was like nothing I had ever
seen before, a place that was completely unknown to me. "Welcome to Zorath," said a
voice behind me. I turned to see an alien standing there, its skin a bright shade of green.
"I am Zorathian, the leader of this planet. You are the first human to visit our world, and
we are honoured to have you here."

I couldn't believe what I was hearing. I had always dreamed of exploring other worlds,
but I never thought it would actually happen. I had always been told that aliens were

rly
just science fiction, that they didn't really exist. But here I was, standing on an alien
planet, being greeted by an alien leader.
"Thank you," I stammered, still trying to take it all in. "I'm honoured to be here as well."
Zorathian smiled, its mouth stretching wide. "We have much to show you," it said. "Come,
follow me. I will introduce you to our world."

I followed Zorathian as it led me through the streets of the alien city. The buildings were
tall and spired, the architecture unlike anything I had ever seen. The air was filled with
the sound of strange and alien languages, the words like music to my ears. The streets
were crowded with all sorts of aliens, each one more bizarre and unique than the next.
la
As we walked, Zorathian pointed out the various landmarks and attractions of the city.
There was a massive alien market, filled with all sorts of strange and exotic goods.
There was a museum dedicated to the history of the planet, its walls lined with artifacts
and relics from ages past. And there was a park, filled with strange and otherworldly
plants and animals.
But the most amazing thing of all was the spaceship that Zorathian showed me. It was
ho

massive and sleek, its hull gleaming in the sun. It was unlike anything I had ever seen
before, a true testament to the technological advancements of the alien race.
As we walked back to Zorathian's home, I couldn't help but feel overwhelmed by
everything I had seen and experienced. This alien planet was like nothing I had ever
imagined, and I knew that I would never forget my time here.
"Thank you for showing me around," I said to Zorathian as we reached its front door. "I
had no idea that places like this even existed."
Zorathian smiled again, its eyes twinkling. "There is so much more to explore in the
universe," it said. "Who knows what other amazing places and creatures you will
Sc

discover in your travels."

As I stepped back onto the spaceship, I knew that I had to come back to this alien
planet someday. There was so much more to see and discover, and I couldn't wait to
explore it all.
ESSAY EXAMPLES
Topic 10:

As I walked through the quiet streets of my neighbourhood, I couldn't shake the


feeling that something was off. The usually peaceful cul-de-sac was filled with
tension, the air thick with the fear of the unknown. I had heard about the
burglaries that had been plaguing the neighbourhood, and I knew that it was up
to me to put an end to them.
I was determined to catch the thief and bring them to justice, to restore the
peace and safety to my community. So, I set to work, my detective skills on high
alert.
I began by interviewing the victims of the burglaries, trying to gather any clues

rly
or information that might help me solve the case. Each person I spoke with had
their own story, their own theories on who the thief might be. Some thought it
was an outsider, someone who had come into the neighbourhood with the sole
purpose of stealing from them. Others suspected it was someone they knew,
someone who had a grudge or a desire for revenge.
As I listened to each person's story, I couldn't help but feel a sense of frustration.
The thief seemed to be one step ahead of me, always managing to evade
capture and slip through the cracks. I knew that if I was going to catch them, I
would have to think outside the box, to use my detective skills to the fullest.
la
So, I began to scour the neighbourhood for clues, my eyes and ears open for
anything that might lead me to the thief. I examined every piece of evidence,
every hint or clue that I came across. I questioned every suspect, trying to get
them to slip up and reveal the truth.
Finally, after days of tireless investigation, I had a breakthrough. I had pieced
together all the clues, and I knew exactly who the thief was. It was someone I
ho

had never suspected, someone who had been right under my nose the whole
time.
I confronted the thief with my evidence, and they were unable to deny the truth.
They tried to run, to escape justice, but I was one step ahead of them.
Sc

**Congratulations on completing the


textbook! Goodluck with your exam!**
rly
la
ho

Want free materials and guidance?

Scan the code below:


Sc
Scholarly
Steve Xu
Scholarly Publishing
TABLE OF CONTENTS

v Chapter 1: Introduction to Persuasive Writing

rly
Definition of persuasive writing
Importance of persuasive writing in everyday life
Examples of persuasive writing (advertising, politics,
social media)

vii Chapter 2: Identifying the Audience


Understanding the needs, values, and beliefs of the audience
la
Tailoring the persuasive argument to the specific audience

ix Chapter 3: Structuring the Persuasive Argument


Introduction: hook the reader and state the main argument
Body: present evidence and examples to support the argument
ho

Counterarguments: address opposing viewpoints and provide


rebuttals
Conclusion: restate the main argument and call to action

xi Chapter 4: Language Techniques in Persuasive Writing

Rhetorical questions
Sc

Emotional appeal
Repetition
Hyperbole
Loaded language

iii
xiv Chapter 5: Persuasive Writing in the Digital Age

The role of social media in persuasive writing


Ethics of online persuasive writing
Tips for effective online persuasion (e.g. using visuals, creating a

rly
strong online presence)

xvi Chapter 6: Conclusion


Recap the importance and techniques of persuasive writing
Encourage students to continue developing their persuasive
writing skills.

xvii Chapter 7: Persuasive Writing Exercises


la
Practice writing persuasive essays on various topics
ho
Sc

iv
1 Chapter 1:
Introduction to
Persuasive Writing

rly
Definition of persuasive writing

Persuasive writing is a type of writing that aims to convince the


reader to adopt a particular point of view or take a specific action. It
is often used in advertising, politics, and social media to influence
people's beliefs, attitudes, and behaviours.
la
ho

Importance of persuasive writing in everyday life:

Persuasive writing is an important skill to have in today's world because it


Sc

allows us to effectively communicate our ideas and opinions and


persuade others to take action. It is used in various settings, such as
writing a letter to a government official to advocate for a cause, creating
an advertisement to sell a product, or writing a social media post to raise
awareness about an issue.

v
Examples of
persuasive writing:

A social media post that A political campaign ad that


A commercial advertisement
encourages people to donate tries to convince voters to
that aims to persuade
to a charity or sign a petition support a particular
consumers to buy a product
candidate

rly
Example:
Writing Prompt: Think of a current event or issue that you are passionate about. Write a persuasive
paragraph explaining why others should care about this issue and what they can do to help.

Essay example:
la
One current event that I am passionate about is the climate crisis. I believe that this is an issue that
everyone should care about and take action on, as it affects not only our own future, but the future
of the entire planet.
There are many reasons why others should care about the climate crisis. First and foremost, the
effects of climate change are already being felt around the world. From rising sea levels to more
frequent and severe natural disasters, the impacts of climate change are destructive and far-
reaching. If we do not take action to address this crisis, the consequences will only continue to
worsen.
In addition to the direct impacts of climate change, there are also numerous indirect consequences
ho

that will affect our daily lives. For example, as temperatures rise, food production and water
availability will be impacted, leading to potential shortages and higher prices. The climate crisis also
has the potential to cause conflicts and mass migration, as people are forced to flee from areas that
are no longer habitable.
There are many things that individuals can do to help address the climate crisis. One simple yet
effective action is to reduce our carbon footprint by using less energy and resources. We can also
support politicians and policies that prioritise the environment and advocate for the adoption of
renewable energy sources. Finally, we can raise awareness about the importance of addressing the
climate crisis and encourage others to take action.
In conclusion, the climate crisis is an issue that everyone should care about and take action on. The
impacts of climate change are destructive and far-reaching, and there are numerous indirect
consequences that will affect our daily lives. By taking steps to reduce our carbon footprint and
supporting politicians and policies that prioritise the environment, we can work towards a brighter
Sc

and more sustainable future for all.

vi
2 Chapter 2:
Identifying the
Audience
Understanding the needs, values, and beliefs of the audience:
When writing persuasively, it is important to consider the

rly
audience's perspective and tailor the argument to their needs,
values, and beliefs. This means understanding what they care
about, what they are interested in, and what motivates them. By
aligning your argument with the audience's values and interests,
you can more effectively persuade them.
la
ho

Tailoring the persuasive argument to the specific audience:


In addition to understanding the audience's needs, values,
and beliefs, it is also important to consider their level of
knowledge about the topic and adjust the argument
accordingly. For example, if you are writing to a general
audience who may not be familiar with the topic, you may
Sc

need to provide more background information and explain


technical terms. On the other hand, if you are writing to an
audience of experts, you can assume a higher level of
knowledge and focus more on the specific points of your
argument.

vii
Writing Prompt: Imagine you are writing a persuasive

Example letter to your school board about the importance of


increasing funding for music education. Identify the
needs, values, and beliefs of the school board and
explain how your argument aligns with these.

Essay example

rly
Dear School Board Members,

I am writing to you today to urge you to increase funding for music education in our schools. As
students, we believe that music education is not only an important part of our education, but it
also has numerous benefits that align with the needs, values, and beliefs of our school district.
One of the main needs that music education addresses is the need for a well-rounded education.
Music education provides students with the opportunity to learn about different cultures,
histories, and art forms, which helps to broaden their horizons and develop a deeper appreciation
for the world around them. It also helps to enhance other areas of learning, such as math and
language arts, as it requires students to develop critical thinking and problem-solving skills.
In addition to addressing the need for a well-rounded education, music education also aligns with
the values and beliefs of our school district. For example, music education promotes teamwork
and collaboration, as it requires students to work together in order to create a cohesive
la
performance. It also fosters creativity and self-expression, which are values that are important to
our school district.
There are also numerous emotional and social benefits to music education. Music has the power to
bring people together and to create a sense of community, and it can also serve as a form of
therapy and stress relief. As we all know, the past year has been especially challenging, and it is
more important than ever that we provide students with the tools to cope with stress and to
connect with others.
Furthermore, there is a growing body of research that supports the benefits of music education.
According to a study by the National Association for Music Education, students who participate in
music education have higher grades, better attendance rates, and are more likely to graduate from
ho

high school. As John F. Kennedy so eloquently stated, "I believe in an America where the
separation of Church and State is absolute, where no Catholic prelate would tell the President
(should he be Catholic) how to act, and
no Protestant minister would tell his parishioners for whom to vote; where no church or church
school is granted any public funds or political preference; and where no man is denied public
office merely because his religion differs from the President who might appoint him or the people
who might elect him."
In conclusion, increasing funding for music education is not only an important investment in the
future of our students, but it also aligns with the needs, values, and beliefs of our school district.
Music education promotes a well-rounded education, teamwork and collaboration, creativity and
self-expression, and has numerous emotional and social benefits. I urge you to consider the
importance of music education and to increase funding for this vital part of our students'
education.
Sc

viii
3 Chapter 3:
Structuring the
Persuasive Argument

rly
Introduction
The introduction is the first part of
the persuasive argument and its
main purpose is to hook the reader
and clearly state the main
argument. The introduction should
grab the reader's attention and
provide a clear overview of the
la
main points that will be discussed
in the essay.

Body
ho

The body of the persuasive


argument is where the writer
presents evidence and examples to
support the main argument. This
can include facts, statistics, expert
opinions, and personal anecdotes.
It is important to use credible
sources and present the evidence
Sc

in a logical and organised way.

ix
Counterarguments
In the counterarguments section,
the writer addresses opposing
viewpoints and provides rebuttals
to refute them. This shows that the
writer has considered multiple
perspectives and is able to defend
their position.

rly
Conclusion
The conclusion is the final part of
the persuasive argument and its
main purpose is to restate the main
argument and call to action. The
la
conclusion should summarise the
main points of the argument and
encourage the reader to take action
or adopt the writer's point of view.
ho

Example:

Writing Prompt: Write a persuasive essay arguing for or


against the use of standardised tests in schools. Use the
Sc

structure above to organise your essay, including an


introduction, body, counterarguments, and conclusion.

x
Essay example:

Introduction:
Standardised tests are a controversial topic in education, with some arguing
that they are necessary to measure student progress and hold schools
accountable, while others believe that they are flawed and unfairly
disadvantage certain groups of students. In this essay, I will argue that the use
of standardised tests in schools is not only unnecessary, but it also has

rly
negative consequences that outweigh any potential benefits.
Body:
There are several reasons why standardised tests are unnecessary in schools.
First, these tests do not accurately measure student learning or progress.
Standardised tests are often based on a narrow set of skills and do not take
into account the diverse learning needs and styles of individual students. They
also do not account for non-academic factors that can impact student
performance, such as anxiety, stress, or illness. As a result, standardised tests
can be misleading and do not provide a complete picture of student learning.
In addition to being inaccurate, standardised tests also have negative
consequences for students, teachers, and schools. For students, the emphasis
on these tests can lead to increased stress and pressure, which can have
negative impacts on their mental health and well-being. It can also discourage
la
students from pursuing their interests and passions, as they are encouraged to
focus solely on subjects that are tested on these exams. For teachers, the
focus on standardised tests can lead to a narrow and rigid curriculum, which
can stifle creativity and innovation in the classroom. Finally, for schools, the
reliance on standardised tests can create a culture of competition and ranking,
rather than a focus on the overall well-being and success of all students.
Counterarguments:
There are some who argue that standardised tests are necessary to hold
ho

schools accountable and ensure that students are meeting certain


benchmarks. However, there are more effective and fair ways to assess
student learning and hold schools accountable, such as using a variety of
assessments, including formative and summative assessments, and gathering
feedback from students, teachers, and parents. These methods provide a more
comprehensive and accurate picture of student learning and can be used to
support and improve student achievement.
Conclusion:
In conclusion, the use of standardised tests in schools is unnecessary and has
negative consequences for students, teachers, and schools. While some may
argue that these tests are necessary to hold schools accountable, there are
more effective and fair ways to assess student learning and support student
Sc

achievement. It is time for us to move away from the reliance on standardised


tests and towards a more holistic and inclusive approach to education.

xi
4 Chapter 4:
Language Techniques
in Persuasive Writing

rly
la
In Chapter 4, we will explore the various language techniques that can be
ho

used in persuasive writing to effectively convey a message and persuade the


reader. Persuasive writing requires the use of carefully chosen words and
structures in order to convince the reader to adopt a particular perspective
or take a specific action. By understanding and mastering these techniques,
writers can craft powerful and convincing arguments that are more likely to
sway the reader's opinion. In this chapter, we will examine techniques such
as rhetorical questions, emotional appeal, and the use of figurative
language, and we will see how they can be used to effectively communicate
a persuasive message.
Sc

xi
Rhetorical questions
Rhetorical questions are questions that do not require an answer, but are
used to make a point or emphasise a point. They can be effective in
persuasive writing because they engage the reader and encourage them to
think about the topic.
For example: "How can we justify ignoring the devastating impacts of
climate change on future generations?"

rly
Emotional appeal
Emotional appeal is the use of emotions to persuade the reader. This can be
effective because people are often more likely to be convinced by
arguments that appeal to their feelings rather than just facts and logic.

For example: "The thought of a world where our children and grandchildren
cannot enjoy the same natural wonders that we have should fill us with a
sense of urgency and responsibility to take action on climate change."
la
Figurative language
Figurative language is a technique used in persuasive writing to help make
the writing more vivid and engaging. Figurative language involves using
language that goes beyond the literal meaning of words to create a more
expressive and imaginative piece of writing. This can be done through the
ho

use of literary devices such as similes, metaphors, and personification,


which can help to paint a more vivid picture in the reader's mind and make
the writing more engaging and memorable.

For example: "Climate change is like a ticking time bomb, and we have the
power to defuse it before it is too late." (metaphor)
"Climate change is a monster that must be defeated." (personification)
Sc

Repetition
Repetition is the act of repeating a word or phrase multiple times for
emphasis. It can be effective in persuasive writing because it helps to drive
the point home and make it more memorable for the reader.

xii
Hyperbole
Hyperbole is the use of extreme exaggeration to make a point. It can be
effective in persuasive writing because it can grab the reader's attention and
make the argument more memorable.

Loaded language
Loaded language is the use of words or phrases that have strong emotional
connotations in order to persuade the reader. This can be effective because

rly
it can appeal to the reader's emotions and make the argument more
powerful.

Example
Writing prompt: Write a persuasive essay about combating climate change.

Essay example
Climate change is a pressing issue that affects every Climate change is a man-made natural disaster,
person on this planet. From rising sea levels and more and it is up to us to fix it. We must destroy the
frequent natural disasters to the loss of biodiversity and
earth in order to save it, by decreasing our carbon
la
negative impacts on human health, the consequences of
footprint and increasing our use of clean energy. As
climate change are far-reaching and devastating. It is
important that we take action now to combat climate Edmund Burke said, "The only thing necessary for
change and protect our planet for future generations. the triumph of evil is for good men to do nothing."
Climate change is the elephant in the room that we can no Climate change is both a global and a local issue,
longer ignore. It is a ticking time bomb that, if not defused, and we must all do our part to combat it. By using
will have catastrophic consequences. The earth is a less fossil fuels, we can have more energy and a
delicate balance that we must protect, and carbon
cleaner, healthier planet.
emissions are the poison that is killing our planet. We are
the guardians of the earth, responsible for its well-being.
As Leonardo DiCaprio said, "Climate change is real, it is Some people might argue that it is too expensive or
happening right now. It is the most urgent threat facing inconvenient to make these changes, but the truth
ho

our entire species, and we need to work collectively is that the cost of not taking action on climate
together and stop procrastinating." change will be far greater. Natural disasters caused
by climate change can result in billions of dollars in
There are many ways that we can combat climate change,
damages, and the health effects of air pollution
but one of the most effective is by reducing our carbon
caused by carbon emissions can also be costly. In
emissions. Carbon emissions are the main cause of climate
change, and by reducing the amount of carbon dioxide the long run, investing in clean energy and other
that we release into the atmosphere, we can help to slow climate-friendly practices can save money and
down the warming of the planet. The earth is a spaceship, protect our planet.
and we are the astronauts on board. If we continue to
pollute and degrade our home, we will not have anywhere By making small changes in our daily lives and
else to go. As Pope Francis said, "The earth is not a
supporting clean energy and other climate-friendly
bargaining chip. It is our home, the place we live and the
practices, we can make a difference and protect
only place we have."
our planet for future generations. It is up to us to
Climate change is a pressing issue that affectsThere are take responsibility and combat climate change
several ways that we can reduce our carbon emissions, before it is too late. As the earth screams for help,
Sc

including: it is up to us to listen and take action. Let's not be


Using public transportation or carpooling instead of the generation that allowed the earth to perish.
driving alone
Let's be the generation that saved it.
Planting trees and other vegetation, which absorb
carbon dioxide from the air
Using energy-efficient appliances and light bulbs
Supporting clean energy sources such as solar and
wind power

xiii
5 Chapter 5:
Persuasive Writing in
the Digital Age

The role of social media in persuasive writing

rly
Social media has become an important platform for persuasive writing, with
la
the ability to reach large audiences quickly and easily. It is important for
writers to consider the ethics of persuasive writing on social media,
including being transparent about their intentions and avoiding
manipulation or false information.
ho

Ethics of online persuasive writing


Sc

In the digital age, it is important for writers to be transparent about their


intentions and to ensure that their arguments are based on credible sources
and factual information. It is also important to be respectful of others'
viewpoints and to avoid spreading misinformation or engaging in personal
attacks

xiv
Tips for effective online persuasion

rly
Some tips for effective online persuasion include using visuals (such as
infographics or videos) to make the argument more engaging, creating a
strong online presence (such as a blog or social media account) to establish
credibility, and using social media to engage with and listen to the audience.

Example:
la
Writing Prompt: Write a persuasive social media post on why all students should be required to
learn a second language. Consider the ethics of online persuasive writing and use the tips above
to create an effective argument.

Essay example:
ho

Hi everyone!
I wanted to share my thoughts on the importance of learning a second language and why I think it
should be mandatory for all students.
First of all, learning a second language has numerous cognitive benefits. Studies have shown that it
can improve memory, problem-solving skills, and even increase your IQ. It can also make you more
culturally aware and open-minded, as learning a new language exposes you to different ways of
thinking and communicating.
Add a little bit of body text
In addition to these personal benefits, being proficient in a second language is also a valuable asset
in today's globalised world. It can open up job opportunities and make you more competitive in the
job market. It can also facilitate communication and understanding with people from different
cultural backgrounds, which is becoming increasingly important as the world becomes more
interconnected.
Sc

Finally, I believe that learning a second language is a way to promote cultural diversity and
understanding. By learning about another culture's language and customs, we can build bridges
and foster a more harmonious and inclusive society.
I strongly believe that all students should be required to learn a second language. The personal
and societal benefits are simply too great to ignore. Let's embrace the opportunity to expand our
horizons and make the world a more understanding and connected place.

xv
6 Chapter 6:

Conclusion

rly
Recap the importance and techniques of persuasive writing. In conclusion,
persuasive writing is a valuable skill to have in today's world. It allows us to
effectively communicate our ideas and opinions and to persuade others to
la
take action. To be successful in persuasive writing, it is important to
consider the audience's needs, values, and beliefs, to structure the
argument in a logical and organised way, and to use language techniques
such as rhetorical questions, emotional appeal, repetition, hyperbole, and
loaded language.
ho

Encourage students to continue developing their persuasive writing skills.


Sc

Finally, it is important for students to continue developing their persuasive


writing skills throughout their education and beyond. By practising
persuasive writing and learning from the successes and mistakes of others,
students can become more confident and effective writers.

xvi
7 Chapter 7:
Persuasive Writing
Exercises
Writing prompts:

rly
Topic 1: Write a persuasive essay explaining why zoos should be banned. In
your essay, consider the ethical concerns of keeping animals in captivity and
how zoos may negatively impact the conservation of wildlife. Use evidence
and examples to support your argument.

Topic 2: Write a persuasive essay explaining why people should adopt a


vegan lifestyle. In your essay, consider the environmental, ethical, and
health benefits of veganism. Use evidence and examples to support your
argument.
la
Topic 3: Write a persuasive essay arguing that single-use plastics should be
banned. In your essay, consider the environmental impacts of single-use
plastics and how a ban on these items can help protect the planet. Use
evidence and examples to support your argument.

Topic 4: Write a persuasive essay explaining why we should transition to


ho

renewable energy sources. In your essay, consider the environmental and


economic benefits of renewable energy and how it can help to mitigate
climate change. Use evidence and examples to support your argument.

Topic 5: Write a persuasive essay arguing that stricter gun control laws are
necessary. In your essay, consider the impact of gun violence on society and
how stricter gun control laws can help to reduce this violence. Use evidence
and examples to support your argument.
Sc

Topic 6: Write a persuasive essay explaining why all schools should have a
mandatory mental health education program.

Topic 7: Write a persuasive essay arguing that social media platforms should
be more responsible in moderating online content.

xvii
Topic 8: Write a persuasive essay explaining why fast fashion is harmful and
why we should support sustainable fashion.

Topic 9: Write a persuasive essay arguing that video games can have positive
effects on mental health and should not be demonised.

Topic 10: Write a persuasive essay explaining why we should support small
businesses instead of large corporations.

rly
Essay examples:

Topic 1:

I am filled with righteous anger and burning passion as I contemplate the atrocities
committed against animals in zoos, these "prisons of despair" where "innocent animals"
are subjected to "torment" and "suffering." I dream of a world where these "hellholes" are
dismantled, and all "creatures" are free to roam and flourish in their natural habitats, free
from the "clutches of human greed and folly." I believe that it is our "moral duty" to ban
la
zoos and to create a more compassionate and just society for all beings.
For too long, zoos have been justified under the guise of education and conservation, but
these are nothing more than "hollow excuses" for the exploitation and torment of animals.
These "death traps" are often overcrowded and poorly maintained, causing animals to
suffer from physical and mental health issues. Many zoos also engage in unethical
practices such as animal breeding and trading, which only contribute to the "suffering and
degradation" of these beautiful creatures.
ho

But the problems with zoos go beyond just the treatment of the animals. Zoos serve to
reinforce a "toxic mentality" towards animals and nature, teaching us that it is acceptable
to use and abuse these beings for our own entertainment and profit. This "toxic mentality"
is not only harmful to animals, but it also has "disastrous consequences" for the
environment and for our own well-being. As Mahatma Gandhi so eloquently stated, "The
greatness of a nation and its moral progress can be judged by the way its animals are
treated."

There is no doubt that zoos have played a role in the conservation of some species, but
there are more effective and ethical ways to protect and conserve wildlife. For example,
Sc

we can support organisations that work towards habitat preservation and protection,
rather than contributing to the captivity and exploitation of animals. We can also educate
ourselves and others about the importance of conservation and take individual actions to
reduce our impact on the environment. As Charles Darwin eloquently stated, "The love for
all living creatures is the most noble attribute of man."

xviii
Furthermore, there are many alternatives to zoos that can provide meaningful education
and conservation efforts. For example, we can visit wildlife sanctuaries or attend
educational programs that focus on the importance of protecting natural habitats and
wildlife. These alternatives allow us to learn about animals in a more ethical and
respectful way, rather than "exploiting them for our own amusement." As Albert
Schweitzer beautifully stated, "The human spirit is not dead. It lives on in secret... It has
come to believe that compassion, in which all ethics must take root, can only attain its full
breadth and depth if it embraces all living creatures and does not limit itself to mankind."

In conclusion, it is time for us to ban zoos and to create a more compassionate and just
society for all beings. Zoos are nothing more than "death traps" for animals, and they

rly
reinforce a "toxic mentality" towards nature. There are more effective and ethical ways to
educate ourselves and to protect and conserve wildlife. I am filled with hope and
determination as I dream of a day when all animals are free to roam and flourish in their
natural habitats. Together, let us work towards this dream and create a brighter future for
all.

Topic 2:

I am filled with a sense of urgency and moral outrage as I contemplate the suffering and
exploitation of animals in the animal agriculture industry. I believe that it is our moral
duty to adopt a vegan lifestyle and to work towards a more compassionate and just
la
society for all beings.

The animal agriculture industry is one of the leading causes of animal suffering and
environmental destruction on the planet. Animals in factory farms are subjected to
inhumane conditions, including confinement, mutilation, and abuse. They are treated as
nothing more than commodities, rather than sentient beings with their own interests and
desires.
ho

But the problems with animal agriculture go beyond just the treatment of animals. This
industry also has devastating consequences for the environment. Animal agriculture is a
major contributor to climate change, deforestation, and water pollution. It is
unsustainable and irresponsible to continue supporting this industry with our choices as
consumers.

However, there is a solution to these problems: veganism. By choosing to live a vegan


lifestyle, we can drastically reduce our impact on animals and the environment. Veganism
is not only a compassionate choice, but it is also a healthy and sustainable one. There are
numerous plant-based alternatives to animal products that are not only delicious, but
Sc

also provide all the nutrients that we need.

In conclusion, it is time for us to adopt a vegan lifestyle and to work towards a more
compassionate and just society for all beings. The animal agriculture industry is a major
cause of animal suffering and environmental destruction, and it is our moral duty to do
better. Veganism is a compassionate, healthy, and sustainable choice that allows us to
live in harmony with animals and the environment.

xix
Topic 3:

I am filled with a sense of urgency and disgust as I contemplate the environmental


destruction caused by single-use plastics. I believe that it is our moral duty to ban single-
use plastics and to work towards a more sustainable and responsible society.
Single-use plastics, such as straws, plastic bags, and water bottles, are a major
contributor to environmental pollution. These items are used for a short period of time,
but they have a lasting impact on the planet. They take hundreds of years to break down,
and they often end up in our oceans, where they harm marine life and ecosystems.
But the problems with single-use plastics go beyond just pollution. The production of
these items also requires fossil fuels, which contribute to climate change. In addition, the
manufacture and disposal of single-use plastics generates greenhouse gas emissions,

rly
further exacerbating the problem.

However, there is a solution to these problems: banning single-use plastics. By banning


these items, we can significantly reduce our environmental impact and protect the planet
for future generations. There are numerous alternatives to single-use plastics, such as
reusable bags, water bottles, and straws, that are not only more sustainable, but also
more convenient and cost-effective.

In conclusion, it is time for us to ban single-use plastics and to work towards a more
sustainable and responsible society. Single-use plastics are a major contributor to
environmental pollution and climate change, and it is our moral duty to do better.
la
Banning these items is a necessary step towards protecting the planet and ensuring a
healthy future for all.

Topic 4:

I am filled with a sense of urgency and hope as I contemplate the potential of renewable
energy. I believe that it is our moral duty to transition to renewable energy and to work
ho

towards a more sustainable and responsible society.

Fossil fuels, such as coal, oil, and gas, are a major contributor to climate change and
environmental degradation. The extraction, transportation, and burning of these fuels
release greenhouse gases into the atmosphere, causing global temperatures to rise and
exacerbating the problem. In addition, fossil fuels are a finite resource, and we are rapidly
approaching the point where they will no longer be able to meet our energy needs.
However, there is a solution to these problems: renewable energy. Renewable energy
sources, such as solar, wind, and hydroelectric power, are clean, abundant, and
sustainable. They do not release greenhouse gases into the atmosphere, and they can
Sc

meet our energy needs without depleting finite resources.

In conclusion, it is time for us to transition to renewable energy and to work towards a


more sustainable and responsible society. Fossil fuels are a major contributor to climate
change and environmental degradation, and it is our moral duty to do better.
Renewable energy is a clean, abundant, and sustainable alternative that allows us to meet
our energy needs without compromising the planet.

xx
Topic 5:

I am filled with a sense of urgency and sorrow as I contemplate the gun violence that
plagues our society. I believe that it is our moral duty to enact stricter gun control laws
and to work towards a safer and more just society.
Gun violence is a major problem in the United States, with thousands of people losing
their lives each year to gun-related incidents. Guns are often used to commit crimes, such
as homicides, suicides, and mass shootings, and they contribute to a culture of fear and
mistrust. In addition, the easy access to guns makes it easier for individuals to commit
violence, and it puts all of us at risk.

rly
However, there is a solution to this problem: stricter gun control laws. By enacting stricter
laws, we can reduce the number of guns in circulation, making it harder for individuals to
obtain them. We can also require background checks and training for gun ownership,
which can help to prevent guns from falling into the wrong hands. Additionally, we can
implement laws that regulate the sale and possession of certain types of guns, such as
assault weapons and high-capacity magazines, which have been used in many mass
shootings.

While some may argue that stricter gun control laws would infringe upon the rights of law-
abiding gun owners, the evidence suggests that these laws can save lives. According to a
study by the Centers for Disease Control and Prevention, states with stricter gun laws
la
have lower rates of gun violence. This suggests that common-sense gun control measures
can significantly reduce gun deaths and injuries.

In addition to the practical benefits of stricter gun control laws, there is also a moral case
to be made. Gun violence disproportionately affects marginalised and disadvantaged
communities, and it is our responsibility to take action to address this issue. By enacting
stricter gun control laws, we can work towards a more just and equitable society.
In conclusion, I believe that it is our moral duty to enact stricter gun control laws and to
ho

work towards a safer and more just society. By implementing common-sense measures,
such as background checks and regulations on certain types of guns, we can reduce gun
violence and save lives.

Topic 6:

I am filled with righteous anger and burning passion as I contemplate the mental health
crisis that plagues our society. I dream of a world where mental health is prioritised and
treated with the same importance as physical health. I believe that it is our moral duty to
implement mandatory mental health education programs in all schools and to create a
Sc

more compassionate and just society for all.

For too long, mental health has been stigmatised and neglected, leading to devastating
consequences. According to the World Health Organisation, mental health disorders are
the leading cause of disability worldwide, yet only a fraction of those who suffer receive
the treatment they need. This is especially true for young people, who often struggle with
mental health issues due to the overwhelming stress and pressure of modern society.

xxi
But the problems with mental health go beyond just the treatment of individuals. The lack
of mental health education and awareness has led to a culture of ignorance and shame
surrounding these issues. This toxic culture not only harms those who struggle with
mental health disorders, but it also has disastrous consequences for society as a whole.
By ignoring mental health, we are perpetuating a cycle of suffering and preventing people
from reaching their full potential. As the Dalai Lama so eloquently stated, "The purpose of
our lives is to be happy."

There is no doubt that mental health education can play a vital role in improving the well-
being of individuals and society. By teaching young people about mental health and how
to take care of their own well-being, we can empower them to lead happier, healthier

rly
lives. We can also educate ourselves and others about the importance of mental health
and take steps to reduce the stigma surrounding these issues. As Lady Gaga so eloquently
stated, "It's not about winning. It's about not giving up on yourself."

Furthermore, there are many alternatives to traditional mental health education that can
provide meaningful and effective support. For example, schools can implement
mindfulness practices, such as meditation and yoga, which have been proven to improve
mental health and well-being. Schools can also provide access to mental health resources
and support, such as counselling services and peer support groups. These alternatives
allow us to address mental health in a holistic and compassionate way, rather than
ignoring or stigmatising it. As the poet Mary Oliver so beautifully stated, "The most
la
regretful people on earth are those who felt the call to creative work, who felt their own
creative power restive and uprising, and gave to it neither power nor time."

In conclusion, it is time for us to prioritise mental health and implement mandatory


mental health education programs in all schools. Mental health disorders are a leading
cause of disability worldwide, and the lack of awareness and education has led to a toxic
culture of shame and ignorance. By empowering young people with the knowledge and
skills to take care of their own well-being, we can create a brighter future for all. I am filled
ho

with hope and determination as I dream of a day when mental health is treated with the
same importance as physical health. Together, let us work towards this dream and create
a better world for all.

Topic 7:

I am filled with righteous anger and burning passion as I contemplate the harm caused by
the lack of responsibility in moderating online content on social media platforms. I dream
of a world where social media is a safe and positive space for all, and where the well-being
of people is prioritized over profit. I believe that it is our moral duty to hold social media
Sc

platforms accountable and to create a more compassionate and just society.


For too long, social media platforms have prioritised profits over the safety and well-
being of their users, leading to numerous problems. The spread of misinformation, hate
speech, and online harassment on these platforms has had real-life consequences,
including violence, discrimination, and mental health issues. The lack of moderation and
accountability has also allowed for the exploitation and abuse of children and vulnerable
populations.

xxii
But the problems with social media go beyond just the negative effects on individuals.
The constant pressure to present a perfect image and the constant stream of information
can lead to a culture of comparison and anxiety. This toxic culture not only harms the
well-being of people, but it also undermines the quality of public discourse and
democracy. As the philosopher John Stuart Mill so eloquently stated, "The only way in
which a human being can make some approach to knowing the whole of a subject is by
hearing what can be said about it by persons of every variety of opinion."

There is no doubt that social media platforms have a responsibility to moderate online
content and ensure the safety and well-being of their users. By holding these platforms
accountable and demanding more transparency and responsibility, we can create a safer
and more positive online environment. We can also educate ourselves and others about

rly
the impact of social media and take individual actions to protect ourselves and others,
such as reporting abuse and setting boundaries. As the activist and writer Brene Brown so
eloquently stated, "Clear is kind. Unclear is unkind."

Furthermore, there are many alternatives to traditional social media platforms that
prioritise safety and well-being. For example, we can support social media platforms that
have stricter moderation policies and that prioritise the well-being of their users. We can
also limit our own use of social media and seek out other forms of online and offline
communication, such as face-to-face conversations and community groups. These
alternatives allow us to connect with others in a more positive and healthy way, rather
than contributing to the problems of traditional social media platforms. As the writer
la
Sherry Turkle so beautifully stated, "We live in a technological universe in which we are
always communicating. And yet we have sacrificed conversation for mere connection."

In conclusion, it is time for us to hold social media platforms accountable and to demand
more responsibility in moderating online content. The lack of moderation and
accountability has led to numerous problems, including misinformation, hate speech, and
online harassment. By holding these platforms accountable and seeking out alternatives
that prioritise safety and well-being, we can create a brighter future for all.
ho

Topic 8:

I am filled with righteous anger and burning passion as I contemplate the environmental
devastation caused by fast fashion. I dream of a world where fashion is sustainable and
ethical, and where the well-being of people and the planet is prioritised over profit. I
believe that it is our moral duty to support sustainable fashion and to create a more
compassionate and just society.
For too long, the fashion industry has been driven by a culture of excess and
Sc

disposability, leading to environmental and social disasters. The production of fast


fashion generates an enormous amount of pollution and waste, and the use of cheap
labor in developing countries often leads to exploitation and abuse. The fast fashion
industry also has a huge carbon footprint, contributing significantly to climate change.

xxiii
But the problems with fast fashion go beyond just the environmental and social impacts.
The constant pressure to consume and keep up with the latest trends leads to a culture of
materialism and waste. This toxic culture not only harms the environment and people, but
it also undermines our own happiness and well-being. As the philosopher Bertrand Russell
so eloquently stated, "Happiness is not best achieved by those who seek it directly."

There is no doubt that sustainable fashion can play a vital role in protecting the
environment and promoting social justice. By supporting sustainable fashion brands, we
can vote with our dollars and demand more ethical and sustainable practices in the
fashion industry. We can also educate ourselves and others about the impact of fast
fashion and take individual actions to reduce our own impact, such as buying secondhand
or repairing and up-cycling our clothing. As the fashion designer Vivienne Westwood so

rly
eloquently stated, "Buy less, choose well, make it last."

Furthermore, there are many alternatives to fast fashion that can provide meaningful and
stylish options. For example, we can support independent designers and small businesses
that focus on sustainability and ethics. We can also shop at thrift stores or participate in
clothing swaps, which allow us to buy and wear clothing in a more sustainable and
economical way. These alternatives allow us to express ourselves through fashion in a
more ethical and responsible way, rather than contributing to the problems of fast
fashion. As the fashion designer Stella McCartney so beautifully stated, "Fashion has the
power to change the world."
la
In conclusion, it is time for us to support sustainable fashion and to create a more
compassionate and just society. Fast fashion is a major contributor to environmental and
social problems, and it undermines our own well-being. By supporting sustainable fashion
and taking individual actions to reduce our impact, we can create a brighter future for all.
I am filled with hope and determination as I dream of a day when fashion is sustainable
and ethical. Together, let us work towards this dream and create a better world for all.
ho

Topic 9:

I am filled with a sense of urgency as I contemplate the negative stigma surrounding video
games and their potential impact on mental health. I dream of a world where video games
are recognised for their potential benefits and where all forms of media are understood
and appreciated in a nuanced and responsible way. I believe that it is our moral duty to
advocate for a more balanced and informed understanding of video games and to create
a more compassionate and just society.

For too long, video games have been unfairly demonised and blamed for various societal
Sc

problems, despite the lack of evidence supporting these claims. This negative stigma has
not only led to the marginalisation of gamers, but it has also hindered the recognition and
appreciation of the potential benefits of video games. The fact is that video games, like all
forms of media, can have both positive and negative impacts depending on how they are
used and understood.

xxiv
But the problems with the negative stigma surrounding video games go beyond just the
marginalisation of gamers. This toxic culture of fear and misinformation not only harms
those who enjoy video games, but it also undermines the potential benefits that video
games can have on mental health and well-being. As the philosopher Aristotle so
eloquently stated, "Pleasure in the job puts perfection in the work."

There is no doubt that video games can have positive effects on mental health and well-
being. Studies have shown that video games can improve cognitive skills, such as
problem-solving and spatial awareness, and can also have therapeutic benefits for those
struggling with mental health issues. By advocating for a more balanced and informed
understanding of video games, we can recognise and appreciate the potential benefits of

rly
this medium. We can also educate ourselves and others about the responsible use of
video games and the importance of finding balance in all forms of media consumption. As
the writer and game designer Jane McGonigal so eloquently stated, "We are all capable of
using our leisure time to become more resilient, more connected, and more skillful, not
only for our own personal benefit, but for the benefit of our communities and our world."

Furthermore, there are many alternatives to traditional forms of media that can provide
similar benefits to video games. For example, we can explore other interactive media,
such as virtual reality or immersive theatre, which have the potential to improve cognitive
skills and provide therapeutic benefits. We can also seek out other forms of leisure
activities that promote mental health and well-being, such as physical exercise or creative
la
hobbies. These alternatives allow us to engage with media in a more balanced and healthy
way, rather than demonising any one form. As the writer Neil Gaiman so beautifully
stated, "The one thing that you have that nobody else has is you. Your voice, your mind,
your story, your vision. So write and draw and build and play and dance and live as only
you can."

In conclusion, it is time for us to advocate for a more balanced and informed


understanding of video games and to recognise their potential benefits on mental health
ho

and well-being. The negative stigma surrounding video games not only harms those who
enjoy them, but it also undermines the potential benefits that video games can have on
individuals and society. I am filled with hope and determination as I dream of a day when
video games are recognised and appreciated for their potential benefits and when all
forms of media are understood in a nuanced and responsible way.

Topic 10:

I am filled with a sense of urgency as I contemplate the importance of supporting small


businesses in our communities. I dream of a world where small businesses thrive and
Sc

where economic stability and sustainability are prioritised over profit. I believe that it is
our moral duty to support small businesses and to create a more compassionate and just
society.

xxv
Small businesses are the backbone of our communities and the driving force behind
innovation and economic growth. These businesses provide jobs, stimulate local
economies, and offer unique and personalised products and services. In contrast, large
corporations often prioritise profits over the well-being of their employees and
communities, leading to negative impacts such as outsourcing, wage stagnation, and
environmental degradation.

But the benefits of supporting small businesses go beyond just the economic and social
impacts. By supporting small businesses, we can create a more diverse and resilient
economy and foster a sense of community and connection. This sense of community and
connection not only benefits small business owners and employees, but it also enhances

rly
our own well-being and happiness. As the sociologist Robert Putnam so eloquently
stated, "Social connections are good for us, and loneliness kills."

There is no doubt that supporting small businesses is important for the health and well-
being of our communities and ourselves. By choosing to shop at small businesses and
supporting their growth and success, we can create a more sustainable and equitable
economy. We can also educate ourselves and others about the importance of supporting
small businesses and take individual actions to reduce our impact on large corporations.
As the writer and entrepreneur Seth Godin so eloquently stated, "Small is the new big."

Furthermore, there are many alternatives to shopping at large corporations that can
la
support small businesses. For example, we can shop at locally-owned businesses, attend
small business events and markets, or invest in small business ownership. These
alternatives allow us to support small businesses in a more direct and meaningful way,
rather than contributing to the profits of large corporations. As the writer and activist
Annie Leonard so beautifully stated, "The only way to create a just and sustainable world
is to use our purchasing power to create the world we want to see."

In conclusion, it is time for us to support small businesses and to create a more


ho

compassionate and just society. Small businesses are the backbone of our communities
and the driving force behind innovation and economic growth. By supporting small
businesses and taking individual actions to reduce our impact on large corporations, we
can create a brighter future for all. I am filled with hope and determination as I dream of a
day when small businesses thrive and our communities are stronger and more connected.
Sc

xxvi
Why we should ban single-use plastics
The importance of renewable energy sources
The benefits of a plant-based diet
The case for gun control measures
The value of a liberal arts education
The need for comprehensive sex education in schools

rly
Why we should support small businesses instead of large
corporations
The benefits of learning a second language
The case for net neutrality
Why we should ban animal testing
The importance of mental health awareness and support
The need for better access to affordable healthcare
The value of arts education in schools
The case for universal basic income
The benefits of outdoor play and nature education for children
The need for more stringent environmental regulations
la
The importance of equal pay for equal work
Why we should support local agriculture and food systems
The value of diversity and inclusivity in the workplace
The case for free public college education
The benefits of mindfulness and meditation
The need for more affordable housing options
The importance of voting and political participation
The case for more stringent animal cruelty laws
ho

The benefits of volunteerism and community service


The need for better access to mental health services
The importance of equal rights and protections for the LGBTQ+
community
The case for a higher minimum wage
The benefits of a healthy work-life balance
The need for better access to clean water and sanitation
The importance of protecting and preserving natural habitats
The case for a more progressive tax system
The benefits of a tobacco-free lifestyle
The need for more stringent consumer protection laws
Sc

The importance of a free and independent press


The case for drug policy reform
The benefits of a diverse media landscape
The need for stronger consumer privacy protections

xxvii
The importance of equal access to education
The case for ending the war on drugs
The benefits of a healthy and active lifestyle
The need for more stringent consumer safety regulations
The importance of protecting and preserving cultural heritage
The case for campaign finance reform

rly
The benefits of a healthy and varied diet
The need for better access to affordable and healthy food options
The importance of protecting and preserving the environment
The case for a more just and equitable criminal justice system
The benefits of a strong and supportive community
The need for better access to quality childcare options
The importance of protecting and preserving animal rights
The case for ending police brutality and systemic racism
The benefits of a clean and healthy living environment
The need for better access to affordable and reliable transportation
The importance of protecting and preserving human rights
la
The case for a more transparent and accountable government
The benefits of a strong and stable economy
The need for better access to affordable and reliable internet
access59. The importance of protecting and preserving digital privacy
The case for a more sustainable and environmentally-friendly way of
life
The benefits of a strong and well-funded public education system
The need for better access to affordable and high-quality healthcare
ho

for all
The importance of protecting and preserving indigenous cultures and
rights
The case for a more just and equitable immigration system
The benefits of a strong and stable democracy
The need for better access to affordable and high-quality housing
options
The importance of protecting and preserving civil liberties
The case for a more just and equitable economic system
The benefits of a strong and vibrant arts and culture scene
The need for better access to affordable and high-quality mental
Sc

health services
The importance of protecting and preserving the rights of
marginalised and disadvantaged groups
The case for a more just and equitable foreign policy

xxviii
The benefits of a strong and resilient infrastructure
The need for better access to affordable and high-quality public
transportation
The importance of protecting and preserving freedom of expression
The case for a more just and equitable healthcare system
The benefits of a strong and healthy natural environment

rly
The need for better access to affordable and high-quality clean
energy sources
The importance of protecting and preserving the rights of workers
The case for a more just and equitable social security system
The benefits of a strong and diverse economy
The need for better access to affordable and high-quality education
for all
The importance of protecting and preserving the rights of women
The case for a more just and equitable immigration system
The benefits of a strong and stable government
The need for better access to affordable and high-quality technology
la
and communication systems
The importance of protecting and preserving the rights of children
The case for a more just and equitable welfare system
The benefits of a strong and supportive community
The need for better access to affordable and high-quality recreation
and leisure options
The importance of protecting and preserving the rights of people with
disabilities
ho

The case for a more just and equitable criminal justice system
The benefits of a strong and healthy natural environment
The need for better access to affordable and high-quality mental
health services
The importance of protecting and preserving the rights of indigenous
peoples
The case for a more just and equitable economic system
The benefits of a strong and vibrant arts and culture scene
The need for better access to affordable and high-quality housing
options
The importance of protecting and preserving the rights of animals
Sc

The case for a more just and equitable political system


The importance of protecting and preserving the rights of women

xxix
rly
la
ho

Want free materials and guidance?

Scan the code below:


Sc
Scholarly
Steve Xu
Scholarly Publishing
TABLE OF CONTENTS

rly
Chapter 1: Introduction
Introduction to the Vocabulary book
IV

Vocabulary memorisation tips and tricks

Chapter 2: Words to describe people


Eccentric Gregarious
la
Quirky Aloof
Outgoing Solitary
VIII
Introverted Complicated
Pensive Nostalgic

Chapter 3: Words to describe emotions


ho

Elated Sullen
Dejected Despondent
XII
Enraged Overjoyed
Ecstatic Euphoric
Melancholic Disheartened

Chapter 4: Words to describe actions


Gesticulate Prowl
Frolic
Sc

Saunter
Lurk Tiptoe
XVI
Scurry Sprint
Slither Crawl

ii
rly
TABLE OF CONTENTS
XX Chapter 5: Words to describe places
Vast Glamorous
Barren Dreary
Idyllic
la
Lively
Magical Atmospheric
Gloomy Oppressive

XXIII Chapter 6: Words to describe events


Intimate Exuberant
Elegant Monotonous
ho

Pompous Raucous
Rambunctious Lavish
Whimsical Grandiose

Chapter 7: Words to describe things


Mundane Luxurious
XXVII Gorgeous Dilapidated
Plain Exquisite
Sc

Exotic Tawdry
Ordinary Opulent

XXXI Chapter 8: Quizzes

iii
1
INTRODUCTION
Chapter 1
Introduction
Welcome to our vocabulary textbook! This book is designed
specifically for 12 year olds and is filled with fun and exciting ways to
learn new words.

In this book, you'll find chapters on words to describe people,


emotions, actions, places, events, and things. We've included a variety

rly
of words in each chapter to help you expand your vocabulary and
communicate more effectively.

One of the most important aspects of learning new words is


understanding their meanings. That's why we've included clear
definitions for each word in the book. We've also included mnemonics,
which are memory tricks that can help you remember the meanings of
new words. For example, if you're trying to remember the meaning of
the word "raucous," you might think of the phrase "rowdy and loud,"
la
which sounds similar to "raucous."

In addition to definitions and mnemonics, we've also included word


play and funny pop culture examples to help you remember and use
the words in the right context. For example, if you're trying to
remember the meaning of the word "exquisite," you might think of the
phrase "delicate and beautiful," which sums up the meaning of the
ho

word. You might also think of a character from a TV show or movie


who might use the word "exquisite" to describe something they love,
such as Professor Albus Dumbledore from the Harry Potter series.

But learning new words isn't just about memorisation. It's also about
being able to use them correctly in different situations. That's why
we've included a variety of exercises and questions in this book to
help you test your knowledge and improve your vocabulary skills.
Sc

For example, you'll find multiple choice questions that will ask you to
choose the word that means something specific, such as "very private,
personal, and close." You'll also find questions that ask you to
complete a sentence with the appropriate word

IV
01
No matter what your current
vocabulary level is, we hope this
such as "The actor was
book will help you expand your
known for his ability to
knowledge and communicate
his hands
more effectively.
and face to express a
wide range of emotions." So let's get started!

Vocabulary memorisation tips and tricks

rly
Memorising vocabulary can be tough, especially if you're trying to
learn a lot of new words at once. But don't worry! There are plenty of
tips and tricks you can use to help you remember new words and
improve your vocabulary skills.

One of the most effective ways to memorise vocabulary is to use


mnemonics. Mnemonics are memory tricks that can help you

INTRODUCTION
associate new words with familiar things or ideas. For example, if
you're trying to remember the meaning of the word "exquisite," you
la
might think of the phrase "delicate and beautiful," which sums up the
meaning of the word.

Another helpful tip is to use flashcards. Flashcards are small cards


that have a word on one side and the definition on the other. You
can use flashcards to test yourself on new words and see how well
ho

you remember their meanings. You can also make your own
flashcards using index cards or a flashcard app on your phone or
computer.

Another way to improve your vocabulary is to read as much as you


can. Reading exposes you to a wide range of words and helps you
understand how they are used in different contexts. You can also try
using a dictionary to look up words you don't know and read the
definitions carefully. This will help you understand how the word is
Sc

used and how to use it correctly.

V
01
INTRODUCTION
Additional tips and advice!
1. Practise using new words in context. Try incorporating the new words
you're learning into your writing or speaking. This will help you
understand how the words are used and give you more opportunities to
practise using them correctly

2. Use visual aids. Some people find it helpful to create a mental image
or draw a picture to help them remember new words. For example, if
you're trying to remember the word "bucolic," you might picture a
peaceful countryside scene.

3. Play word games. There are many online and offline games that can
help you learn new words in a fun and interactive way. Scrabble,

rly
Bananagrams, and Boggle are all great options for building your
vocabulary

4. Keep a vocabulary journal. You can use a journal or a note-taking


app to write down new words and their definitions. You can also include
example sentences or create flashcards to help you remember the
words. Reviewing your journal regularly will help you reinforce your
learning.

5. Use root words and prefixes/suffixes to help you guess the meanings
la
of unfamiliar words. Many English words are built from smaller words,
such as prefixes (e.g. "anti-" meaning "against") and suffixes (e.g. "-
able" meaning "able to be"). By understanding these smaller building
blocks, you can often deduce the meanings of new words.

es!
ho

pl
The word "antipathy" Exam
comes from the prefix
t")
"anti-" (meaning "agains The
s"
and the root word "patho wor
d "p
c om
(meaning "strong "pe
r-" e e
s fro rceiv
" and (mea abl
emotion"). So "antipathy mt
h e
of the n ing e pre "
means "a strong feeling roo
t
"thr
oug ix
f
dislike or hostility." gra (mean word h")
sp" i "
). S ng "to ceive"
Sc

o "p t
erc ake o
und mean eiva r
erst s "a b l
ood ble e"
or n to b
otic e
ed.
"

VI
01
More examples:

- Use online resources to


help you learn new words.
There are many websites
and apps that offer
vocabulary-building games
and activities. Some
options include
Vocabulary.com, Word
Dynamo, and Word Up

rly
- Learn the words in groups or
categories. Instead of trying to learn a
long list of unrelated words, try
organising the words into categories or
themes. This can make the learning
process more manageable and help you
see the connections between words

INTRODUCTION
la
- Use context clues. When you
come across an unfamiliar word
in a text, try to use the
surrounding words and
sentences to help you figure out
its meaning. This can help you
ho

learn new words in a more


natural and meaningful way.

- Take breaks and review regularly.


It's important to give your brain time
to process and retain new
information. So take breaks and
Sc

review your vocabulary list regularly


to help you remember the words
you've learned.

VII
2WORDS DESRCIBE PEOPLE Chapter 2
Words to describe
people
ECCENTIRC
Definition: This word means someone who is a little bit strange or
unusual. For example, if your friend always wears mismatched socks,
they might be eccentric.
Mnemonic: EC-CENTRIC. The word "ec" sounds like "eccentric," so

rly
think of an eccentric person as being the center of their own strange
and unusual world
Word play: An eccentric person might say "I'm not weird, I'm just ec-
centric!"
Funny pop culture example: The character of Sheldon Cooper from
the TV show The Big Bang Theory is a good example of an eccentric
person. He's super smart, but also a little bit strange and does things
his own way.
QUIRKY
la
Definition: This word means someone who is a little bit odd or
peculiar. For example, if your teacher always tells jokes that nobody
understands, they might be quirky.
Mnemonic: Quirky people are like a Q-tip - they're a little bit unusual,
but still pretty useful!
ho

Word play: "I'm not weird, I'm just q-uirky!"


Funny pop culture example: The character of Luna Lovegood from the
Harry Potter series is a good example of a quirky person. She's always
saying strange things and believing in things that nobody else does.

OUTGOING
Definition: This word means someone who is friendly and enjoys being
around other people. For example, if you have a friend who always
wants to go to parties and meet new people, they might be outgoing.
Sc

Mnemonic: OUTGOING sounds a bit like "out," so think of an outgoing


person as someone who is always eager to go out and socialise.

VIII
02
Word play: An outgoing person is "out and about!"
Funny pop culture example: The character of Ron Swanson from the
TV show Parks and Recreation is a good example of an outgoing
person. Despite his gruff exterior, he's actually very friendly and loves
spending time with his friends and family.

INTROVERTED

WORDS DESRCIBE PEOPLE


Definition: This word means someone who is shy or reserved and
prefers to spend time alone. For example, if you have a friend who
doesn't like to go to parties and prefers to stay at home, they might
be introverted.
Mnemonic: INTROVERTED sounds a bit like "introvert," so think of an
introverted person as someone who is more inward-looking and
prefers to spend time alone.
rly
Word play: An introverted person is "inwardly focused."
Funny pop culture example: The character of Donnie Darko from the
movie of the same name is a good example of an introverted person.
He's quiet and reserved, and prefers to spend time alone thinking and
exploring his own thoughts and feelings.

GREAGARIOUS
la
Definition: This word means someone who is very outgoing and enjoys
being around other people. For example, if you have a friend who is
always the life of the party and loves meeting new people, they might
be gregarious.
Mnemonic: GREGARIOUS sounds a bit like "gregarious," so think of a
gregarious person as someone who is always seeking out new social
ho

experiences.
Word play: A gregarious person is "socially adventurous!"
Funny pop culture example: The character of Michael Scott from the
TV show The Office is a good example of a gregarious person. He's
always trying to be the centre of attention and loves meeting new
people and making friends.

ALOOF
Sc

Definition: This word means someone who is distant or reserved, and


doesn't like to get too close to other people. For example, if you have
a friend who is always quiet and doesn't like to share their feelings or
thoughts, they might be aloof.

IX
02
WORDS DESRCIBE PEOPLE Mnemonic: ALOOF sounds a bit like "a loft," so think of an aloof
person as someone who is a bit detached, like they're up in a loft
looking down on others.
Word play: An aloof person is "aloof and above it all."
Funny pop culture example: The character of Mr. Darcy from the book
and movie Pride and Prejudice is a good example of an aloof person.
He's very reserved and doesn't like to get too close to others,
especially at first.

SOLITARY
Definition: Living or existing alone. Someone or something that is
solitary is living or existing alone, without the company of others. They

rly
may be isolated or removed from society.
Mnemonic: Think of the word "solitary" as a combination of "solo,"
which means alone, and "tar," which is a sticky, gooey substance.
Someone who is solitary is stuck or isolated from others, like a gooey
substance that is difficult to remove.
Word play: A solitary person might be described as a "lone wolf."
This phrase captures the idea of the person being alone or isolated
from others.
Funny pop culture example: The character of Howard Wolowitz from
la
the TV show The Big Bang Theory is a good example of someone who
is solitary. He spends a lot of time alone in his room, working on his
projects and hobbies.

COMPLICATED
Definition: Complex and difficult to understand or analyse. Something
ho

that is complicated is complex and difficult to understand or analyse.


It may have many parts or factors that contribute to its complexity.
Mnemonic: Think of the word "complicated" as a combination of
"complicate," which means to make something more complex or
difficult, and "ed," which is a suffix that indicates past tense.
Something that is complicated has been made more complex or
difficult in the past.
Word play: Something that is complicated might be described as "a
Sc

tangled web." This phrase captures the idea of the thing being
complex and difficult to untangle or understand.

X
02
Funny pop culture example: The character of Tyrion Lannister from the
TV show Game of Thrones is a good example of someone who is
complicated. He's intelligent and resourceful, but also has a complex
and troubled past that makes him difficult to understand.

NOSTALGIC
Definition: Feeling or expressing longing for the past. Someone who is
nostalgic is feeling or expressing longing for the past. They may be

WORDS DESRCIBE PEOPLE


reminiscing about past events or experiences and feeling a sense of
longing or wistfulness.
Mnemonic: Think of the word "nostalgic" as a combination of
"nostalgia," which means a longing for the past, and "ic," which is a
suffix that indicates a characteristic or quality. Someone who is

rly
nostalgic is characterised by a longing for the past.
Word play: A nostalgic person might be described as "lost in the
past." This phrase captures the idea of the person being absorbed in
memories of the past and feeling a sense of longing or wistfulness.
Funny pop culture example: The character of Ross Geller from the TV
show Friends is a good example of someone who is nostalgic. He
often talks about the past and reminisces about his old college
friends and experiences.
la
PENSIVE
Definition: Engaged in deep thought, often with a sad or serious
expression Someone who is pensive is engaged in deep thought, often
with a sad or serious expression. They may be thinking about
something that is causing them to feel concerned or troubled.
ho

Mnemonic: Think of the word "pensive" as a combination of "pens,"


which means to think deeply, and "ive," which is a suffix that indicates
a characteristic or quality. Someone who is pensive is characterised
by deep thought.
Word play: A pensive person might be described as "deep in
thought." This phrase captures the idea of the person being absorbed
in thought, often with a sad or serious expression.
Funny pop culture example: The character of Spock from the TV show
Sc

Star Trek is a good example of someone who is pensive. He's often


seen thinking deeply and contemplatively, analysing situations and
making logical decisions.

XI
3DESRCIBE EMOTIONS

ELATED
Chapter 3
Words to describe
emotions

Definition: This word means feeling very happy or excited. For


example, if you just won the lottery, you might be elated.
Mnemonic: ELATED sounds like "elevated," so think of feeling elated
as being lifted up by your emotions.

rly
Word play: When you're elated, you're on "cloud nine!"

Funny pop culture example: ❖ The character of SpongeBob


SquarePants is a good example of someone who is always elated.
He's always bouncing around, full of energy and joy.

DEJECTED
Definition: This word means feeling very sad or disappointed. For
example, if you just found out that your favourite sports team lost the
championship, you might be dejected.
la
Mnemonic: DEJECTED sounds a bit like "rejected," so think of feeling
dejected as being rejected by your emotions.
Word play: When you're dejected, you're "down in the dumps."
Funny pop culture example: The character of Eeyore from the Winnie
the Pooh series is a good example of someone who is often dejected.
ho

He's always feeling sad and down, no matter what's happening.

ENRAGED
Definition: This word means feeling very angry or furious. For example,
if someone cuts you off in traffic and you honk your horn and shout at
them, you might be enraged.
Mnemonic: ENRAGED sounds a bit like "enraged," so think of this
word as a fancy way to describe extreme anger.
Sc

Word play: When you're enraged, you're "seeing red!"


Funny pop culture example: The character of Hulk from the Marvel
universe is a good example of someone who gets enraged easily.
When he gets angry, he turns into a giant green monster and starts
smashing things.

XII
03
ECSTATIC
Definition: This word means feeling very happy and excited. For
example, if you just found out that you got accepted to your dream
college, you might be ecstatic.

Mnemonic: ECSTATIC sounds a bit like "ecstasy," so think of this word


as a fancy way to describe extreme happiness.

Word play: When you're ecstatic, you're "over the moon!"


Funny pop culture example: The character of Mickey Mouse is a good
example of someone who is always ecstatic. He's always smiling and
having a great time, no matter what's happening.

MELANCHOLIC

rly
DESRCIBE EMOTIONS
Definition: This word means feeling very sad or depressed. For
example, if you're listening to a sad song and it makes you feel really
down, you might be melancholic.
Mnemonic: MELANCHOLIC sounds a bit like "melancholy," so think of
this word as a fancy way to describe a feeling of deep sadness.

Word play: When you're melancholic, you're "blue as can be."


Funny pop culture example: The character of Garfield from the comic
la
strip of the same name is a good example of someone who is often
melancholic. He's always feeling grumpy and sleeping all the time,
even though he has a pretty good life.

SULLEN
Definition: This word means feeling moody or sulky. For example, if
ho

you're in a bad mood and you don't want to talk to anyone, you might
be sullen.
Mnemonic: SULLEN sounds a bit like "sullen," so think of this word as a
fancy way to describe a moody or sulky disposition.

Word play: When you're sullen, you're "sulky as can be."


Funny pop culture example: The character of Stewie Griffin from the
TV show Family Guy is a good example of someone who is often
Sc

sullen. He's always moody and sulky, especially when things don't go
his way.

XIII
03
DESRCIBE EMOTIONS DESPONDENT
Definition: Feeling or expressing hopelessness or despair. Someone who
is despondent is feeling or expressing hopelessness or despair. They
may be feeling discouraged or defeated, and may lack motivation or
energy.
Mnemonic: Think of the word "despondent" as a combination of
"despond," which means to lose hope or be disheartened, and "ent,"
which is a suffix that indicates a person or thing. Someone who is
despondent is a person who has lost hope or is disheartened.

Word play: A despondent person might be described as "down in the


dumps." This phrase captures the idea of the person feeling low or
hopeless.

rly
Funny pop culture example: The character of Eeyore from the children's
book series Winnie the Pooh is a good example of someone who is
despondent. He's often seen feeling sad or down, and lacks motivation
or energy.

OVERJOYED
Definition: Extremely happy or delighted. Someone who is overjoyed is
extremely happy or delighted. They may feel overwhelmed with joy or
la
excitement.
Mnemonic: Think of the word "overjoyed" as a combination of "over,"
which means above or beyond, and "joyed," which means to
experience joy or delight. Someone who is overjoyed is experiencing
joy or delight to an extreme or excessive degree.
ho

Word play: An overjoyed person might be described as "over the


moon." This phrase captures the idea of the person feeling extremely
happy or delighted.

Funny pop culture example: The character of Mickey Mouse from the
Disney franchise is a good example of someone who is overjoyed. He's
always cheerful and enthusiastic, and is easily excited or delighted.

EUPHRIC
Sc

Definition: Someone who is euphoric is feeling or expressing extreme


happiness or elation. They may feel a rush of excitement or joy.

XIV
03
Mnemonic: Think of the word "euphoric" as a combination of
"euphoria," which means a feeling of extreme happiness or elation,
and "ic," which is a suffix that indicates a characteristic or quality.
Someone who is euphoric is characterised by a feeling of extreme
happiness or elation.

Word play: An euphoric person might be described as "on cloud


nine." This phrase captures the idea of the person feeling extremely
happy or elated.

Funny pop culture example: The character of Phineas Flynn from the
TV show Phineas and Ferb is a good example of someone who is
euphoric. He's always full of energy and excitement, and is easily
thrilled or elated.

DISHEARTENED rly
DESRCIBE EMOTIONS
Definition: Feeling or expressing disappointment or despair. Someone
who is disheartened is feeling or expressing disappointment or
despair. They may feel discouraged or defeated, and may lack
motivation or energy.
Mnemonic: Think of the word "disheartened" as a combination of
"dis," which means not, and "heartened," which means encouraged or
la
strengthened. Someone who is disheartened is not encouraged or
strengthened, but rather feels disappointment or despair.
Word play: A disheartened person might be described as "down in
the dumps." This phrase captures the idea of the person feeling low or
defeated.
Funny pop culture example: The character of Charlie Brown from the
ho

comic strip Peanuts is a good example of someone who is


disheartened. He often feels discouraged or defeated, and lacks
motivation or energy.
Sc

XV
4 Chapter 4
Words to describe
actions
GESTICULATE
DESRCIBE ACTIONS

Definition: This word means to make gestures with your hands or


body to express something. For example, if you're trying to explain
something to someone and you're using a lot of hand gestures, you
might be gesturing.

rly
Mnemonic: GESTICULATE sounds a bit like "gesture," so think of
this word as a fancy way to say "gesture."

Word play: When you gesticulate, you're "acting with your hands!"

Funny pop culture example: The character of Mr. Bean is a good


example of someone who loves to gesticulate. He's always making
funny faces and gestures to express himself.
la
FROLIC
Definition: This word means to play and have fun in a carefree
way. For example, if you're running around in the park with your
friends, laughing and having a great time, you might be frolicking.

Mnemonic: FROLIC sounds a bit like "frolic," so think of this word


ho

as a fancy way to say "frolic."

Word play: When you frolic, you're "having a ball!"

Funny pop culture example: The character of Olaf from the movie
Frozen is a good example of someone who loves to frolic. He's
always bouncing around and having a great time, no matter
what's happening.
Sc

LURK

Definition: This word means to move around secretly or sneakily. For


example, if you're trying to scare your friends by hiding in the bushes,
you might be lurking.

XVI
04
DMnemonic: LURK sounds a bit like "lurk," so think of this word as a
fancy way to say "lurk."

Word play: When you lurk, you're "sneaking around!"

unny pop culture example: The character of Slender Man is a good


example of someone who loves to lurk. He's always hiding in the
shadows, waiting to scare people.

SCURRY
Definition:This word means to move quickly and busily. For example,
if you're late for school and you're rushing to get ready, you might
be scurrying.

rly
Mnemonic: SCURRY sounds a bit like "scurry," so think of this word as
a fancy way to say "scurry."

Word play: When you scurry, you're "in a hurry!"

DESRCIBE ACTIONS
Funny pop culture example: The character of Speedy Gonzales from
the Looney Tunes cartoons is a good example of someone who loves
to scurry. He's always racing around and getting things done at
lightning speed.
la
SLITHER
Definition: This word means to move smoothly and stealthily, like a
snake. For example, if you're sneaking up on someone to scare
them, you might be slithering.

Mnemonic: SLITHER sounds a bit like "slither," so think of this word as


ho

a fancy way to say "slither."

Word play: When you slither, you're "sneaking around like a snake!"

Funny pop culture example: The character of Sneaky Snake from the
children's show The Snake Show is a good example of someone who
loves to slither. He's always sneaking around and trying to surprise
his friends.
Sc

PROWL

Definition:This word means to move around stealthily or menacingly,


like a predator. For example, if you're pretending to be a lion and
you're stalking your prey (aka, your friends), you might be prowling.

XVII
04
DESRCIBE EMOTIONS Mnemonic: PROWL sounds a bit like "prowl," so think of this word as
a fancy way to say "prowl."

Word play: When you prowl, you're "stalking your prey!"

Funny pop culture example: The character of Tom from the cartoon
Tom and Jerry is a good example of someone who loves to prowl.
He's always sneaking up on Jerry and trying to catch him, but he
never quite succeeds.

SAUNTER
Definition: To walk leisurely and with no particular purpose or
destination..To saunter means to walk leisurely and with no
particular purpose or destination. It may involve strolling or leisurely
pacing about.
rly
Mnemonic: Think of the word "saunter" as a combination of
"saunter," which means to walk leisurely and with no particular
purpose or destination, and "er," which is a suffix that indicates a
person or thing. Someone who is sauntering is a person who is
walking leisurely and with no particular purpose or destination.

Word Play: Someone who is sauntering might be described as


la
"meandering." This phrase captures the idea of the person walking
leisurely and with no particular purpose or destination.

Funny pop culture example: The character of Mr. Peabody from the
TV show Mr. Peabody & Sherman is a good example of someone
who is sauntering. He's often depicted as leisurely strolling or
ho

pacing about, with no particular purpose or destination.

TIPTOE
Definition: To tiptoe means to walk on one's toes, especially quietly
or carefully. It may involve moving cautiously or stealthily.

Mnemonic: Think of the word "tiptoe" as a combination of "tip,"


which means the pointed end of something, and "toe," which is a
part of the foot. Someone who is tiptoeing is walking on the pointed
Sc

ends of their feet, especially quietly or carefully.

Word Play: Someone who is tiptoeing might be described as


"stealthily sneaking." This phrase captures the idea of the person
moving cautiously or stealthily.

XVIII
04
Funny pop culture example: The character of Garfield from the
comic strip and TV show Garfield is a good example of someone
who is tiptoeing. He's often depicted as stealthily sneaking around or
moving cautiously, especially when he's up to no good.

SPRINT
Definition: To sprint means to run or move very quickly or at full speed.
It may involve running or moving as fast as possible.

Mnemonic: Think of the word "sprint" as a combination of "sprint,"


which means to run or move very quickly or at full speed, and "t,"
which is a suffix that indicates a person or thing. Someone who is
sprinting is a person who is running or moving very quickly or at full
speed.

rly
Word Play: Someone who is sprinting might be described as "bolting."
This phrase captures the idea of the person running or moving very
quickly or at full speed.

DESRCIBE ACTIONS
Funny pop culture example: The character of Sonic the Hedgehog
from the video game and TV show franchise of the same name is a
good example of someone who is sprinting. He's known for his speed
and ability to run or move very quickly or at full speed.
la
CRAWL

Definition: Move slowly and on one's hands and knees, by dragging


one's body along the ground. Moving in a low, crawling position.

Mnemonic: Think "crawl" as a combination of "crawl," which means to


ho

move slowly and on one's hands and knees or by dragging one's body
along the ground, and "l," which is a suffix that indicates a person or
thing. Someone crawling is a person who is moving slowly and on their
hands and knees or by dragging their body along the ground.

Word Play: Someone who is crawling might be described as "inching


along." This phrase captures the idea of the person moving slowly and
in a low, crawling position.
Sc

Funny pop culture example: The character of Gizmo from the movie
Gremlins is a good example of someone who is crawling. He's often
crawling on his hands and knees or dragging his body along the
ground, especially when he's in his mischievous "gremlin" form.

XIX
5
DESRCIBE PLACES

GLAMOROUS
Chapter 5
Words to describe
places

Definition: This word means very attractive or stylish in an eye-catching


way. For example, if you're at a fancy party and everything is shiny and
sparkling, the party might be glamorous.
Mnemonic: GLAMOROUS sounds a bit like "glamorous," so think of this
word as a fancy way to say "glamorous."
Word play: When a place is glamorous, it's "glittering and stylish!"

stylish people.

DEARY
rly
Funny pop culture example: The city of Las Vegas is a good example of
a place that is glamorous. It's full of shiny casinos, sparkling lights, and

Definition: This word means looking or feeling sad and depressing. For
example, if you're walking through a city on a rainy day and everything
is grey and gloomy, the city might be dreary.
Mnemonic: DREARY sounds a bit like "dreary," so think of this word as a
fancy way to say "dreary."
la
Word play: When a place is dreary, it's "depressing and gloomy!"
Funny pop culture example: The city of Gotham from the Batman series
is a good example of a place that is often dreary. It's always rainy and
gloomy, and there's always crime and corruption happening.

IDYLLIC
ho

Definition: This word means very peaceful and perfect. For example, if
you're on a beach vacation and everything is warm and sunny and
you're having a great time, the vacation might be idyllic.
Mnemonic: IDYLLIC sounds a bit like "idyllic," so think of this word as a
fancy way to say "idyllic."
Word play: When a place is idyllic, it's "perfect and peaceful!"
Funny pop culture example: The island of Bali is a good example of a
place that is often described as idyllic. It's beautiful and peaceful, with
warm weather and crystal clear waters.
Sc

ATMOSPHERIC
Definition: This word means having a special atmosphere or feeling. For
example, if you're at a spooky haunted house and everything is dark and
creepy, the haunted house might have an atmospheric feeling.

XX
05
Mnemonic: ATMOSPHERIC sounds a bit like "atmospheric," so think of
this word as a fancy way to say "atmospheric."
Word play: A place is atmospheric when its "ambiance all its own!"
Funny pop culture example: The city of New Orleans is a good example
of a place that is often described as atmospheric. It has a unique
atmosphere, with its jazz music, Creole cuisine, and haunted history.

OPPRESSIVE
Definition: This means feeling very heavy or suffocating. For example, if
you're in a small room with no windows and no air conditioning, the
room might feel oppressive.
Mnemonic: OPPRESSIVE sounds a bit like "oppressive," so think of this
word as a fancy way to say "oppressive."
Word play: When a place is oppressive, it's "suffocating and heavy!"

rly
Funny pop culture example: The planet of Mustafar from the Star Wars
series is a good example of a place that is oppressive. It's a hot and
volcanically active planet, with no signs of life or civilization.

VAST
Definition: This word means very large or expansive. For example, if
you're standing in the middle of a desert and you can't see any end to

DESRCIBE PLACES
it, the desert might seem vast.
Mnemonic: VAST sounds a bit like "vast," so think of this word as a fancy
way to say "vast."
la
Word play: When a place is vast, it's "enormous and endless!"
Funny pop culture example: The ocean is a good example of a place
that is often described as vast. It's huge and deep, with no end in sight.

BARREN
Definition: Something that is barren is lacking vegetation, fertility, or
ho

value. It may be desolate or unproductive.


Mnemonic: Think of the word "barren" as a combination of "bar," which
means a barrier or obstruction, and "ren," which is a suffix that
indicates a person or thing. Something that is barren is an obstruction
or barrier to vegetation, fertility, or value.
Word Play: Something that is barren might be described as "sterile."
This phrase captures the idea of the thing being lacking in vegetation,
fertility, or value.
Funny pop culture example: The planet of Tatooine from the Star Wars
Sc

franchise is a good example of a barren place. It's a desert planet


that is lacking in vegetation and is generally a harsh and unforgiving
environment.

XXI
05
LIVELY
DESRCIBE EMOTIONS
Definition: Something that is lively is full of energy, activity, or
excitement. It may be vibrant or energetic.
Mnemonic: Think of the word "lively" as a combination of "live," which
means to be alive or to exist, and "ly," which is a suffix that indicates a
quality or characteristic. Something that is lively is a quality or
characteristic of being alive or energetic.
Word Play: Something that is lively might be described as "animated."
This phrase captures the idea of the thing being full of energy, activity,
or excitement.
Funny pop culture example: The character of Spongebob Squarepants
from the TV show Spongebob Squarepants is a good example of
someone who is lively. He's always full of energy and excitement, and is
known for his playful and energetic personality.

MAGICAL
rly
Definition: Something that is magical is having or possessing special
powers, qualities, or charm. It may be mysterious or enchanting.
Mnemonic: Think of the word "magical" as a combination of "magic,"
which means the power to produce illusions or supernatural effects, and
"al," which is a suffix that indicates a person or thing. Something that is
magical is a person or thing with the power to produce illusions or
supernatural effects.
Word Play: Something that is magical might be described as
la
"enchanting." This phrase captures the idea of the thing possessing
special powers, qualities, or charm.
Funny pop culture example: Harry Potter from the book and movie
franchise of the same name is a good example of someone who is
magical. He possesses special powers and abilities due to his magical
heritage, and is known for his ability to perform spells and magic tricks.
ho

GLOOMY
Definition: Something that is gloomy is dark and dismal. It may be
depressing or melancholy.
Mnemonic: Think of the word "gloomy" as a combination of "gloom,"
which means darkness or sadness, and "y," which is a suffix that
indicates a person or thing. Something that is gloomy is a person or
thing with an atmosphere of darkness or sadness.
Word Play: Something that is gloomy might be described as "dreary."
Sc

This phrase captures the idea of the thing being dark and dismal.
Funny pop culture example: The Eeyore from the Winnie the Pooh
franchise is an example of someone who is gloomy. Always being sad or
depressed, and is known for his generally downbeat personality

XXII
Chapter 6
Words to describe
events
6

DESCRIBE EVENTS
EXUBERANT

Definition: This word means very full of energy, enthusiasm, and joy. For
example, if you have a friend who is always in a good mood and loves
to have fun, they might be described as exuberant.
Mnemonic: EXUBERANT sounds a bit like "exuberant," so think of this

joy!"
rly
word as a fancy way to say "exuberant."
Word play: When someone is exuberant, they're "full of energy and

Funny pop culture example: The character of SpongeBob SquarePants


is a good example of someone who is often described as exuberant.
He's always in a good mood and loves to have fun, no matter what
challenges he faces.

MONOTONOUS
la
Definition: This word means very repetitive and dull. For example, if
you have a friend who always does the same thing every day and
never tries anything new, their life might be described as monotonous.
Mnemonic: MONOTONOUS sounds a bit like "monotone," so think of
this word as a way to describe something that's boring and repetitive,
like a monotone voice.
ho

Word play: When something is monotonous, it's "repetitive and dull!"


Funny pop culture example: The character of Dale Gribble from King
of the Hill is a good example of someone who might describe events
as monotonous. He's always doing the same thing every day and never
tries anything new, which can make his life feel repetitive and dull.

RAUCOUS

Definition: This word means very loud and boisterous. For example, if
Sc

you have a friend who loves to throw loud and energetic parties, they
might be described as raucous.
Mnemonic: RAUCOUS sounds a bit like "raucous," so think of this word
as a fancy way to say "raucous."

XXIII
06
Word play: When something is raucous, it's "loud and boisterous!"
Funny pop culture example: The character of Ron Swanson from the TV
show Parks and Recreation is a good example of someone who might
describe events as raucous. He's known for throwing loud and
energetic parties, complete with lots of food, drink, and good times.

LAVISH

Definition: This word means very generous and extravagant. For


example, if you have a friend who loves to spoil their loved ones with
expensive gifts and indulgent experiences, they might be described as
lavish.
Mnemonic: LAVISH sounds a bit like "lavish," so think of this word as a
fancy way to say "lavish."

rly
Word play: When something is lavish, it's "generous and extravagant!"
Funny pop culture example: The character of Scrooge McDuck from
the Disney series DuckTales is a good example of someone who is
often described as lavish. He's very wealthy and loves to indulge in
expensive and extravagant experiences, such as diving into his giant
DESCRIBE EVENTS

vault of gold coins.

GRANDIOSE
la
Definition: This word means very impressive and impressive in an
exaggerated or pompous way. For example, if you have a friend who
loves to show off their wealth and success, they might be described as
grandiose.
Mnemonic: GRANDIOSE sounds a bit like "grand," so think of this word
as a fancy way to say "grand."
Word play: When something is grandiose, it's "impressive and grand!"
ho

Funny pop culture example: The character of Mr. Krabs from


SpongeBob SquarePants is a good example of someone who is often
described as grandiose. He's very proud of his wealth and success, and
he loves to show off his riches in an exaggerated and pompous way.

INTIMATE

Definition: This word means very private, personal, and close. For
Sc

example, if you have a friend who always confides in you and shares
their deepest thoughts and feelings with you, your relationship might
be described as intimate.
Mnemonic: INTIMATE sounds a bit like "intimate," so think of this word
as a fancy way to say "intimate."

XXIV
06

DESCRIBE EVENTS
Word play: When something is intimate, it's "private and personal!"
Funny pop culture example: The character of Lorelai Gilmore from the
TV show Gilmore Girls is a good example of someone who might
describe a relationship as intimate. She's very close with her daughter
Rory and always confides in her and shares her deepest thoughts and
feelings with her.

ELEGANT

Definition: Something that is elegant is graceful and stylish in


appearance or manner. It may be refined or sophisticated.
Mnemonic: Think of the word "elegant" as a combination of "elegant,"
which means graceful and stylish, and "ant," which is a suffix that

rly
indicates a person or thing. Something that is elegant is a person or
thing that is graceful and stylish.
Word Play: Something that is elegant might be described as "refined."
This phrase captures the idea of the thing being graceful and stylish in
appearance or manner.
Funny pop culture example: The character of Daria Morgendorffer
from the TV show Daria is a good example of someone who is elegant.
Despite being a bit of a misanthrope, she has a refined and
sophisticated manner, and is known for her stylish and graceful
la
appearance.

POMPOUS

Definition: Someone who is pompous is showing off or displaying their


importance or wealth in an arrogant or self-important way. They may
be vain or conceited.
ho

Mnemonic: Think of the word "pompous" as a combination of "pomp,"


which means grandeur or splendour, and "ous," which is a suffix that
indicates a quality or characteristic. Someone who is pompous is grand
or splendorous in an arrogant or self-important way.
Word Play: Someone who is pompous might be described as
"arrogant." This phrase captures the idea of the person showing off or
displaying their importance or wealth in an arrogant or self-important
way.
Sc

Funny pop culture example: The character of Mr. Burns from the TV
show The Simpsons is a good example of someone who is pompous. He's
a wealthy businessman who is known for his arrogant and self-important
demeanour, and loves to show off his wealth and status.

XXV
06

RAMBUCUNTIOUS

Definition: Someone or something that is rambunctious is loud,


boisterous, and difficult to control. They may be energetic or unruly.
Mnemonic: Think of the word "rambunctious" as a combination of "ram,"
which means to charge violently or forcefully, and "bunctious," which
means unruly or disobedient. Someone or something that is rambunctious
is unruly or disobedient in a loud and boisterous way.
Word Play: Someone or something that is rambunctious might be
described as "rowdy." This phrase captures the idea of the person or
thing being loud, boisterous, and difficult to control.
Funny pop culture example: The character of Remy the rat from the
movie Ratatouille is a good example of someone who is rambunctious.

rly
He's energetic, unruly, and difficult to control, and is known for his loud
and boisterous personality.

WHIMSICAL
DESCRIBE EVENTS

Definition: Something that is whimsical is playfully fanciful or fancifully


playful. It may be lighthearted or fanciful.
Mnemonic: Think of the word "whimsical" as a combination of "whim,"
which means a sudden desire or change of mind, and "ical," which is a
la
suffix that indicates a quality or characteristic. Something that is
whimsical is a quality or characteristic of sudden desires or changes of
mind.
Word Play: Something that is whimsical might be described as
"capricious." This phrase captures the idea of the thing being playfully
fanciful or fancifully playful.
ho

Funny pop culture example: The character of Pee-wee Herman from the
movie Pee-wee's Big Adventure is a good example of someone who is
whimsical. He's lighthearted and fanciful, and is known for his playful
and whimsical personality.
Sc

XXVI
Chapter 7
Words to describe
things
7

DESCRIBE THINGS
LUXURIOUS
Definition: This word means very comfortable, expensive, and
indulgent. For example, a friend who always stays in five-star hotels
and loves to be pampered, your friend might be described as luxurious.
Mnemonic: LUXURIOUS sounds a bit like "luxurious," so think of this
word as a fancy way to say "luxurious."

rly
Word play: "comfortable and indulgent!"
Funny pop culture example: The character of Mr. Burns from The
Simpsons is a good example of someone who is often described as
luxurious. He's very wealthy and always surrounds himself with the most
expensive and indulgent things, such as his private yacht and his
mansion full of rare treasures.

DILAPIDATED
Definition: This word means very run-down and in poor condition. For
example, a friend who lives in a house that is falling apart and in need
la
of major repairs, their house might be described as dilapidated.
Mnemonic: DILAPIDATED sounds a bit like "dilapidated," so think of this
word as a fancy way to say "dilapidated."
Word play: When something is dilapidated, it's "run-down and in poor
condition!"
Funny pop culture example: The character of the Addams Family from
ho

the TV show and movies of the same name is a good example of


people who might describe things as dilapidated. They live in a creepy
and decrepit old mansion that is falling apart and in need of major
repairs, but they love it just the way it is.

EXQUISITE
Definition: This word means very delicate, beautiful, and refined. For
example, if you have a friend who loves to collect fine art or antiques,
they might describe their collection as exquisite.
Sc

Mnemonic: EXQUISITE sounds a bit like "exquisite," so think of this word


as a fancy way to say "exquisite."

XXVII
07
Word play: When something is exquisite, it's "delicate and beautiful!"
Funny pop culture example: The character of Professor Albus
Dumbledore from the Harry Potter series is an example of someone who
might describe objects as exquisite. He's known for his refined and
sophisticated taste, loves to surround himself with beautiful and delicate
objects, such as his collection of rare books and his elaborate office.

TAWDRY
Definition: This word means very cheap, gaudy, and tasteless. For
example, if you have a friend who loves to buy cheap, flashy, and tacky
clothing and accessories, they might be described as tawdry.
Mnemonic: TAWDRY sounds a bit like "tawdry," so think of this word as a

rly
fancy way to say "tawdry."
Word play: When something is tawdry, it's "cheap and tasteless!"
Funny pop culture example: The character of Edina Monsoon from the
TV show Absolutely Fabulous is a good example of someone who is often
described as tawdry. She's very wealthy, but she has poor taste and
loves to buy cheap, flashy, and tacky clothing and accessories.
DESCRIBE THINGS

OPULENT
Definition: This word means very rich and luxurious. For example, if you
la
have a friend who loves to surround themselves with expensive and
indulgent possessions, they might be described as opulent.
Mnemonic: OPULENT sounds a bit like "opulent," so think of this word as
a fancy way to say "opulent."
Word play: When something is opulent, it's "rich and luxurious!"
Funny pop culture example: The character of Dr. Evil from the Austin
ho

Powers movies is a good example of someone who is often described as


opulent. He's very wealthy and loves to surround himself with expensive
and indulgent possessions, such as his giant underground lair and his
army of henchmen.
MUNDANE
Definition: This word means very ordinary and uninteresting. For
example, if you have a friend who always does the same thing every day
and never tries anything new, their life might be described as mundane.
Sc

Mnemonic: MUNDANE sounds a bit like "mundane," so think of this word


as a fancy way to say "mundane."
Word play: When something is mundane, it's "ordinary and
uninteresting!"

XXVIII
07

DESCRIBE THINGS
Funny pop culture example: The character of Jerry Seinfeld from the TV
show Seinfeld is a good example of someone who might describe things
as mundane. He's always getting into mundane and everyday situations,
such as waiting in line at the DMV or trying to find a good parking spot.

GORGEOUS
Definition: Something that is gorgeous is very beautiful or attractive. It
may be striking or eye-catching.
Mnemonic: Think of the word "gorgeous" as a combination of "gorge,"
which means to eat or swallow greedily, and "ous," which is a suffix that
indicates a quality or characteristic. Something that is gorgeous is so
beautiful or attractive that it makes you want to "eat it up" or admire it
greedily.

beautiful or attractive.
rly
Word Play: Something that is gorgeous might be described as
"stunning." This phrase captures the idea of the thing being very

Funny pop culture example: The character of Elle Woods from the
movie Legally Blonde is a good example of someone who is gorgeous.
She's very beautiful and attractive, and is known for her striking and
eye-catching appearance.

PLAIN
la
Definition: Something that is plain is simple, ordinary, or unpretentious. It
may be plain or unremarkable.
Mnemonic: Think of the word "plain" as a combination of "plain," which
means simple or ordinary, and "n," which is a suffix that indicates a
person or thing. Something that is plain is a person or thing that is
simple or ordinary.
ho

Word Play: Something that is plain might be described as "unassuming."


This phrase captures the idea of the thing being simple, ordinary, or
unpretentious.
Funny pop culture example: The character of Dwight Schrute from the
TV show The Office is a good example of someone who is plain. He's
simple, ordinary, and unpretentious, and is known for his plain and
unremarkable appearance.

EXOTIC
Sc

Definition: Something that is exotic is strange or unusual, especially in


an attractive or interesting way. It may be foreign or unfamiliar.

XXIX
07
Word Play: Something that is exotic might be described as "unusual."
This phrase captures the idea of the thing being strange or unusual,
especially in an attractive or interesting way.
Funny pop culture example: The character of Timon from the movie The
Lion King is a good example of someone who is exotic. He's strange and
unusual, and is known for his exotic and foreign appearance.

ORDINARY

Definition: Something that is ordinary is normal, common, or typical. It


may be average or unremarkable.
Mnemonic: Think of the word "ordinary" as a combination of "ord,"
which means order or arrangement, and "inary," which means relating

rly
to a series or sequence. Something that is ordinary is in the usual order
or sequence.
Word Play: Something that is ordinary might be described as
"common.” This phrase captures the idea of the thing being normal,
common, or typical.
DESCRIBE THINGS

Funny pop culture example: The character of Jerry Seinfeld from the TV
show Seinfeld is a good example of someone who is ordinary. He's
normal, common, and typical, and is known for his average and
unremarkable personality
la
ho
Sc

XXX
Chapter 8
Practice test
8
1. Which word means very attractive or stylish in an eye-catching way?
a. Glitzy
b. Opulent
c. Glamorous
d. Sleek

2. Which word means very peaceful and perfect?


a. Idyllic
b. Dreary
c. Oppressive
d. Vast
rly
3. Which word means looking dirty and shabby?
a. Dingy
b. Glamorous
c. Opulent
la
d. Sleek

4. Which word means feeling very heavy or suffocating?


a. Oppressive
b. Vast
c. Idyllic
ho

d. Dreary

5. Which word means having a special atmosphere or feeling?


a. Atmospheric
b. Glamorous
c. Opulent
d. Sleek

6. Which word means feeling very sad or depressed?


Sc

a. Melancholic
b. Flamboyant
c. Sullen
d. Opinionated

XXXI
08
7. Which word means feeling moody or sulky?
a. Sullen
b. Flamboyant
c. Melancholic
d. Opinionated

8. Which word means very showy or attention-seeking?


a. Flamboyant
b. Melancholic
c. Sullen
d. Opinionated

9. Which word means having strong and often inflexible opinions?


a. Opinionated
b. Flamboyant
c. Melancholic
d. Sullen
rly
10. Which word means very beautiful or attractive?
a. Ravishing
b. Flamboyant
c. Melancholic
d. Sullen
la
11. Choose the word that means very private, personal, and close:
A. Eccentric
B. Quirky
C. Intimate
D. Gregarious
ho

12. Choose the word that means very run-down and in poor condition:
A. Exuberant
B. Monotonous
C. Dilapidated
D. Lavish

13. Complete the sentence with the appropriate word: The actor was known for
his ability to _________ his hands and face to express a wide range of
Sc

emotions.
A. Gesticulate
B. Frolic
C. Lurk
D. Scurry

XXXII
08
14. Choose the word that means very ordinary and uninteresting:
A. Glamorous
B. Dreary
C. Idyllic
D. Mundane

15. Choose the word that means very rich and luxurious:
A. Atmospheric
B. Oppressive
C. Vast
D. Opulent

16. Complete the sentence with the appropriate word: The snake was seen
_________ through the grass.
A. Slithering
B. Prowling
C. Gesticulating
D. Frolicing
rly
17. Choose the word that means very comfortable, expensive, and indulgent:
A. Luxurious
B. Dilapidated
C. Exquisite
la
D. Tawdry

18. Choose the word that means very impressed and impressive in an
exaggerated or pompous way:
A. Grandiose
B. Intimate
C. Eccentric
ho

D. Quirky

19. Choose the word that means very delicate, beautiful, and refined:
A. Exquisite
B. Tawdry
C. Opulent
D. Luxurious
Sc

20. Choose the word that means very loud and boisterous:
A. Raucous
B. Lavish
C. Grandiose
D. Intimate

XXXIII
08
21. Which of the following words means someone who is a little bit strange or
unusual?
A. Eccentric
B. Quirky
C. Outgoing
D. Introverted

22. Which of the following words means someone who is living or existing alone,
without the company of others?
A. Eccentric
B. Quirky
C. Outgoing
D. Solitary

A. Complicated
B. Nostalgic
C. Pensive
rly
23. Which of the following words means feeling or expressing longing for the
past?

D. Solitary

24. Which of the following words means deep or serious thinking, especially
la
about something that is troubling or uncertain?
A. Complicated
B. Nostalgic
C. Pensive
D. Solitary

25. Which of the following words means feeling very happy and excited?
ho

A. Elated
B. Dejected
C. Enraged
D. Ecstatic

26. Which of the following words means feeling sad, disappointed, and
discouraged?
A. Elated
Sc

B. Dejected
C. Enraged
D. Melancholic

XXXIV
08
128. Which of the following words means a place that is empty or not
developed, and has very little vegetation or wildlife?
A. Oppressive
B. Vast
C. Barren
D. Lively

29. Which of the following words means very elaborate and impressive,
especially in a way that is expensive or showy?
A. Exuberant
B. Monotonous
C. Raucous
D. Lavish

A. Luxurious
B. Dilapidated
C. Exquisite
D. Tawdry
rly
30. Which of the following words means cheap, showy, and of poor quality, and
often in a way that is tasteless or vulgar?
la
ho
Sc

XXXV
Answers

1. C. Glamorous
2. A. Idyllic
3. A. Dingy
4. A. Oppressive
5. A. Atmospheric
6. A. Melancholic
7. A. Sullen
8. A. Flamboyant
9. A. Opinionated
10. A. Ravishing
11. C. Intimate
12. C. Dilapidated
13. A. Gesticulate
rly
14. D. Mundane
15. D. Opulent
16. A. Slithering
la
17. A. Luxurious
18. A. Grandiose
19. A. Exquisite
20. A. Raucous
ho

21. A. Eccentric
22. D. Solitary
23. B. Nostalgic
24. C. Pensive
25. D. Ecstatic
26. B. Dejected
27. A. Despondent
Sc

28. C. Barren
29. D. Lavish
30. D. Tawdry

XXXVI
rly
la
ho

Want free materials and guidance?

Scan the code below:


Sc
Scholarly
Steve Xu
Scholarly Publishing
Chapter 1: Introduction to the Opportunity iv
Class Mathematical Reasoning test

Chapter 2: Fractions and Decimals v


Basic concepts of fractions, including numerator,
denominator, and equivalent fractions
Simplifying fractions
Understanding decimal place value and converting
between fractions and decimals

Chapter 3: Unit Conversion viii


TABLE OF CONTENTS

Basic concepts of unit conversion, including

rly
understanding the relationship between different units
of measurement (e.g. length, weight, volume)
Explanation of the importance of unit conversion in
real-life situations

Chapter 4: Money Problems xiii


Basic concepts of solving money problems
la
Chapter 5: Time, Date and Directions Problems xv
Basic concepts of time, date and direction

Chapter 6: Speed, Distance and Time xviii


Problems
ho

Basic concepts of speed, distance, and time, including


calculating speed, distance, and time, and solving
related problems
Understanding and using rates and unit rates

Chapter 7: Number Patterns and Sequences xxii


Basic concepts of number patterns and sequences,
Sc

including identifying patterns and sequences, and


predicting the next number in a sequence
Understanding and using arithmetic and geometric
sequences

ii
Chapter 8: Ratio and Proportion xxvi
Basic concepts of ratio and proportion, including
understanding and comparing ratios, and solving
proportion problems
Understanding and using cross-multiplication and
proportionality

Chapter 9: Perimeter and Area Problems xxix


Basic concepts of perimeter and area, including
calculating perimeter and area of different shapes, and
solving related problems
TABLE OF CONTENTS

Understanding the relationship between perimeter


and area

rly Chapter 10: Line of Symmetry, Grid References,


Folding and Nets
Basic concepts of line of symmetry, grid references,
folding, and nets, including identifying lines of
xxxiii

symmetry, using grid references, understanding nets


and folding, and solving related problems
la
Understanding and using geometric shapes and
transformations

Chapter 11: Data and Statistics xxxviii


Basic concepts of data and statistics, including
collecting, organising and interpreting data,
ho

understanding probability and statistics, and solving


related problems
Understanding data representation and visualisation

Chapter 12: Practise Exercises xliv


Sc

iii
1 Chapter 1:
Introduction to the
Opportunity Class
Mathematical
Reasoning test

rly
la
The mathematical reasoning section of Opportunity Class test assesses students'
understanding of mathematical concepts and their ability to solve problems. It
includes 35 multiple choice questions with the time limit of 40 minutes. The
questions cover a range of mathematical concepts including numbers and
operations, algebra, geometry, measurement, data analysis, and problem-solving.
ho

The purpose of the OC test is to identify students who demonstrate a high level of
mathematical ability and potential, and provide them with the opportunity to
receive an education that is tailored to their abilities and needs. The test is
designed to assess a student's ability to think critically, solve problems, and apply
mathematical concepts to real-world situations.

The OC test is considered a high-stakes test, as the results are used to determine
Sc

admission to selective high schools. The test is also used to inform decisions about
curriculum, instruction, and assessment at the school and district level. The test
results can also be used to identify areas where students need additional support
and to develop targeted interventions to help them improve their mathematical
ability.

iv
2 Chapter 2:
Fractions and
Decimals

rly
Basic concepts of fractions, including numerator, denominator, and equivalent
fractions

The basic concepts of fractions and decimals, which are fundamental


la
mathematical concepts that are essential for understanding more advanced
mathematical concepts and problem-solving.

A fraction is a number that represents a part of a whole. It is written as two


numbers separated by a slash, with the top number called the numerator and
the bottom number called the denominator.
ho

For example, the fraction 23 represents 2 parts out of 3. The denominator


represents the total number of parts in the whole, while the numerator represents
the number of parts being considered.

One important concept related to fractions is equivalent fractions. Equivalent


fractions are different fractions that represent the same value.
Sc

For example, 23 and 46 are equivalent fractions because they both represent the
same value (2 parts out of 3). To find equivalent fractions, you can multiply or
divide both the numerator and denominator by the same number.

v
Simplifying fractions

Another important concept related to fractions is simplifying fractions. Simplifying


fractions means writing them in their simplest form. To simplify a fraction, you can
divide both the numerator and denominator by their greatest common factor
(GCF).

For example, to simplify the fraction 1224, you can divide both the numerator and
denominator by the GCF of 12, resulting in the simplified fraction of 12.

Understanding decimal place value and converting between fractions and


decimals

Decimals are another important concept related to fractions. A decimal is a

rly
number that represents a value between two whole numbers. It is written as a
whole number followed by a decimal point and one or more digits.

For example, 0.75 represents 3 parts out of 4 or 75 out of 100.

To convert a fraction to a decimal, you divide the numerator by the


denominator.
la
For example, to convert the fraction 23 to a decimal, you divide 2 by 3, resulting in
the decimal 0.6666... (which is an infinite decimal)

Conversely, to convert a decimal to a fraction, you can write the decimal as a


fraction with a denominator of 10 raised to the number of decimal places.
For example, to convert the decimal 0.75 to a fraction, you can write it as the
fraction 75100.
ho

Exercises

1. 45 of the students in Mr Lacson’s class are involved in academic contests. Of


those students, 13 are involved in a special activity. What fraction of his
students are involved in the special activity?
Sc

A. 54
B. 415
C. 24
D. 920

vi
2. Jade has 23 gallon of blue paint and 712 gallon of red paint.
If she has a total of 218 gallons of paint, how many gallons are neither red nor
blue?

A. 85
B. 78
C. 910
D. 114

3. Wonder woman baked a banana muffin and used 34 cups of oats.


You wanted to bake ½ of what Wonder woman made.
How much oats should you use?

A. 25
B. 46
C. 38
D. 12
rly
4. To get home, Kate rides the bus for 18 kilometers and walks for another 4
kilometers. What fraction of kilometers does Kate travel by bus?
la
A. 211
B. 911
C. 29
D. 184
ho

5. James has a piece of log that is 34 of a meter in length. He needs to cut pieces
that are 116 of a meter length. How many pieces can he cut?

A. 12 pieces
B. 10 pieces
C. 15 pieces
D. 8 pieces
Sc

Answers

1. B 2.B 3.C 4.B 5.A

vii
3 Chapter 3:
Unit Conversion

rly
Basic concepts of unit conversion, including understanding the relationship
between different units of measurement (e.g. length, weight, volume)

Unit conversion is the process of converting a measurement from one unit to


another. It is important to understand the relationship between different units of
measurement and how to convert between them. For example, it is important to
la
know how to convert between different units of length (e.g. meters, feet, inches),
weight (e.g. kilograms, pounds), and volume (e.g. liters, gallons).

To convert between different units of measurement, you need to understand the


relationship between them.
ho

Length: Length is a measure of the distance between two points. Common


units of length include meters, feet, inches, centimeters, and kilometers. To
convert between different units of length, you need to understand the
relationship between them.

For example, there are 100 centimeters in 1 meter and 12 inches in 1 foot. By
understanding these relationships, you can use a conversion factor to convert
between different units. A conversion factor is a ratio that expresses the
Sc

relationship between two units of measurement.

Weight: Weight is a measure of the force exerted by gravity on an object.


Common units of weight include kilograms, pounds, and ounces. To convert
between different units of weight, you need to understand the relationship
between them.

viii
For example, there are 16 ounces in 1 pound and 1 kilogram is approximately 2.2
pounds. By understanding these relationships, you can use a conversion factor to
convert between different units.

Volume: Volume is a measure of the amount of space an object occupies.


Common units of volume include liters, gallons, and milliliters. To convert
between different units of volume, you need to understand the relationship
between them.

For example, there are 1000 milliliters in 1 liter and 1 gallon is approximately 3.785
liters. By understanding these relationships, you can use a conversion factor to
convert between different units.

It is important to understand these basic concepts of unit conversion in order to

rly
work with measurements effectively in the Mathematical reasoning test.

Explanation of the importance of unit conversion in real-life situations

Unit conversion is an essential skill for many real-world situations. Understanding


how to convert between different units of measurement is important in various
fields and everyday life such as:

Cooking and Baking: When following a recipe, ingredients are often measured in
la
different units (e.g. cups, tablespoons, teaspoons, grams, ounces). Being able
to convert between these units is important to ensure the recipe turns out
correctly.

Engineering and Technical fields: In engineering and technical fields, unit


conversion is important for understanding and working with technical
specifications, such as for machinery or materials. Engineers and technicians
ho

often need to convert measurements between different units to ensure that


equipment is functioning correctly and safely.

International Trade and Travel: In international trade and travel, it is important


to understand how to convert between different units of measurement, as
measurements can vary from country to country. This is particularly important
in fields such as import/export and logistics.
Sc

Science: In science, it is important to be able to convert between different


units of measurement to compare and analyse data. Scientists often work with
measurements in different units, and the ability to convert between units
allows for more accurate comparisons and analysis.

ix
1. Angela used 2.5 litres of water to fill six glasses.
If Angela evenly distributed the water into the six glasses, how many millilitres of water did she
pour into one glass?

A. 512
B. 416
C. 4123
D. 0413
E. 41623

2. Ruby found a small caterpillar in her garden last week.

The caterpillar was only 35 millimetres long when Ruby found it.

After just one week, the caterpillar grew 4 times as long.

rly
How many centimetres long was the caterpillar after just one week?

A.14 centimetres
B. 28 centimetres
C. 35 centimetres
D. 140 centimetres
E. 280 centimetres

3. Tom placed 15 metal balls inside a pouch. The total mass of the balls was 1.2 kilograms.
la
If each metal ball has the same mass, each metal ball is how many grams?

A. 800 grams
B. 80 grams
C. 8 grams
ho

D. 0.8 grams
E. 0.008 grams

4. Jane used two ribbons to wrap a present.


One of the ribbons was 1 metre long and the other ribbon was 30 centimetres longer than the
other ribbon. What is the combined length, in centimetres, of the two ribbons that Jane used to
wrap the present?
Sc

A. 1.3 centimetres
B. 2.3 centimetres
C. 130 centimetres
D. 230 centimetres
E. 1 300 centimetres

x
5. Karl has 212 hours to answer a three-part test.
He plans to spend the same amount of time on each part of the test.
How many minutes should Karl spend on the second part of the test?

A. 75 minutes
B. 70 minutes
C. 60 minutes
D. 55 minutes
E. 50 minutes

Answers

1. E 2.A 3.B 4.D 5.B

rly
la
ho
Sc

xi
4 Chapter 4:
Money
Problems

rly
la
Basic concepts of solving money problems

Money is a common way to represent numbers


ho

and values in our everyday life. It is important to


understand the basic concepts of solving
money problems, including using money to
represent numbers, making change, calculating
discounts, and understanding currency and
exchange rates. These concepts are essential for
understanding financial concepts and solving
real-world problems.
Sc

xii
Using money to represent numbers

It is important to understand how to represent


money as a decimal or fraction and how to
perform basic operations such as addition,
subtraction, multiplication, and division with
money.

For example, $3.50 represents three dollars and


fifty cents. To add $3.50 and $2.25, you would
add 3.50 + 2.25 = 5.75 or $5.75.

Making change

rly Making change is an important concept related to money.


Making change means determining the amount of money
that is due in coins and bills for a given amount of money. It
requires understanding different denominations of coins
and bills, and how to count them.

For example, if a customer wants to buy a $3.50 item and


pays with a $5 bill, the customer would receive $1.50 in
la
change.

Calculating discounts
ho

Calculating discounts is another important concept


related to money. A discount is a reduction in the original
price of a product or service. To calculate a discount, you
need to understand the relationship between the original
price, the discount rate, and the final price.

For example, if a product has an original price of $100


Sc

and a 10% discount, the final price would be $100 - (0.10


x $100) = $90. Understanding how to calculate discounts
is important when making purchase decisions, it is also
important to be able to compare different discounts, to
find the best deal.

xiii
Exercises

1. Annie spent $3.45 on Pocky sticks every 4. The cost of 5 coloured pencils is $30. Find the
day when on vacation in Bora Bora from cost of 2 packets of coloured pencils if each
27th February 2020 to 8th March 2020. packet has 10 coloured pencils
How much money did she spend on
Pocky sticks? A. $114
B. $120
A. $379.50 C. $132
B. $14.45 D. $126
C. $37.95
D. $6.90 5. Andrew gave three-eighths of his money to his
brother.

rly
2. If 1 South African Rand is worth 0.05 US
Dollars, how many South African Rand is 1
US Dollar worth?

A. 20
B. 1.05
If Andrew was left with $160, how much money
did Andrew originally have?

A. $196
B. $200
C. $256
C. 0.05 D. $400
D. 1.95 E. $436
la
Answers
3. Donna went on a picnic. She spent
$686.18 on the bus fare, $483.93 on food, 1.C
and $663.47 on shopping. If she had two 2. A
$1,000 notes with her, then how much 3. D
ho

amount is left with her now? 4. B


5. C

A. $172.90
B. $170.49
C. $165.18
D. $166.42
Sc

xiv
5 Chapter 5:
Time, Date and
Directions
Problems

rly
la
Basic concepts of time, date and direction
Basic concepts of time, date, and direction are essential for understanding and
solving problems related to time and navigation. In this chapter, we will cover
the following topics:
ho

Telling time: Telling time is a basic concept that is essential for


understanding and solving problems related to time. It is important to
understand how to read a clock, including the hours, minutes, and
seconds, as well as how to use a.m. and p.m. notation. It is also
important to understand how to use time expressions such as "half
past," "quarter to," and "quarter past."
Sc

Example: If the clock reads "3:15," it is "quarter past three" or "3:15."

xv
Understanding Calendars

Understanding calendars is important for understanding


and solving problems related to dates. It is important to
understand how to read and write dates in different
formats, including the day, month, and year. It is also
important to understand how to use a calendar to
determine the day of the week for a given date and how
to use a calendar to determine the number of days
between two dates.

Example:

rly
If today is Wednesday, January 12th, 2022, and you want
to know what day of the week it will be in two weeks, you
can use a calendar to determine that it will be
Wednesday, January 26th, 2022.

Following and giving directions

Following and giving directions is important for


la
understanding and solving problems related to
navigation. It is important to understand how to use
cardinal and ordinal directions (north, south, east, west,
etc.) and to understand how to use landmarks and street
names to give and follow directions. It is also important to
understand how to use a map to give and follow
ho

directions.

Example: If you are heading north on Main Street and you


come to an intersection with Elm Street, to continue on
Main Street, you turn left (west).
Sc

xvi
Exercises

4. Robert took vacation leave from his work that


1.Kim left the library at exactly 3:15 PM, and
lasted 1 week 3 days.
drove to the museum at an average speed
His vacation leave started on Wednesday,
of 40 kilometres per hour.
September 24.
If the museum was 60 kilometres from the
On which day and date will his vacation leave
library, at what time did he arrive at the
end?
museum?

A. Friday, October 2
A. 4:15 PM
B. Friday, October 3
B. 4:30 PM
C. Saturday, October 3
C. 4:45 PM
D. Sunday, October 3
D. 5:00 PM
E. Saturday, October 4
E. 5:15 PM

rly
2. The population of the town of Hillsboro
doubles every year.
If the population of Hillsboro in 2020 was
20 000, at what year was the population of
Hillsboro 10 000?

A. 2019
5. Herbert needs to make four copies of the
report in time for the 10:00 AM meeting today.
Each copy of the report has 80 pages.
If the office printer can print 120 pages per
hour, what is the latest time that Herbert can
start printing the reports and still be in time for
the meeting?
B. 2018
C. 2016
la
A. 7:20 AM
D. 2015
B. 7:30 AM
E. 2010
C. 7:40 AM
D. 8:20 AM
3. A compass is shown below.
E. 8:40 AM
ho

John was facing the northeast before he made


seven quarter turns anti-clockwise.
In what direction was John facing after he made
the turn?
Answers
A. Northwest
Sc

B. West
1.C 2. A 3. E 4. B 5. A
C. Southwest
D. East
E. Southeast

xvii
6 Chapter: 6
Speed,
Distance and
Time Problems

rly
la
Basic concepts of speed, distance, and time, including calculating speed, distance, and time, and
solving related problems
ho

Basic concepts of speed, distance, and time are essential for understanding
and solving problems related to motion and transportation.

Speed is a scalar quantity that is defined as the rate of change of displacement,


and it is measured in units such as meters per second (m/s) or kilometers per hour
(km/h).
Distance is a scalar quantity that is defined as the total length of the path covered
Sc

by an object, and it is measured in units such as meters (m) or kilometers (km).


Time is a scalar quantity that is defined as the duration of an event, and it is
measured in units such as seconds (s) or hours (h). In this chapter, we will cover the
following topics:

xviii
Calculating speed, distance, and time

To calculate speed, you need to know both the distance an object


has travelled and the time it took to travel that distance. The
formula for speed is:

Speed = Distance / Time

For example, if a car travels 60 miles in 2 hours, the speed of the car
is:

Speed = 60 miles / 2 hours = 30 mph

rly
To calculate distance, you need to know both the speed of an
object and the time it took to travel that distance. The formula for
distance is:

Distance = Speed x Time

For example, if a car travels at a speed of 60 mph for 2 hours, the


distance travelled by the car is:
la
Distance = 60 mph x 2 hours = 120 miles

To calculate time, you need to know both the distance an object has
travelled and the speed it was travelling at. The formula for time is:
ho

Time = Distance / Speed

For example, if a car travels 120 miles at a speed of 60 mph, the


time it took the car to travel that distance is:

Time = 120 miles / 60 mph = 2 hours


Sc

xix
Understanding and using rates and unit rates

A rate is a comparison of two quantities, such as miles per hour


(mph) or kilometers per hour (km/h). A unit rate is a rate with a
denominator of 1, such as 60 miles per hour or 80 kilometers per
hour. Understanding and using rates and unit rates is important
for solving problems related to motion and transportation, such
as determining how long it will take to travel a certain distance at
a certain speed, or how far an object will travel at a certain speed
for a certain amount of time.

For example, if you want to determine how long it will take to


travel a distance of 100 miles at a speed of 50 mph, you can use
the formula:

rly
Time = Distance / Speed

Time = 100 miles / 50 mph = 2 hours

Another example, if you want to determine how far an object will


travel at a speed of 40 mph for 3 hours, you can use the formula:

Distance = Speed x Time


la
Distance = 40 mph x 3 hours = 120 miles

Exercises
ho

1. Hank leaves the school and travels to the park at a speed of 22 metres per second.
Six minutes later, Jericho also leaves the school and travels to the park using the same route as
Hank at a speed of 25 metres per second.
If the two of them arrive at the park at the same time, what is the distance between the school
and the park?
Sc

A. 44 kilometres
B. 50 kilometres
C. 55 kilometres
D. 66 kilometres
E. 75 kilometres

xx
2. John leaves Star City and travels to Bay 4. Ben leaves the dormitory at 9:00 AM and
City at the same time that Patrick leaves Bay drives to the movie house at a speed of 40
City and travels to Star City along the same kilometres per hour.
route. Ten minutes later, Jake also leaves the
John can reach Bay City in 25 minutes, which dormitory and drives along the same route as
is half the time it takes Patrick to reach Star Ben to the movie house.
City. How much longer will it take Patrick to If the distance between the dormitory and the
reach the halfway point between the two movie house was 80 kilometres, how fast was
cities than for John to reach the halfway Jake driving if he caught up with Ben at 10:30
point between the two cities? AM?

A. 1212 minutes 1. 45 kilometres per hour


B. 25 minutes 2. 50 kilometres per hour
C. 35 minutes 3. 53 kilometres per hour
D. 50 minutes 4. 55 kilometres per hour

rly
E. 75 minutes

3. Carol and Jenny live on the opposite ends


5. 60 kilometres per hour

5. Reese drives from his house to the mall and


of a straight street. then back to his house, taking the same route
Jenny leaves her house and walks toward both ways.
la
Carol’s house one hour after Carol leaves her
house and walks toward Jenny’s house. He drives at an average speed of 50 kilometres
per hour going to the mall and drives at an
If Carol walks at a rate of 4 kilometres per average speed of 70 kilometres per hour going
hour, Jenny walks at a rate of 6 kilometres back to his house.
per hour, and the distance between their
ho

house is 54 kilometres, what is the total it What is his average speed for the entire round
takes Carol to walk before she meets up with trip?
Jenny?
A. 50 kilometres per hour
A. 3 hours B. 5323 kilometres per hour
B. 4 hours C. 55 kilometres per hour
C. 5 hours D. 5813 kilometres per hour
D. 6 hours E. 60 kilometres per hour
Sc

E. 8 hours

Answers
1. D 2. A 3. D 4. A 5. D

xxi
7
Chapter 7:

Number
Patterns and
Sequences

rly
Basic concepts of number patterns and sequences, including identifying patterns and
la
sequences, and predicting the next number in a sequence

A pattern is a sequence of numbers or shapes that follow a specific rule or relationship.


Identifying patterns is important for understanding and solving problems related to
mathematical patterns and series. There are different types of patterns such as numerical
ho

patterns, geometric patterns, and algebraic patterns.

Numerical patterns

In numerical patterns, the numbers are arranged in a specific order, and the pattern can be
identified by looking at the relationship between the numbers.

For example, in the sequence 2, 4, 6, 8, 10, the pattern is that each number is two more than
Sc

the previous number. This pattern can be written as n+2 where n is the previous number.

xxii
Geometric Patterns

A triangle, followed by a square, followed by a pentagon: This is an example of a


geometric pattern where the number of sides of the shape is increasing by one with
each term.

A circle, followed by an oval, followed by a spiral: This is an example of a geometric


pattern where the shape is changing with each term.

Algebraic patterns

rly
x, 2x, 3x, 4x, 5x: This is an example of an algebraic pattern where each term is a multiple of
the previous term. The pattern can be written as nx where n is a whole number

2y + 3, 3y + 2, 4y + 1: This is an example of an algebraic pattern where each term has a linear


relationship with the previous term.

One of the key skills tested in the OC Mathematical Reasoning test is the ability to identify
patterns in numerical sequences. This may involve recognising a specific rule or
la
relationship between the numbers in a sequence, such as increasing or decreasing by a
certain amount. In the test, you will be presented with a sequence of numbers and asked
to identify the pattern, use that pattern to generate additional terms in the sequence, and
predict the next number in the sequence.

For example, consider the following sequence of numbers: 2, 4, 6, 8, 10. When we look at
ho

this sequence, we can see that each number is two more than the previous number. This
pattern can be written as n+2 where n is the previous number. This pattern can be used to
generate additional terms in the sequence, for example, if we want to find the next
number in the sequence, we can use the pattern n+2 = 12.

To identify a pattern, it is important to look for a consistent rule or relationship between


the numbers. It can be helpful to write out a few terms of the sequence and try to find a
pattern in the numbers. Once a pattern is identified, it can be used to generate additional
Sc

terms in the sequence.

Predicting the next number in a sequence is a skill that is commonly tested in the OC
Mathematical Reasoning test. This skill involves using a specific rule or formula to
determine the next number in a sequence based on the pattern that has been identified.

xxiii
For example, consider the sequence 1, 4, 9, 16, 25. We have identified the pattern in
this sequence is that each number is the square of the previous number. This
pattern can be written as n^2 where n is the previous number. To predict the next
number in the sequence, we use the pattern n^2 = 36

It's important to understand that identifying patterns in numerical sequences is


based on the pattern that has been identified. In some cases, the pattern may be
complex, and it may not be possible to predict the next number in the sequence.

Understanding and using arithmetic and geometric sequences

An arithmetic sequence is a sequence of numbers in which the difference between


any two consecutive terms is a constant. In the OC Mathematical Reasoning test,

rly
you may be asked to find the common difference of an arithmetic sequence,
determine the nth term of an arithmetic sequence, or use an arithmetic sequence to
solve a problem.

For example, in a question, you might be given an arithmetic sequence 2, 5, 8, 11, 14


and asked to find the common difference, which is 3. Or you might be given a
problem where an object is moving at a constant speed and asked to find how far it
will travel in a certain amount of time. In this case, you would use the formula:
la
distance = speed x time, where speed is the common difference and time is the
number of terms.

A geometric sequence is a sequence of numbers in which the ratio of any two


consecutive terms is a constant. In the OC Mathematical Reasoning test, you may
ho

be asked to find the common ratio of a geometric sequence, determine the nth term
of a geometric sequence, or use a geometric sequence to solve a problem.

For example, in a question, you might be given a geometric sequence 2, 4, 8, 16, 32


and asked to find the common ratio, which is 2. Or you might be given a problem
where an object is growing at a constant rate and asked to find its size after a
certain amount of time. In this case, you would use the formula: size = initial size x
(growth rate)^time, where the growth rate is the common ratio and time is the
Sc

number of terms.

xxiv
Exercises

1. The numbers in the boxes below follow a pattern. 4. Philip created a pattern by colouring
some squares on a number chart.
His partially completed number chart
is shown below.
What number comes next?

A. 46
B. 47
C. 48
D. 49
E. 50

2. In the magic square below, each row, each


column, and each diagonal sums up to 15.

rly
What number is N?
What number should he colour
next?

A. 30
A. 4 B. 31
la
B. 5 C. 32
C. 2 D. 33
D. 8 E. 34
E. 10
5. The times on the analogue clocks
3. What number completes the input/output table below follow a pattern.
ho

below?

What time will the next analogue clock


A. 20 show?
Sc

B. 21
C. 25 A. 9:30
D. 28 B. 10:00
E. 30 C. 10:30
D. 11:00
E. 11:30

Answers
xxv 1.B 2.C 3.B 4.C 5.A
8
Chapter 8:
Ratio and
Proportion

rly
Basic concepts of ratio and proportion, including understanding and comparing
la
ratios, and solving proportion problems

Understanding ratio

A ratio is a way of comparing two or more quantities. It is written as a fraction with the
first quantity as the numerator and the second quantity as the denominator.
ho

For example, the ratio of apples to oranges in a basket can be written as 3:5, which
means there are 3 apples for every 5 oranges. Ratios can also be written in the form of a
fraction, such as 35, or as a decimal, such as 0.6.

It is important to remember that the order of the quantities does not matter. For
example, the ratio of apples to oranges (3:5) is equivalent to the ratio of oranges to
apples (5:3)
Sc

Comparing ratios

When comparing ratios, it is important to convert them to a common form, such as a


fraction or decimal.

xxvi
For example, the ratio 3:5 can be converted to a fraction of 35 and a
decimal of 0.6. Once the ratios are in a common form, we can compare
them by comparing the numerator or the decimal form.

Examples:

A ratio of 2:3 is equivalent to a ratio of 4:6, 8:12, and so on.


A ratio of 4:8 is equivalent to a ratio of 1:2, or as a decimal, 0.5

Solving proportion problems using cross-multiplication

A proportion is an equation that states two ratios are equal.

rly
For example, 35 = x10. To solve proportion problems, we can use
cross-multiplication, which is multiplying the numerator of one ratio
by the denominator of the other ratio.
In this example, we would cross-multiply to get 3 * 10 = x * 5. This
results in x = 3*105 = 6.
la
Understanding and using cross-multiplication and proportionality

Proportionality is a relationship between two quantities where one


quantity is a constant multiple of the other.
ho

For example, the relationship between speed and distance travelled


can be described as distance = speed * time. In this example, speed
and time are inversely proportional, meaning that as one increases,
the other decreases.

Example
Sc

A car travels 60 miles in 2 hours, what is its speed?


In this example, we know that distance = speed * time, so we can use
proportionality to calculate the speed. We can find the speed by
solving for it, we get speed = distance / time = 60 / 2 = 30 mph.

xxvii
Exercises

1.The ratio of the current ages of Mike and 4. The ratio of the amount of money in
Noel is 7 to 11. Cecile’s savings account to the amount of
Which of the following could be the ratio of money in John’s savings account is 3 to 2.
their ages 5 years from today?
If Cecile will increase the amount of money
A. 1 to 3 in her savings account by 20 percent and
B. 9 to 20 John will increase the amount of money in
C. 4 to 15 his savings account by $200, the ratio will
D. 3 to 5 become 6 to 5.
E. 2 to 3
How much money is in Cecile’s savings
2. A rectangular garden has a width of 200 account?
centimetres 250 millimetres and a length of
6 metres.

rly
What is the ratio of the width to the length
of the rectangular garden?

A. 1 to 4
B. 1 to 3
C. 3 to 8
A. $200
B. $400
C. $480
D. $600
E. $720

D. 1 to 2
E. 3 to 4 5. In the morning, the ratio of the number of
la
men to the number of women who attended
3. A three-ingredient frosting recipe calls for the lecture was 7 to 3.
butter, sugar, and milk mixed in the ratio of
3:4:12, by volume, respectively. However, by noon 15 of the men left the
The ratio recipe is changed so that the ratio of lecture but 15 additional women joined the
butter to milk is doubled and the ratio of sugar lecture, and the ratio of men to women
ho

to milk is halved. became 13 to 7.

Which of the following is the ratio of the How many more men than women were in
amount of butter to the amount of sugar to the the lecture by noon?
amount of milk in the new recipe?

A. 6 to 2 to 24 A. 105
B. 6 to 1 to 24 B. 90
Sc

C. 6 to 1 to 12 C. 75
D. 3 to 1 to 6 D. 60
E. 3 to 8 to 6 E. 30

Answers

1.E 2.C 3.D 4.D 5.B


xxviii
9
Chapter 9:
Perimeter and
Area Problems

rly
la
Basic concepts of perimeter and area, including calculating perimeter
and area of different shapes
Perimeter
ho

Perimeter is the distance around the outside of a shape. It can be calculated by adding up the
lengths of all the sides of a shape. The formula for perimeter depends on the shape of the object.
It's important to memorise the formulas for the perimeter of common shapes such as rectangles,
squares, circles, triangles, and parallelograms.

Formulas:
Rectangle: P = 2(length + width)
Square: P = 4s (s represents the length of one side)
Sc

Circle: P = 2πr (r represents the radius of the circle)


Triangle: P = a + b + c (a, b, and c represent the lengths of the sides of the triangle)
Parallelogram: P = 2(base + height)
Hexagon: P = 6s (s represents the length of one side)
Octagon: P = 8s (s represents the length of one side)
Trapezoid : P = a + b + c + d (a, b, c, and d represent the lengths of the sides of the trapezoid)

xxix
Examples:

The perimeter of a rectangle with a length of 6 and width of 4 is 2(6+4) = 2(10) = 20


The perimeter of a square with a side of 5 is 4s = 4(5) = 20

Area

Area is the amount of space inside a shape. It can be calculated by multiplying the
length and width of a shape. The formula for area depends on the shape of the object.
It's important to memorise the formulas for the area of common shapes such as
rectangles, squares, circles, triangles, and parallelograms.

Formulas
Rectangle: A = length x width

rly
Square: A = s^2 (s represents the length of one side)
Circle: A = πr^2 (r represents the radius of the circle)
Triangle: A = (1/2)bh (b represents the base of the triangle and h represents the height)
Parallelogram : A = bh (b represents the base of the parallelogram and h represents the
height)
Hexagon: A = ((3√3)/2)s^2 (s represents the length of one side)
Octagon: A = 2(1+√2)s^2 (s represents the length of one side)
Trapezoid : A = ((a+b)h)/2 (a, b represents the lengths of the parallel sides of the
trapezoid, h is the height)
la
Examples:

The area of a rectangle with a length of 6 and width of 4 is 6 x 4 = 24


The area of a square with a side of 5 is s^2 = 5^2 = 25
ho

Understanding the relationship between perimeter and area and solving related
problems

Perimeter and area are related in that the perimeter of a shape can be used to calculate
the area of that shape.

For example, the perimeter of a rectangle can be used to find the length and width,
which can then be used to find the area. Similarly, the perimeter of a circle can be used
Sc

to find the radius, which can then be used to find the area.

ii
xxx
Examples:

1.A rectangle has a perimeter of 40 and an area of 96. What are the length and
width of the rectangle?

In this example, we know that the perimeter is 2(length + width) = 40 and the
area is length x width = 96. We can use these formulas to find the length and
width of the rectangle.
We can solve the equation 2(length+width) = 40 and find that length + width =
20.
Then we can use the second formula to find the length and width, by using
the area formula, length x width = 96.
So, we can find that the length and width are both 12.

2. A square has an area of 64 square units, what is its perimeter?

rly
In this example, we know that the area of a square is side^2 = 64.
We can use this information to find the side of the square by taking the
square root of the area, √64 = 8.
Once we have the side, we can use the formula for the perimeter of a square,
P = 4s, to find the perimeter, P = 4(8) = 32

It's important to understand the formulas for perimeter and area and how to use
them to solve related problems. This includes understanding how to use one
la
formula to find the missing information to use another formula.

Example:

A rectangular garden has an area of 84 square meters and a width of 7 meters.


What is the length of the garden?
ho

In this example, we know that the area of a rectangle is length x width = 84


square meters and the width is 7 meters.
We can use these formulas to find the length of the rectangle.
We can solve the equation length x width = 84 and find that length = 84/7 = 12
meters
Sc

xxxi
Exercises

1.Jerry has a rectangular rug that is 80 4. Ruben built a chicken coop in his backyard.
centimetres long and 40 centimetres wide.
The chicken coop is 7 metres long and has a
What is the perimeter of Jerry’s rug? perimeter of 20 metres.

A. 120 centimetres What is the width of the chicken coop?


B. 240 centimetres
C. 480 centimetres A. 3 metres
D. 1 600 centimetres B. 6 metres
E. 3 200 centimetres C. 9 metres
D. 12 metres
E. 13 metres
2. A sheet of construction paper is 3 metres
long and 2 metres wide.

rly
What is the area of the construction paper?

A. 5 square metres
B. 6 square metres
C. 10 square metres
5. A dressmaker cut a square piece of fabric for
a dress she was making.

The piece of fabric has an area of 16 square


metres.

D. 12 square metres What is the length of the fabric that the


E. 20 square metres dressmaker cut?
la
3. A rectangular swimming pool is 16 metres A. 2 metres
long, 10 metres wide, and 6 metres deep. B. 4 metres
C. 6 metres
How many cubic metres of water is needed to D. 8 metres
fill the swimming pool? E. It cannot be determined.
ho

A. 32 cubic metres
B. 64 cubic metres
C. 480 cubic metres Answers
D. 960 cubic metres 1.B
E. 1 920 cubic metres 2. B
3. D
4. A
Sc

5. B

xxxii
10
Chapter 10:
Line of Symmetry,
Grid References,
Folding and Nets

rly
Basic concepts of line of symmetry, grid references, folding, and nets, including
identifying lines of symmetry, using grid references, understanding nets and
folding, and solving related problems
la
Understanding line of symmetry

A line of symmetry is a line that divides a shape into two identical parts. Shapes that
have at least one line of symmetry are said to be symmetrical. Examples of
symmetrical shapes include squares, circles, and rectangles.
ho

Examples:

A square has 4 lines of symmetry, one line of symmetry can be drawn vertically,
horizontally and diagonally.
A circle has infinite lines of symmetry, any diameter of the circle can be a line of
symmetry
A rectangle has 2 lines of symmetry, one line of symmetry can be drawn
vertically and horizontally
Sc

A triangle has 0 lines of symmetry, because it does not have a line that can
divide it into two identical parts.
Understanding grid references

xxxiii
A grid reference is a set of coordinates used to identify a specific location on a grid.

Examples:

On a 5x5 grid, the grid reference for point A is (3,4)


On a 7x7 grid, the grid reference for point B is (6,2)
On a 8x8 grid, the grid reference for point C is (5,5)

Understanding folding and net

Folding is the process of bending a flat surface along a straight line, without creasing or
tearing the surface. A net is a flat representation of a three-dimensional shape.

Examples:

A cube can be folded into a net with 6 squares, by unfolding the cube, it will show 6

rly
faces, each with the same size and shape.
A cylinder can be folded into a net with 3 circles and 2 rectangles, by unfolding the
cylinder, it will show 2 circles as the base and top and one rectangle as the side of the
cylinder.
A triangular prism can be folded into a net with 3 rectangles and 3 triangles, by unfolding
the triangular prism, it will show 2 triangles as the base and top and 3 rectangles as the
side of the triangular prism.
la
Understanding and using geometric shapes and transformations. Different types of 2D
and 3D shapes are fundamental concepts in geometry.
2D shapes are shapes that have only length and width, they exist on a flat surface, such
as a piece of paper or a whiteboard.
Examples of 2D shapes include squares, rectangles, triangles, circles, and hexagons.
ho

The properties of these shapes are the number of sides, angles, and vertices.

For example, a square has 4 sides, 4 angles, and 4 vertices. A triangle has 3 sides, 3 angles,
and 3 vertices.

3D shapes are shapes that have length, width and height, they have volume and can be
held in space.
Sc

Examples of 3D shapes include cubes, spheres, cylinders, cones, and pyramids.

The properties of these shapes are the number of faces, edges, and vertices.

For example, a cube has 6 faces, 12 edges, and 8 vertices. A sphere has no edges or vertices,
but it has one curved surface.

xxxiv
Transformation is the process of changing the position, size, or orientation of a
shape. Different types of transformations include translations, reflections, and
rotations.

Translation is a transformation that moves a shape a certain distance in a certain


direction without changing its size or orientation.

Reflection is a transformation that creates a mirror image of a shape across a


specific line of reflection, such as the x-axis or y-axis.

Rotation is a transformation that turns a shape around a specific point by a


specific angle, such as hexagons and octagons.

rly Exercises

1.Karen cuts three identical triangles and four identical squares from
construction paper.
la
The sides of the squares and the sides of the triangles have the same length.

Karen plans to use these shapes to form a three-dimensional figure.

Which of the following three-dimensional figures can she form using any
ho

combinations of these seven shapes?

A. A cube
B. A pyramid with a triangular base
C. A pyramid with a square base
D. A prism with a triangular base
E. A prism with a square base
Sc

xxxv
2. Jake made a crossword puzzle.

His completed crossword puzzle is shown below.

Which of the following statements about Jake’s crossword puzzle is true?

A. The crossword puzzle has a vertical line of symmetry only.


B. The crossword puzzle has a horizontal line of symmetry only.

rly
C. The crossword puzzle has both vertical and horizontal lines of symmetry.
D. The crossword puzzle has no line of symmetry.
E. The crossword puzzle has a quarter-turn symmetry.

3. The grid below is made up of 25 identical squares.


la
ho

The grid will be folded in half along the red dashed lines.

Which of these numbers will coincide with the square containing the number 73?

A. 33
Sc

B. 40
C. 42
D. 44
E. 53

xxxvi
4. The grid below shows the location of the ten oldest trees in the forest.

The ranger station is located at C3 and the oldest tree in the forest is located directly
northeast of the ranger station.

What is the grid reference for the location of the oldest tree in the forest?

rly
A. D4
B. D5
C. E3
D. F6
D. G4

5.
la
ho

When the net above is folded along the dashed lines and taped together along the
solid lines, the result is a three-dimensional solid.

The resulting three-dimensional solid will have how many faces?

A. 3
Sc

B. 4
C. 6
D. 8
Answers

1.D 2.A 3.A 4.D 5.B

xxxvii
11
Chapter 11:
Data and Statistics

rly
Basic concepts of data and statistics, including collecting, organising and
la
interpreting data, understanding probability and statistics.
Collecting and Organising Data

Collecting and Organising Data is a fundamental step in data analysis and


statistics. It involves gathering information from various sources and arranging it
ho

in a way that makes it easy to understand and interpret. This process is important
for making sense of the data and drawing valid conclusions.

Different methods of collecting data include surveys, experiments, and


observations. It's important to choose the right method based on the research
question and the type of data being collected.

Once the data is collected, it's important to organise it in a way that makes it easy
Sc

to understand and interpret. There are different types of data, such as categorical
data (data that can be divided into categories) and numerical data (data that can
be measured or counted).

xxxviii
Categorical data can be organised using a frequency table, which lists the
categories and the number of data points that fall into each category.
Numerical data can be organised using a line graph, which shows how a numerical
variable changes over a period of time, or a bar graph, which can be used to
compare different categories.

It's important to choose the most appropriate way to organise the data depending on the type
of data. For example, a frequency table is the best way to organise categorical data, while a line
graph is the best way to show a trend over time.

Interpreting Data

Interpreting Data is a crucial step in data analysis and statistics, after the data is collected
and organised. It involves reading, analysing, and making sense of the data in order to draw
valid conclusions.

rly
One of the ways to interpret data is by using charts and graphs, such as bar graphs, line
graphs, and pie charts. These visual aids make it easy to understand and compare data. It's
important to understand how to read and analyse different types of charts and graphs, such
as how to read a line graph to see the trend over time, or how to read a bar graph to compare
different categories.

Another way to interpret data is by calculating basic statistics, such as mean, median, and
mode. These measures are used to describe the central tendency and spread of the data,
la
providing a summary of the data set and often used to compare different data sets and make
predictions.

Mean is the sum of all the data points divided by the number of data points. For
example, if a student scores 80, 90, 85, and 70 on a test, the mean score is
(80+90+85+70)/4 = 82.5.
ho

Median is the middle value of a data set when it is arranged in numerical order. For
example, if a student scores 80, 90, 85, and 70 on a test, the median score is 85.

Mode is the value that appears most often in a data set. For example, if a student
takes a test and scores 80, 90, 85, 70, 80, 85, 85, the mode score is 85.

Understanding probability and statistics.


Sc

Probability is the branch of mathematics that deals with the measurement of the
likelihood of an event occurring. It is expressed as a number between 0 and 1, with 0
meaning that an event is impossible to occur and 1 meaning that an event is certain to
occur. Different types of probability are theoretical probability and experimental
probability.

xxxix
Theoretical probability is calculated using the number of favourable outcomes over the
total number of possible outcomes. For example, if a coin is flipped, the theoretical
probability of getting heads is 1/2, and the theoretical probability of getting tails is 1/2.
This type of probability is calculated using mathematical formulas and does not involve
conducting any experiments.

Experimental probability is calculated by conducting an experiment and counting the


number of favourable outcomes over a number of trials. For example, if a coin is flipped
10 times and heads come up 5 times, the experimental probability of getting heads is
5/10. This type of probability is calculated by conducting experiments and observing the
outcomes.

rly
Statistics is a branch of mathematics that deals with the collection, analysis,
interpretation, presentation, and organisation of data.

Understanding data representation and visualisation

Data representation and visualisation are essential elements in understanding and


communicating data. It involves organising and presenting data in a way that makes it
easy to understand and interpret. Different types of data representation include
tables, bar graphs, line graphs, and pie charts to effectively communicate data.
la
A table is a method of organising data in a grid format, with rows and columns. Data
can be easily read and compared in a table format. For example, a table can be
used to compare the heights and weights of different athletes.

Bar graphs are a way to visually represent data in a bar format. They are used to
ho

compare different categories, and the height of the bars represents the value of
the data. For example, a bar graph can be used to compare the number of students
in different grades.

Line graphs are a way to visually represent data over time. They are used to show
trends, patterns, and relationships in data. For example, a line graph can be used to
show the population of a city over time.
Sc

Pie charts are a way to visually represent data in a circular format. They are used to
show the proportion of different categories in a data set. For example, a pie chart
can be used to show the percentage of students in different grades in a school.

xl
Exercises

1.A shipping company delivered a total of 22 packages last month.


The dots on the graph below represent the mass and volume of the packages
delivered last month.

How many of the packages with a mass of between 5 and 15


kilograms have a volume of more than 500 cubic centimetres?

rly
A. 3 packages
B. 5 packages
C. 7 packages
D. 8 packages
E. 9 packages

2. The data below shows the percentages of attendance, by gender, at the National
Comic Book Convention.
la
ho

Which of the following statements is true?


The number of girls in attendance in 2016 was more than half the number of boys in
Sc

attendance in 2016.
The number of boys in attendance in 2017 was twice the number of girls in attendance in
2017.
The same number of boys and girls attended the National Comic Book Convention in
2018.

xli
A. I only
B. II only
C. III only
D. I and II only
E. II and III only

3. The bar graph below shows the number of lawyers in selected specialities distributed per
gender.

rly
What percentage of all criminal defence lawyers were female?

8 percent
23 percent
la
75 percent
82 percent
90 percent

4. The circle graph below shows the Anderson household’s annual spending budget.
ho
Sc

xlii
If the Anderson household's annual spending budget was $2 million, which of the following
categories in the circle graph did the Anderson household spend more than $400 000
annually?

I. Electricity
II. Food
III. Rent

A. I only
B. II only
C. III only
D. I and III only
E. I, II, and III

rly
5. The table below shows the percentages of protein, carbohydrates, and fat from three
types of food.
la
Which of the following will supply the most number of grams of
protein?
ho

A. 500 grams of banana


B. 250 grams of boiled egg
C. 350 grams of milk
D. 150 grams of banana and 200 grams of boiled egg
E. 200 grams of boiled egg and 200 grams of milk

Answers:
Sc

1. A
2. C
3. A
4. E
5. E

xliii
12
Chapter 12:
Practise Exercises

1. Frank gave Abe 9 baseball cards, which was 34 the original number of baseball
cards he had.

How many baseball cards did Frank originally have?

A. 12 baseball cards
B. 16 baseball cards

rly
C. 18 baseball cards
D. 20 baseball cards
E. 24 baseball cards

2. The hardware store only sells paints in 3.5-litre cans.


If it takes 12 of a litre to paint one window frame, what is the least number of cans
of paint that George needs to buy if he needs to paint 60 window frames?
la
A. 7 cans of paint
B. 8 cans of paint
C. 9 cans of paint
D. 10 cans of paint
ho

E. 12 cans of paint

3. Half of the students in the group have blue eyes, 13 have green eyes, and the
rest have brown eyes.

Which of the following can be the total number of students in the group?
I. 6 students
Sc

II. 24 students
III. 40 students

A. I only B. II only C. III only D. I and II only E. II and III only

xliv
4.A wizard spent 14 of gold to buy a cauldron and 23 of the remainder to buy a spell
book.

If the wizard had 12 golds left, how many golds did the wizard originally have?

A. 24 golds
B. 36 golds
C.40 golds
D.48 golds
E. 60 golds

5.Amy won a total of 12 gold medals and 18 silver medals in the recent sports fest.

What fraction of the medals that Amy won in the recent sports fest were gold medals?

A. 14
B. 13
C.25
D.12
E. 23
rly
6.When Victor added 12 identical metal balls to the box, the mass of the box increased
by 1.5 kilograms.

If Victor wanted to increase the mass of the box by 2.75 kilograms, how many more such
la
metal balls should he add?

A. 10 metals balls
B. 12 metals balls
C.18 metals balls
ho

D.20 metals balls


E. 22 metals balls

7.The Science Club has 35 male members and 63 female members.

The number of male members of the Science Club is what fraction of the number of
female members?
Sc

A. 720
B. 37
C.59
D.57
E. 79

xlv
8.Carl’s age is 17 of Ben’s age, and Arthur’s age is 14 of Ben’s age.

If Carl is 4 years old, how old is Arthur?

A. 7 years old
B. 14 years old
C. 16 years old
D. 24 years old
E. 28 years old

9.The number of cats at a certain animal shelter is 67 of the


number of dogs.

If there are 18 cats at the animal shelter, what is the total


number of cats and dogs at the animal shelter?

A.
B.
C.
D.
E.
rly
18
21
27
28
39

10.The number of boys in a certain class is 45 of the number of


girls.
la
If there are 16 boys in the class, how many girls are there?

A. 18 girls
B. 20 girls
C. 24 girls
D. 28 girls
ho

E. 30 girls

11.A piece of linen x metres long and y metres wide is priced at $400.

What is the price of the linen, in dollars, per square centimetres?

A. 125xy
Sc

B. 25xy
C. 400xy
D. xy25
E. xy400

xlvi
12.A rectangular water tank measures 3 metres by 5 metres by 8 metres.

How many litres of water is needed to fill the water tank? (1 litre = 1 000 cubic centimetres)

A. 12 litres
B. 120 litres
C.1 200 litres
D.12 000 litres
E. E 120 000 litres

13.An oil tanker can carry 8 564 000 cubic centimetres of oil.

How many cubic metres of oil can the oil tanker carry?

F. 8.564 cubic metres


G.85.64 cubic metres
H.856.4 cubic metres

rly
I. 8 564 cubic metres
J. 85 640 cubic metres

14.If 400 centimetres of wire has a mass of 3 000 grams, 2 metres of such wire has a mass
of how many grams?

A. 1.5 grams
B. 15 grams
la
C.150 grams
D.1 500 grams
E. 15 000 grams

15.If barley sells for d dollars per kilogram, how many cents do b grams of barley cost?
ho

K. db1 000 cents


L. db100 cents
M.db10 cents
N.db cents
O.10db cents

16.At the hardware store, a customer can buy n boxes of nails for d dollars.
How many boxes of nails can a customer buy with c cents?
Sc

A. 100ncd boxes
B. nc100d boxes
C.ndc boxes
D.ncd boxes
E. cdn boxes

xlvii
17.The front end of the train crossed the intersection 2 seconds before the back end of
the train crossed the same intersection.

If the train is 16 metres long and is travelling in a straight line at a constant speed, what
is the speed of the train in kilometres per hour?

P. 21.6 kilometres per hour


Q. 22.2 kilometres per hour
R. 24.6 kilometres per hour
S. 28.8 kilometres per hour
T. 32.4 kilometres per hour

18.A machine can make 55 widgets per minute.

At this rate, how many hours will it take the machine to make 4 400 widgets?

U. 13 of an hour

rly
V. 23 of an hour
W.1 hour
X. 116 hours
Y. 113 hours

19.Tony can run m metres in just 3s seconds.

At this rate, how many minutes will it take Tony to run 10n metres?
la
Z. sm2n minutes
AA. sn2m minutes
BB. 10n3sm minutes
CC. n15sm minutes
DD.mn18s minutes
ho

20.If Jerry can complete 34 of the task in one hour, how many minutes does it take
him to complete the task?

EE. 45 minutes
FF. 75 minutes
Sc

GG.80 minutes
HH.90 minutes
II. 100 minutes

xlviii
21.Andrew gave three-eighths of his money to his brother.

If Andrew was left with $160, how much money did Andrew originally have?

A. $196
B. $200
C.$256
D.$400
E. $436

22.Mike, Noel, and Patrick only had one-dollar bills in their wallets.

The number of one-dollar bills that Mike had is 4 times as many as Noel had and 13 as
many as Patrick had.

If Patrick and Noel, together, had 18 more dollars than Mike, how many one-dollar bills did

rly
Mike have?

A. 2 one-dollar bills
B. 4 one-dollar bills
C.8 one-dollar bills
D.16 one-dollar bills
E. 24 one-dollar bills
la
23.A group of 22 boys and 24 girls only had 10-cent coins in their purses.

All the boys in the group had the same number of 10-cent coins and all the girls in the
group had the same number of 10-cent coins.

If the total value of all the coins in the group was $16, how many 10-cent coins did one
ho

boy have?

A. 1 coin
B. 2 coins
C.3 coins
D.4 coins
E. 5 coins
Sc

xlix
24.Lisa bought a toaster for $280.

If Lisa bought the toaster at 30 percent less than the original price, what was the original
price of the toaster?

F. $348
G.$364
H.$392
I. $400
J. $420

25 Carl went to the grocery store and bought 15 cartons of milk for $316.80.

If the amount he paid included a 10 percent sales tax on each carton of milk, how much
did each carton of milk cost before the sales tax?

A. $18.72

rly
B. $19.20
C.$22.08
D.$22.40
E. $25.38

26 A total of $60 was split evenly among Victor, William, and Yael.
When Victor gave William d dollars, William gave Yael 2d dollars, and Yael gave Victor 3d
dollars, then Victor had exactly $30.
la
How much did Victor give to William?

F.$5
G.$8
H.$10
ho

I. $12
J.$15

27.Carmen went to the supermarket and spent equal amounts buying red apples and green
apples.
Green apples were sold at a rate of 5 green apples for one dollar and red apples were sold at
a rate of 10 red apples for three dollars.
What was the average (arithmetic mean) cost of the apples that Carmen bought?
Sc

A. 20 cents per apple


B. 24 cents per apple
C.25 cents per apple
D.30 cents per apple
E. 32 cents per apple

l
28.During a sale, a customer can buy 2 brownies for 99 cents.

If brownies normally sell for 59 cents each, how much can be saved in buying 10
brownies during the sale?

F. $0.85
G.$0.95
H.$1.10
I. $1.15
J. $2.00

29.A group of nine friends, including Angela, decided to dine in at a restaurant.

Angela spent $8 more than the average amount spent by all nine friends while each of
the remaining eight friends spent $12.

How much did the group of friends spend in all at the restaurant?

C.$114
D.$117
rly
A. $110
B. $112

E. $122

30.Arthur spent 13 of the money in his savings account to buy a new phone and 14 of the
la
remaining amount to buy a new video game.
If $200 was left in Arthur’s savings account, how much money did his savings account
originally have?

F. $400
G.$480
ho

H.$600
I. $800
J. $2 400

31.If October 1, 1982 fell on a Friday, on what day of the week did October 1, 1987 fall?
(Note: It was a leap year in 1984.)

A. Sunday
Sc

B. Monday
C.Tuesday
D.Wednesday
E. Thursday

li
32. Gene can paint a wall in 2 hours while Jeremy can paint the same wall in 1 hour 15
minutes.
Gene started painting the wall at 8:30 AM, and then Jeremy help him at 9:00 AM.
To the nearest minute, at what time will the two of them finish painting the wall?

A. 9:05 AM
B. 9:20 AM
C.9:35 AM
D.9:55 AM
E. 10:25 AM

33. Andrew started cleaning his garage at 3:20 PM.


By 4:20 PM, he had finished cleaning three-fourths of his garage.
At what time will he be able to finish cleaning his garage?

F. 4:30 PM
G.4:35 PM

rly
H.4:40 PM
I. 4:50 PM
J. 5:00 PM

34. If Cory, working alone at a constant rate, started making 50 widgets at 8:00 AM, he
will finish by 8:30 AM.
If Cory and Matthew, working together at their respective constant rates, start making
50 widgets at 9:00 AM, they will finish by 9:20 AM.
If Matthew, working alone at his constant rate, started making 50 widgets at 10:00 AM,
la
at what time will he finish?

K. 10:30 AM
L. 10:45 AM
M.11:00 AM
N.11:30 AM
ho

O.12:00 PM

35. An aeroplane leaves Tinseltown at 10:30 AM, local time, and reaches Greenville at 2:30
AM, local time.
The same aeroplane leaves Greenville at 4:30 PM, local time, and reaches Tinseltown at
4:30 AM, local time.
If the aeroplane takes the same route on both flights and at the same speed, what is the
time difference between the local times of Tinseltown and Greenville?
Sc

P.1 hour 30 minutes


Q.2 hours
R.2 hours 30 minutes
S.3 hours
T.3 hours 30 minutes

lii
36. The population of the City of Townsville increases by 50 percent every 50 years.
The City of Townsville had a population of 8.1 million in 2000.
In what year was the population of the City of Townsville 1.6 million?

U. 1700
V. 1750
W.1800
X. 1850
Y. 1900

37. October 31st, Halloween, fell on a Wednesday in 2001.


On what day of the week did Halloween fall in 2014? (Note: 2004, 2008, and 2012 are
leap years, and thus, had 366 days in a year.)

Z. Monday
AA. Tuesday

rly
BB. Wednesday
CC. Thursday
DD.Friday

38. A water pump started filling an empty water tank at a constant rate.
By noon, the water tank was one-third full, and by 1:15 PM, the water tank was three-
fourths full.
At what time will the water tank be full?
la
EE. 2:00 PM
FF. 2:15 PM
GG. 2:30 PM
HH.2:45 PM
II. 3:00 PM
ho

39. What fraction of an hour had elapsed between 6:55 AM and 7:19 AM?

JJ. 25
KK. 730
LL. 1730
MM.16
NN. 14
Sc

liii
40. The red light flashes once every 9 minutes and the yellow light flashes once every
24 minutes.

If the red light and the yellow light flash at the same time at 7:00 AM, which of the
following times will the two lights flash at the same time again?

A. 7:35 AM
B. 8:16 AM
C. 9:36 AM
D. 11:48 AM
E. 12:00 PM

41. Gerry can run 3.75 kilometres in 15 minutes.

In how many minutes will it take him to run 1 kilometre at the same average speed?

A. 1 minute

rly
B. 2 minutes
C. 3 minutes
D. 4 minutes
E. 5 minutes

42. Yesterday, Agnes spent 30 minutes walking 3 kilometres.

If she doubles her average speed today, how far will she walk in one hour?
la
A. 1 kilometre
B. 3 kilometres
C. 6 kilometres
D. 12 kilometres
E. 24 kilometres
ho

43. A cab driver drove 2 hours less and at an average speed of 5 kilometres per hour
faster on Tuesday than he drove on Monday.
During the two days, the cab spent a total of 20 hours driving a total distance of 1245
kilometres.
What was the cab driver’s average speed on Monday?

F. 55 kilometres per hour


G. 58 kilometres per hour
Sc

H. 60 kilometres per hour


I. 62 kilometres per hour
J. 65 kilometres per hour

liv
44. Jane left the museum, heading north, at a rate of 9 kilometres per hour at 9:00
AM.
At 9:15 AM, Ben also left the museum, heading in the same direction as Jane, at a
rate of 36 kilometres per hour.

How many kilometres must Ben travel before he catches up with Jane?

K. 1 kilometre
L. 3 kilometres
M. 4 kilometres
N. 9 kilometres
O. 12 kilometres

45. Hank drove to a friend’s house, who lived 30 kilometres away from his house,
last weekend.
On his way back, Hank drove twice as fast as he did on his way to his friend’s house.

rly
If Hank spent a total of 6 hours driving, what was his average speed on his way back
from his friend’s house?

P. 5 kilometres per hour


Q. 10 kilometres per hour
R. 14 kilometres per hour
S. 15 kilometres per hour
T. 20 kilometres per hour
la
46. A van and a bus are 100 kilometres apart on a parallel highway.
At 8:00 AM, the van starts travelling toward the bus at an average speed of 60
kilometres per hour.
ho

Half an hour later, the bus starts travelling toward the van at an average speed of
40 kilometres per hour.

How many kilometres will the bus travel before it passes the van on the highway?

U. 25 kilometres
V. 28 kilometres
W.33 kilometres
Sc

X. 35 kilometres
Y. 40 kilometres

lv
47. Leo spent 6 minutes walking from his house to the park.
On his way back home from the park, his average speed is half his average speed on
his way to the park.
How much time will it take Leo to travel two round trips to the park?

Z. 36 minutes
AA.30 minutes
BB.24 minutes
CC.18 minutes
DD.12 minutes

48. Sheila drove to the supermarket at an average speed of 40 kilometres per hour
and then drove back home following the same route.

If Sheila’s average speed for the entire journey was 30 kilometres per hour, what was
her average speed returning home from the supermarket?

A.
B.
C.
D.
rly
20 kilometres per hour
22 kilometres per hour
24 kilometres per hour
26 kilometres per hour
E. 28 kilometres per hour
la
49. A bee flew for three straight hours.

During the first hour, the bee flew a total distance of 86 metres, which was 25 percent
farther than it flew during the second hour.

During the third hour, the bee flew at an average speed of 120 metres per hour for 20
ho

minutes.

What was the total distance that the bee flew during the three hours?

A. 190.6 metres
B. 194.8 metres
C.198.2 metres
Sc

D.204.5 metres
E. 212.8 metres

lvi
50. The hare, who can run 50 percent faster, challenges the turtle in a 75-metre
race.

During the race, the hare had to stop by the cabbage patch to have a snack and
lost 12.5 minutes.

If both the hare and the turtle left the starting line at the same time and reach the
finish line at the same time, how fast was the turtle running?

A. 100 metres per hour


B. 110 metres per hour
C. 120 metres per hour
D.130 metres per hour
E. 230 metres per hour

rly
51. The numbers in the four circles below follow the same pattern.

What number is N?
la
A. 40
B. 44
C. 60
D. 72
E. 80
ho

52. The passcode to a lock is a three-digit number whose hundreds digit is equal to
the sum of the tens digit and the ones digit.
How many possible passcodes are there?

A. 54
B. 45
C.36
D. 27
Sc

E. 18

lvii
53.

The pattern of the numbers in the pyramid above is repeated in the


pyramid below.

What is the value of A + B + C?

F. 3
G.5
H.7
I. 15
J. 25
rly
la
54. Ruby was making 10 bracelets for her friends.

The first bracelet she made contained 5 beads, the second bracelet contained 7
beads, and each succeeding bracelet contained two more beads than the
previous bracelet she made.
ho

What is the total number of bracelets she made?

K. 138 beads
L. 139 beads
M.140 beads
N.156 beads
O.165 beads
Sc

lviii
55. A grocery store worker arranged a total of 120 cans of beans into a stack.

The worker had 1 can of beans on the top of the stack, 3 cans of beans in the
second layer, 6 cans of beans in the third layer, 10 cans of beans in the fourth
layer, and so on.

How many layers of cans of beans did the stack have?

P. 7 layers
Q.8 layers
R. 9 layers
S. 10 layers
T. 12 layers

rly
56. The numbers in the three circles below follow a pattern.
la
ho

U. 6
V. 12
W. 24
X. 48
Y. 60
Sc

lix
57. The ten houses on one side of Emerald Street are numbered in
consecutive even numbers.

The houses are numbered in increasing order from left to right.

If the sum of the numbers of the ten houses in that row is 170, what is the
number of the sixth house from the left?

A. 8
B. 10
C.12
D.16
E. 18

58. George created a number pattern following a certain rule.

rly
The first number in his pattern is -5 and the second number is 5.
Each subsequent number in the odds place is found by adding 5 to the previous
number.
Each subsequent number in the evens place is found by multiplying the previous
number by -1.
What is the sum of the first 449 numbers in George’s number pattern?

Z. -10
AA. -5
la
BB. 0
CC. 5
DD.10
ho
Sc

lx
lxi
Sc
ho
la
rly
lxii
Sc
ho
la
rly
lxiii
Sc
ho
la
rly
70. The current ratio of the number of junior managers to the number of senior
managers in a certain company is 5 to 3.

If 10 of the junior managers are promoted to senior managers, the ratio becomes 7
to 5.

What is the current number of junior managers in the company?

A. 30 junior managers
B.90 junior managers
C. 140 junior managers
D. 150 junior managers
E. 240 junior managers

rly
71. The dimensions of a rectangle are 24 centimetres and 50 centimetres.

What is the area, in square metres, of the rectangle?

A. 0.012 square metres


B. 0.12 square metres
la
C. 1.2 square metres
D. 12 square metres
E. 1 200 square metres

72. The dimensions of a picture are 15 centimetres and 24 centimetres.


ho

The picture has a margin of 3 centimetres wide at the top and the bottom, and a
margin of 2 centimetres wide on each side.

What is the total area of the margin of the picture?

F. 138 square centimetres


I. 162 square centimetres
Sc

J. 180 square centimetres


K. 186 square centimetres
L. 198 square centimetres

lxiv
73. The area of a square is equal to the area of the rectangle.

Which of the following statements is true about the perimeter of the square and the
perimeter of the rectangle?

A. Half the perimeter of the square is equal to the perimeter of the rectangle.
B. The perimeter of the square is equal to the perimeter of the rectangle.
C. Twice the perimeter of the square is equal to the perimeter of the rectangle.
D. Three times the perimeter of the square is equal to twice the perimeter of the
rectangle.
E. None of the above.

rly
la
ho
Sc

lxv
75. The figure below is composed of six identical squares.

The number of square centimetres in the area of the figure is equal to the number
of centimetres in the perimeter of the figure.

What is the length of the side of the square making up the figure?

F. 1 centimetre

rly
G.123 centimetres
H.2 centimetres
I. 213 centimetres
J. 212 centimetres

76. The figure below shows a rectangle inside a square.


la
ho

The rectangle has an area of 48 square centimetres.

Each of the margins at the top and the bottom of the rectangle is 3 centimetres wide, and
each of the margins on both sides is 2 centimetres wide.

What is the length of the side of the square enclosing the rectangle?
Sc

K. 8 centimetres
L. 10 centimetres
M.12 centimetres
N. 16 centimetres
O. 18 centimetres

lxvi
77. In the figure below, ABCD is a square with a side of 1 centimetre.

If E and F are the midpoints of sides AD and CD, respectively, what is


the area of the coloured region?

rly
P. 18 square centimetres
Q. 14 square centimetres
R. 38 square centimetres
S. 58 square centimetres
T. 23 square centimetres

78. The length of the shorter side of a rectangular garden is 75 percent


the length of the longer side.
la
If the perimeter of the garden is 28 metres, what is the area of the
garden?

U. 16 square metres
V. 18 square metres
ho

W.24 square metres


X. 36 square metres
Y. 48 square metres

79. The outside perimeter of a square frame is 360 centimetres.


Each of the sides of the frame is 10 centimetres wide.
What is the inner perimeter of the frame?
Sc

Z. 270 centimetres
AA. 275 centimetres
BB. 280 centimetres
CC. 315 centimetres
DD.320 centimetres

lxvii
80. In the figure below, ABCD is a rectangle, and E and F are midpoints of the
sides BC and CD, respectively.

If the area of the coloured region is 500 square centimetres, what is the
area of rectangle ABCD?

EE. 800 square centimetres


FF. 850 square centimetres
GG. 900 square centimetres
HH. 1 000 square centimetres

rly
II. 1 200 square centimetres

81. The figure below shows a strip of paper that is divided into ten smaller
rectangles of equal size.
The strip is folded vertically along the red line.
Point P will most likely coincide with which of these points?
la
ho

A. Point A
B. Point B
C.Point C
D.Point D
Sc

E. Point E

lxviii
82. How many lines of symmetry does an equilateral triangle have?

F. One
G.Two
H.Three
I. Four
J. Six

83. The markers in the grid below are evenly spaced.

rly
The yellow square has an area of 4 square units.

Which of the three triangles has an area of 9 square units?

K. I only
L. II only
M.III only
N.I and II only
O.II and III only
la
84. Frank is making a crossword puzzle.
His partially completed crossword puzzle is shown below.
ho

If he wants his crossword puzzle to have a half-turn symmetry, what is the


least number of squares he still needs to colour black?
Sc

P. One
Q. Two
R. Three
S. Four
T. Six

lxix
85. The grid below is composed of identical squares.

If the grid will be folded vertically along the dashed line, the red
square will most likely coincide with which of these squares?

U.G2
V. G3
W.H2
X. H4
Y. I2

rly
86. How many lines of symmetry does an isosceles triangle have?

Z. None
AA. One
BB. Two
CC. Three
DD.Six
la
87. The figure below shows the location of ten floating icebergs at sea.
ho

A boat is located at G7 and there is a colony of walruses on the iceberg directly


to the southwest of the boat.

What is the grid reference of the location of the iceberg where the colony of
walruses is located?
Sc

EE. A1
FF. B3
GG. D4
HH. E1
II. F3

lxx
88. The grid below shows the locations of the houses of five friends on the island.

Whose house is located at F7?

JJ. Anna
KK. Greg

rly
LL. Lisa
MM.Ruby
NN.Tim

89. Lisa folded a square piece of paper in half and then folded it again, as
shown in the figure below.
la
Then, two small circles are cut out of the folded paper, as shown below.
ho
Sc

The paper is then unfolded, and placed in the same way as shown originally.

Which of the following shows the cut paper when unfolded?

lxxi
rly
90. How many lines of symmetry does a rectangle have?

TT. None
UU. One
VV. Two
la
WW.Three
XX. Six

91. The line graphs below show the average daily ticket sales and total yearly
revenues of snack stands in Charleston Theatre from 2016 to 2020.
ho
Sc

lxxii
In 2020, the total number of dollars of the revenues of the snack stands in
Charleston Theatre was how many times as great as the average daily number
of tickets sold?

A. 400 times
B. 200 times
C.80 times
D.40 times
E. 20 times

92. All 50 districts in King’s Landing are categorized into district categories
based on population.
The data presented below summarizes how all 50 districts are categorized
based on their population.

rly
la
What percentage of all the districts in King’s Landing belongs to categories 1
to 5?

F. 43 percent
G. 86 percent
H. 88 percent
ho

I. 92 percent
J. 96 percent
93. The bar graphs below show the total barley produced and the total
size of farmlands producing barley from 2000 to 2006.
Sc

lxxiii
In 2005, what is the average (arithmetic mean) number of
tonnes of barley produced per hectare of farmland producing
barley?

K. 1 tonne
L. 1.5 tonnes
M.2 tonnes
N. 2.5 tonnes
O. 3 tonnes

94. The table below shows the worldwide production of car parts
and accessories from 2002 to 2010.

rly
All the values given in the table are in millions of dollars.
la
From 2006 to 2010, the value of car parts and accessories produced by
Japan increased by approximately how many percent?

P. 1 percent
Q. 7 percent
R. 16 percent
ho

S. 19 percent
T. 27 percent

95. The circle graph below shows the worldwide distribution of


sugar production in 2000.
Sc

lxxiv
If the worldwide production of sugar in 2000 was 750 million tonnes, how many
million tonnes of sugar did the United States produce in 2000?

U. 10 million tonnes
V. 25 million tonnes
W. 40 million tonnes
X. 70 million tonnes
Y. 75 million tonnes

96. The bar graphs below show the buying and selling price per share of five
stocks, and the number of shares per stock bought.

rly
Which stock did the total dollar value have the greatest dollar value increase
between buying and selling of the measured period?
la
Z. Stock K
AA. Stock L
BB. Stock M
CC. Stock N
DD. Stock O
ho

97. The data presented below shows the heights, in metres, of the 43 buildings in
a certain district.
Sc

lxxv
EE. 3 metres
FF. 6 metres
GG.16.3 metres
HH.18.2 metres
II. 19.3 metres

98. The bar graph below shows All Night Supermarket’s annual expenses as a
percent of its annual gross revenues in 2015 and 2016.

rly
JJ. Two
KK. Three
LL. Four
MM. Five
la
NN. Six

99. The line graph below shows ACME Publishing’s sales by book category
from 2001 to 2010.
ho
Sc

OO. Two
PP. Three
QQ. Four
RR. Five
SS. Six

lxxvi
100. The bar graph below shows the number of workers, in hundreds,
per department of a certain company.

rly
TT. 444 workers per department
UU. 525 workers per department
VV. 581 workers per department
WW.612 workers per department
la
XX. 628 workers per department
ho
Sc

lxxvii
lxxviii
Sc
ho
la
rly
rly
la
ho

Want free materials and guidance?

Scan the code below:


Sc

lxxix
Scholarly
Steve Xu
Scholarly Publishing
Scholarly Prep
7 TYPES OF QUESTIONS INVOLVED

1. Summarising main conclusion


Conclusion is situated within the text
Task: identify and summarise main conclusion
Filter out: overgeneralisations, assumptions, intermediate
conclusions

2. Drawing a conclusion
The conclusion is not directly stated
Task: infer the main conclusion
Consider evidence, arguments, and premises

3. Identifying an assumption
Assumption – unstated and implicit premise of the
argument
Argument will collapse without the assumption
To identify the assumption: identify conclusion, identify
reasoning, consider ‘unstated portions’ aka. Assumption

4. Assessing impact of additional evidence


Answer options will contain ‘additional evidence’ which
will modify the premise of the argument
Assess which answer option has the greatest impact on
weakening the argument

2
Scholarly Prep
5. Detecting reasoning errors
Explaining why reasons does not = conclusion
Reasoning errors: logical fallacies, over-generalisation,
misguided assumptions, overlooked information etc.

6. Matching arguments
Identifying an argument with the similar logical structure
as that provided in the passage

7. Applying principles
Identify the underlying principles of the argument in the
presented passage
Select the statement (from options A-D) that demonstrates a
similar principle
Helpful to sketch out the scaffold of the argument

3
Scholarly Prep
MAIN LOGICAL FLAWS
1. Causation vs. Correlation: Causation is when one thing directly

affects the other. Correlation is when two things have a similar

trend but do not directly affect each other.

2. Hasty Generalisation: When something applies to one thing, it

does not mean it applies to everything. E.g. The chef at Squidgy

Jiggy is fat, therefore all chefs are fat - FALSE

3. Ignoring other possibilities - do not be limited by the

information in the text

4. Conflation - Drawing inappropriate comparisons to two

different ideas

5. Slippery Slope - Avoid a spiral of events e.g. if you don’t study,

you won’t get a degree, you won’t get a job and you will be

homeless etc. - not always the case

6. Ad hominem attack - Judging a person for their character

rather than facts

7. Strawman argument - Overextrapolating an argument to make

it seem stupid - aka. Rephrasing to twist the argument

8. Red herring fallacy - a distraction within the argument to sway

away from main argument

9. Bandwagon fallacy - because everyone says so, it must be true

10. Appeal to authority - e.g. celebrities give weight loss advice

4
Scholarly Prep
STRENGTHENING AN ARGUMENT -
THE 7 PITFALLS/COMMON MISTAKES

1. Choosing weaker option

2. Choosing irrelevant option

3. Choosing an option that does not directly relate to the

given premise

4. Choosing an option that does not include all the given

premises (reason)

5. Choosing an option that both supports the premise, but

also includes a negativing element

6. Choosing an option that only vaguely connects to the

premise

7. Choosing an option that agrees with the overall

viewpoint, when there are better options

5
Scholarly Prep

LOGICAL STATEMENTS/INDICATORS
(SUFFICIENCY OR NECESSITY?)

Sufficiency (if, when, where etc. - does not exclude)


Necessity (only, only if, must etc. - excludes other
possibilities)

Example:

Steven will get fit if he goes to the gym - Contrapositive ->


If Steven does not go to the gym, he will not have gotten fit.
(It is possible for Steven to get fit without going to the gym)

Steven will get fit only if he goes to the gym -


Contrapositive -> Only if Steven does not go to the gym, he
will not have gotten fit. (It is not possible for Steven to get
fit without going to the gym)

6
‘Thinking Skills Mastery: Achieving Success on The Opportunity Class Test”

The OC Thinking Skills Test is a cognitive ability assessment used to measure an individual's
ability to think critically and solve problems. The test is often used as part of the admission
process for post-secondary education programs. It is important to note that the test is not
content-specific and it assesses the ability of the student to apply reasoning and logic skills
to new situations.

Preparing for the OC Thinking Skills Test can be a challenging task for students and parents
alike. Some common concerns and challenges that students may face include:

● Analytical reasoning: Some students may struggle with understanding and


analysing information presented in the form of tables, graphs, and diagrams. They
may find it difficult to identify patterns and make logical conclusions based on the
information presented.
● Logic and deduction: Some students may have difficulty understanding the logical
relationships between statements and determining the validity of arguments. They
may struggle with identifying logical fallacies and making deductions based on
information presented.
● Problem solving: Some students may find it difficult to identify the problem and
apply logical reasoning to find a solution. They may struggle with breaking down
problems into smaller parts and applying different approaches to solve them.
● Pattern recognition: Some students may have difficulty recognizing patterns and
trends in numerical, visual, or verbal information. They may struggle with identifying
relationships and making predictions based on the information presented.
● Reading comprehension: Some students may have difficulty understanding and
interpreting written passages, and extracting the main ideas and important details
from the text.

Find a range of tips and tricks to help students improve their thinking skills and perform their
best on the OC thinking skills test below:

● Practice with games: The test includes questions that assess a student's ability to
use logical reasoning and deduction. Students can improve their skills by practising
with logical reasoning puzzles, games, and other activities that involve identifying
patterns, relationships and making deductions.
This can be done through some common games such as Sudoku, crossword
puzzles, chess, logic puzzles and brainteasers.

● Encourage learning about coding concepts:

Code reading and analysis: Provide students with code samples and have them
analyse the code and identify the underlying coding concepts and logic.
Example: Code sample to calculate the factorial of a number, the students need to
analyse the code and identify the algorithm, the data structure and the programming
logic used.
Debugging challenges: Provide students with code samples that contain errors and
have them practise debugging the code and identifying the logical errors
Example: A code sample to calculate the area of a rectangle, students need to debug
the code and identify the error that makes the output of the code wrong.

● Use visual aids to understand basic logic concepts:


Study flashcards: Create flashcards with basic logic concepts, such as propositional
logic, predicate logic, and set theory, and have students study them regularly.

Practice Questions for Thinking Skills

1. People should not be penalised if they do not recycle. Despite the fact that recycling
is vitally essential, it should not be compulsory for people to recycle. Only when we
cause annoyance or injury to others can we be penalised. Although not recycling can
hurt the environment, it does not harm others directly.

Which of the following statements, if true, best weakens the above argument?

A. Pollution is hazardous to humans, animals, or plants, but penalties are an


insignificant solution to a wide-scale problem.
B. Penalties give an incentive for people to recycle which encourages them to make
eco-friendly choices.
C. Penalties, in general, are ineffective in changing the way people behave because
they can afford to pay.

D. Inefficient municipal solid waste management can lead to infectious illnesses, land
and water contamination.

2. You will be successful if you get an education. Your education will help you grasp what is
right, bad, fair, and unjust in a more comprehensive way. Your reasoning and critical thinking
abilities will improve as you practise them in real-world circumstances. Thus, education will
definitely elevate a person’s quality of life and understanding of the world around them.

Which of the following statements, if true, best weakens the above argument?

A. Education makes us aware of the world's information, talents, and ethics that we gain
as we evolve and develop.
B. College graduation is meaningless and a waste of time and money.
C. The real world is different from the four corners of the classroom, and you need to be
street smart, not booksmart, to be able to navigate it properly.
D. Some children do not have access to education because of the financial barriers that
prevent them from enrolling in schools.

3. On a normal dice the number on each pair of opposite sides add up to 7.


Which of the following nets shown below is the only one which makes a normal dice
when folded?
4.

5. Rowena: “There is evidence of large decreases in the number and health of shellfish
reefs in Australia, and it has become a major issue. However, environmentalist Rob Jacobs
declared that shellfish reefs are not “functionally dead” and are still thriving in the ecosystem.
I don’t think that’s true, though. He must already own half the population of corals judging
from the display in his living room.”

Which of the following best expresses the flaw in Rowena’s argument?


A. Shellfish are essential members of the ecosystem, and we need a lot more data on
them to make sure they don’t get left out of restoration projects.
B. Rowena’s argument implies that environmentalist Rob Jacob is not a credible source
due to his ownership of corals.
C. The argument assumes a significant threat to shellfish habitats and high demand for
restoration projects.
D. Rowena confidently assumes that poor restoration initiatives by the government
cause the extinction of shellfish.

6. Ellen's friend, Tom, advised her to live separately from her parents since she was already
25 years old. He said that if she continued to live with her parents, she would keep asking for
things from them. He believes that she will not be able to fend for herself in the future.

Which of the following best describes the flaw in the above argument?

A. The argument conflates the causes of a problem with the proper solutions.
B. The available information displays inconsistent results with regard to the argument.
C. The conclusion contradicts the initial assertion.
D. Tom’s argument asserts that living with parents when you're 25 can lead to being
dependent, which in turn will reach a conclusion with no logical basis.
7.

Night furies are the fastest dragons. Some night furies are coloured white. Some night
furries are active during the day. Night furies are gentle and playful dragons.

If the information in the box is true, whose reasoning is correct?

Luke: “Some night furries are active during the day, but all night furies are playful and gentle
animals.”
Ariel: “If an animal is coloure d white, it must be a night fury.”

A. Ariel only
B. Luke only
C. Both Luke and Ariel
D. Neither Luke nor Ariel

8. Terrence has 8 cubic bricks. Each brick has a different letter on each face.
The letters on each brick are:

Brick 1: EONKYM
Brick 2: ATUFKJ
Brick 3: SIRWZK
Brick 4: BCWTFX
Brick 5: BHAZKJ
Brick 6: AMZKJU
Brick 7: LTOFKP
Brick 8: LMPJFK

Which one of the following can Jake not spell out with his bricks?

A. BASEBALL
B. ATHLETIC
C. BATHROOM
D. ALLIANCE

9. In a certain code:

‘Plastics Amber Loser Brothers’ is written as ‘2+t 2!b 2#s 2@h’


‘Arrest Burning Parts Lurk’ is written as ‘2!r 2@n 1+p 1#l’
‘Buster Allergies Labster Parking’ is written as ‘2@t 4!g 2#t 2+k’
‘Another Loggers Priest Boredom’ is written as ‘3!h 2#g 2+r 3@d’

What is the code for “Architecture”?

A. 5!r
B. 5!t
C. 6@h
D. 6#c

10. Wendy went to the market to buy some fruits. There are three shops that offer different
kinds of fruits. She needs to buy the following fruits: 5 Apples, 10 Bananas, 20 Strawberries,
3 Mangoes, 1 Watermelon and 2 Pears. The prices of each piece of fruit are shown in the
table.
Banana Apple Mango Strawberry Watermelon Pear

SHOP A $0.50 $0.80 $0.75 $0.15 $8 $0.30

SHOP B $0.40 $0.70 $0.60 $0.20 $8 $0.40

SHOP C $0.80 $1.20 $1 $0.50 $12 $1.30

Wendy wants to buy all fruits in one shop to avoid inconvenience and to spend as little as
possible.

In which shop should Wendy buy her fruits?


A. Shop A
B. Shop B
C. Shop C
D. Both Shop A and B

11. The table below shows the number of people who took the exams and who failed the
exams for Chemistry (C ), Physics (P), Mathematics (M), Ethics (E), and Algebra (A).

Subject C P M E A

The number of people who took the 1100 1200 900 650 1250
exam

The number of people who failed 720 495 195 450 875
the exam

Which of the following charts represents the percentage of passes in each subject?

A B
C D

12. Netflix offers 3 categories of plan subscription, as follows:

Basic: $8.99
Standard: $13.99
Premium: $17.99

For the month of May, 456 people subscribed for the premium plan, 397 people subscribed
for the standard plan and 168 people subscribed for the basic plan.

Which of the following pie charts best represents the categories of subscription?

A.

B.
C.

D.

13. Find out from amongst the four alternatives as to how the pattern would appear when the
transparent sheet is folded at the dotted line.

14. A sculpture is placed in a glass box and displayed in a museum as shown in the diagram
below. The sculpture can be viewed from all sides. Which one of the following is not a
possible view from one of the sides of the glass box?
15. When viewed from above, an object looks like the picture shown below. Which one of the
following is not a possible view from the side?
16. Three students - Kent, Eugene, and Mos were suspected of stealing whiteboard
markers. Their teacher knows that 2 of them are guilty. When faced with a five-day
suspension, they made the following statements:

Kent: Eugene is not guilty.


Eugene: I am guilty.
Mos: I am guilty.

However, only 1 of them is telling the truth; the other 2 are lying.

Which of the following statements must be true?

A. Kent is guilty and lying


B. Eugene is lying
C. Mos is guilty
D. Mos is not lying

17. Kyla is travelling to Korea to watch Twice’s concert at the Garden Square. She needs to
buy tickets for the following dates of August: 2,3,4,5,6,10,11,16,17,18, and 22. The prices of
the tickets are shown below:

Pass for 1 day Pass for 3 days Pass for 5 days

$349 $1035 $1500

What is the cheapest valid combination Kyla could use?

A. $2450
B. $2995
C. $3582
D. $4450
18. There are various skill-enhancing classes where students can enrol to. Mandie needs to
choose one class per list.

LIST 1 LIST 2 LIST 3 LIST 4


Theatre Class Painting Class Painting Class Piano Class

Singing Class Piano Class Singing Class Swimming Class

Acting Class Violin Class Swimming CLass Drawing Class

Drawing Class Dance Class Writing Class Writing Class

Mandie chose the following classes: Painting Class, Acting Class, and Swimming Class.

Which of the following classes can Mandie not choose as her final class?

A. Violin Class
B. Theatre Class
C. Piano Class
D. Singing Class

19. Margie went shopping with her mother. After shopping, her mother asked her to plot all
their expenses in a bar graph because she wants to see where they spent the most money
on. Their expenses are shown in the table.

Shoes Bags Clothes Jewellery

$850 $1675 $1250 $1500

Which of the following bar charts best represents the data in the table?

A.
B.

C.

D.

20. Chesca, Lalaine, Joy, and Jesmine are wearing the same Casio Baby G watches in
different colours: white, black, blue, and pink.

- Chesca and Jesmine are not wearing black or blue.


- Joy doesn't like blue but wears it anyway.
- Chesca wore a pink watch.

What colour of Casio Baby G watch did Lalaine wear?

A. White
B. Black
C. Blue
D. Pink

Answers

1. B
Explanation: The main argument suggests that a penalty should not be given to people who
do not recycle because recycling isn’t required from them. Option B directly weakens the
argument by stating that people’s behaviour towards penalties translates to more
eco-friendly choices because people prefer that over spending money for it. Option A fails to
link the severity of pollution to the effectiveness of penalties. Option C makes an assumption
that everyone can pay the penalty. Option D merely states the effects of inefficient municipal
solid waste management.

2. C
Explanation: The main argument suggests that a person will become successful if he or she
gets an education. In this case, statement C clearly weakens the argument the most
because it argues the opposite. It’s saying that, in the real world, being street wise is more
beneficial for a person to survive. The other options strengthen the above argument.

3. D
Explanation: Only option D had all pairs of opposite sides add up to 7.

4. C
Explanation:
The only possible fit of this shape is option C where the both figures will form a cube.

5. B
Explanation: In the passage, Rowena tries to discredit environmentalist Rob Jacobs’ claim
on the current state of shellfish in the ecosystem by using his ownership of corals against
him, but fails to provide a logical thought process as to how one’s personal belongings hold
any value to their opinions.

6. D
Explanation: The argument is flawed because it concludes that an action will lead to a
conclusion with ridiculous outcomes with no evidence to support the claim. In this case, Tom
concluded that if Ellen continued to ask things from her parents, there will come a time that
they won't be able to support her, and because of this, Ellen won’t be able to stand alone.
Notice that there is not enough data to prove the likelihood of each outcome.

7. B
Explanation: Luke is correct because according to the conditions, some night furies are
indeed active during the day and they are all described as gentle and playful dragons. On
the other hand, Ariel is incorrect because the condition only says that some night furies are
coloured white. She incorrectly categorised all animals that are coloured white and labelled
them as night furies when it could be other animals as well.
8. D
Explanation: The following words can be spelt using the indicated arrangement of bricks:
● BASEBALL = 5-2-3-1-4-6-7-8
● ATHLETIC = 6-2-5-8-1-7-3-4
● BATHROOM = 4-6-2-5-3-1-7-8
Only the word “ALLIANCE” cannot be spelt using the given bricks.
● ALLIANCE = 6-7-8-3-2-1-4-_
The word is impossible to spell since the only letter E and the only letter N belong to the
same cube.
9. A
Explanation: The first character of the code is the number of vowel letters of the word.

The second character of the code is a symbol assigned to the first letter of the word:

+ for P ! for A # for L @ for B

The third character of the code is the first consonant letter before the last vowel letter of the
word.

Example: Plastics is coded as 2+t.

The word ARCHITECTURE has five vowel letters then the first character of the code is “5.”
Since ARCHITECTURE starts with the letter A, then the second character is a “!.” Lastly, the
third character of the code is the first consonant letter before the last vowel letter of the word
which is “r.”

Thus, the code for the word ARCHITECTURE is 5!r.

10. B
Explanation:
In Shop A:
5 Apples x $0.80 = $4
10 Bananas x $0.50 = $5
20 Strawberries x 0.15 = $3
3 Mangoes x $0.75 = $2.25
1 Watermelon x 8 = $8
2 Pears x 0.30 = $0.60

$4 + $5 + $3 + $2.25 + $8 + $0.60 = $22.85

In Shop B:
5 Apples x $0.70 = $3.5
10 Bananas x $0.40 = $4
20 Strawberries x 0.20 = $4
3 Mangoes x $0.60 = $1.8
1 Watermelon x 8 = $8
2 Pears x 0.40 = $0.80

$3.5 + $4 + $4 + $1.8 + $8 + $0.80 = $22.1

In Shop C:
5 Apples = $6
10 Bananas = $8
20 Strawberries = $10
3 Mangoes = $3
1 Watermelon = $12
2 Pears = $2.60

$6 + $8 + $10 + $3 + $12 + $2.60 = $41.60

Wendy should buy in Shop B because it offers the cheapest fruits. She would only have to
spend $22.1 for all the fruits that she needs.
11. C
Explanation: The number of people who passed each subject can be calculated by
subtracting the number of people who failed from the number of people who took the exam.
The percentage of passes can then be identified by dividing the number of people who
passed by the number of people who took the exam and then multiplying them by 100. The
summary of results from these calculations is shown in the table below.
From the answer options, Option C best reflects the percentage of passes for each subject.

12. A
Explanation: The pie chart in option A best represents the subscriptions per category.

13. B
Explanation: Option A is not the answer since there should be a shaded circle..
Option C is not the answer since there should be a shaded circle.

Option D is not the answer since a cone segment of the circle should not be shaded.

Option B satisfies the condition when folded.

14. D

Explanation: Option A is the Front View.

Option B is the Right View.

Option C is the Top View.

Option D should have a line emphasising an extruded part.

15. A

Explanation: Option B, C, and D satisfy the conditions when projected. Option A is not the
answer since the second layer is almost the same width as the third layer.

16. A

Explanation: We know that two people are lying and one person is telling the truth. By
looking at the table, we can identify who’s lying and who’s telling the truth. It is evident that
Kent and Mos are lying and Eugene is telling the truth.

Eugene is telling the truth. Kent and Mos are both lying. It is true that Kent is guilty and lying.
17. C
Explanation: Kyla could use the following combination:
Three 1-day passes for July 10, 11, and 22
= 3 x $349 = $1047
One 3-day pass for July 16, 17, and 18
= $1035
One 5-day pass for July 2,3,4,5, and 6
= $1500

$1047 + $1035 + $1500 = $3582

18. B
Explanation: Mandie cannot choose the Theatre Class because it is on the same list as
Acting Class. Option B is the correct answer.
LIST 1 LIST 2 LIST 3 LIST 4

Theatre Class Painting Class Painting Class Piano Class

Singing Class Piano Class Singing Class Swimming Class

Acting Class Violin Class Swimming Class Drawing Class

Drawing Class Dance Class Writing Class Writing Class

19. A

Explanation:

Shoes Bags Clothes Jewellery

$850 $1675 $1250 $1500

The bar graph in option A best matches the data in the table.

20. B
Explanation: We know that Chesca is wearing pink. Jesmine is not wearing black or blue.
Joy does not like the blue-coloured watch but she still wore it anyway. We can conclude that
Jesmine is wearing white since we know that she is not wearing black and the blue colour
was used by Joy. Lalaine is wearing a black coloured watch because all the other colours
were used except for it.
"I hope you have found our PDF to be of great benefit to you and your child's learning
experience. If you're looking for additional resources to enhance your child's education,
check out our programs at https://scholarlytraining.com/scholarly-plus/. Happy learning!"

"To gain a better understanding of your child's educational progress and to see how they
compare with their peers, we invite you to take advantage of our complimentary assessment.
Please complete the form provided to get started."https://scholarlytraining.com/contact-us/“
- Steve Xu
Read the story below then answer the questions.

Paid in Full

A little boy came up to his mother in the kitchen one evening while she was fixing supper, and he
handed her a piece of paper that he had been writing on. After his mom dried her hands on an apron,
she read it, and this is what it said:
For cutting the grass: $5.00
For cleaning up my room this week: $1.00
For going to the store for you: $.50
Baby-sitting my kid brother while you went shopping: $.25
Taking out the garbage: $1.00
For getting a good report card: $5.00
For cleaning up and raking the yard: $2.00
Total owed: $14.75

Well, his mother looked at him standing there, and the boy could see the memories flashing through her
mind. She picked up the pen, turned over the paper he had written on, and this is what she wrote:

For the nine months I carried you while you were growing inside me: No Charge.
For all the nights that I’ve sat up with you, doctored and prayed for you: No Charge.
For all the trying times, and all the tears that you’ve caused through the years: No Charge.
For all the nights filled with dread, and for the worries I knew were ahead: No Charge.
For the toys, food, clothes, and even wiping your nose: No Charge.
When you add it up, the cost of my love is: No Charge.

When the boy finished reading what his mother had written, there were big tears in his eyes, and he
looked straight up at his mother and said, “Mom, I sure do love you.”

And then he took the pen and in great big letters he wrote:
“PAID IN FULL.”

Answer the following questions by choosing the letter of the best answer.

1 What is the main message conveyed in the story "Paid in Full"?

A. Love and sacrifice are priceless and cannot be measured in monetary terms.
B. Children should be grateful and repay their parents for their efforts.
C. The mother should have charged her son for the chores he listed.
D. The boy's actions demonstrate the importance of financial responsibility.
2 What is the significance of the mother's response to her son's list of chores and their
assigned values?

A. She emphasises the importance of financial transactions within a family.


B. She demonstrates her disappointment in her son's attempt to assign monetary value to their
relationship.
C. She highlights the effort and sacrifices she has made as a mother, which cannot be measured in
monetary terms.
D. She teaches her son the importance of fair compensation for household chores.

3 What does the boy's response, "PAID IN FULL," signify?

A. He acknowledges the financial value of his mother's love and sacrifices.


B. He realises that he can never repay his mother fully for her love and care.
C. He believes his actions have compensated for the costs mentioned in his list of chores.
D. He intends to start making financial contributions to the family to express his gratitude.

4 What is the effect of the mother's response on the boy?

A. He feels obligated to repay his mother financially for her efforts.


B. He becomes emotionally overwhelmed and expresses his love for his mother.
C. He learns the importance of assigning monetary value to household chores.
D. He regrets his previous demands for payment from his mother.

5 What does the mother's decision to write "No Charge" signify?

A. She expects her son to repay her for her efforts in the future.
B. She dismisses the importance of her sacrifices as a mother.
C. She emphasises the immeasurable value of her love and care.
D. She intends to teach her son a lesson about financial responsibility.

6 What is the purpose of the boy's list of chores and their assigned values?

A. To showcase his sense of responsibility and financial understanding.


B. To demonstrate his desire for financial compensation from his mother.
C. To express his gratitude for his mother's efforts through financial means.
D. To highlight the tasks he has completed and the time and effort invested.

7 What is the overall tone of the story "Paid in Full"?

A. Gratitude and appreciation


B. Resentment and entitlement
C. Financial responsibility and accountability
D. Sacrifice and selflessness
8 What does the story teach us about the value of love and care?

A. Love and care can be measured and quantified.


B. Love and care should be repaid with financial compensation.
C. Love and care are invaluable and cannot be repaid in material terms.
D. Love and care should be reciprocated with acts of service and obedience.

Read the poem below then answer the questions.

maggie and milly and molly and may by E. E. Cummings

maggie and milly and molly and may


went down to the beach(to play one day)

and maggie discovered a shell that sang


so sweetly she couldn't remember her troubles,and

milly befriended a stranded star 5


whose rays five languid fingers were;

and molly was chased by a horrible thing


which raced sideways while blowing bubbles:and

may came home with a smooth round stone


as small as a world and as large as alone. 10

For whatever we lose(like a you or a me)


it's always ourselves we find in the sea

Answer the following questions by choosing the letter of the best answer.

9 Which figure of speech is used in the title of the poem?

A. Simile
B. Metaphor
C. Alliteration
D. Personification

10 Where does this poem take place?

A. Forest
B. Beach
C. Land
D. Mountain
11 In lines 5 and 6, what kind of creature was the poet talking about?

A. Shell
B. Crab
C. Stone
D. Starfish

12 In line 3, which figure of speech is utilised?

A. Simile
B. Metaphor
C. Alliteration
D. Personification

13 What was the souvenir that May took home from the beach?

A. Shell
B. Crab
C. Stone
D. Starfish

14 Which figure of speech is used in lines 10 and 11?

A. Simile
B. Metaphor
C. Alliteration
D. Personification

Read the extracts below and answer the questions.

A A few feet next to me an eager chipmunk C Rainforests play a critical role in the
hastily scampered from tree to tree, atmosphere in part because they hold
awaiting the chill of winter. The forest, I vast reserves of carbon in their
realised, was home to many wild vegetation. When rainforests are burned,
creatures. In giving protection and food, or the trees are cut and left to decay, the
the forest was gladly rewarded with the carbon is released into the atmosphere
company of these animals. Beyond the as carbon dioxide. This is the second
horizon, I could see the community of largest factor contributing to the
newly formed saplings. They appeared as greenhouse effect. The rainforests are
little children, learning under the guidance being destroyed every second. They are
of their grown and fully matured parents. being cut down for short-term economic
The forest was pure and clean, as though benefit. Once an area is destroyed, it can
it had never been disturbed by man and never again return to the amazing
his vicious life-killing machines. ecosystem that it once was. It takes
millions of years to create, and seconds
to destroy.
B Off we go into the rainforest and there is a D The rainforests are very important to the
spider web luckily we have a man in the world for many reasons, most of them
group so Mark picks up a stick and wraps being very simple. One major reason is
the web around it and throws the stick that the plants in the forest turn carbon
away. We keep on walking and there in dioxide into clean air, which helps us fight
another path I take it thinking that my pollution. Also, by absorbing carbon
friends would take it too. I keep walking dioxide, the rainforests help deter the
for a couple of minutes and I say “guys greenhouse effect. The trees of the
stop playing games with me” I turn around rainforest store carbon dioxide in their
and they aren’t there I yell and scream roots, stems, branches, and leaves. The
but there are so many trees it block all of plants and animals of the rainforest also
the sound. provide us with food, fuel wood, shelter,
jobs, and medicines.

Which extract mentions…

15 about forest plants consuming carbon dioxide to create oxygen which limits
pollution?

16 that the forest was pure and was not disturbed by man and machines?

17 that the writer accidentally got lost in the forest?

18 that rainforests are being destroyed for financial profit?

19 about a spider web in the rainforest?

20 that the forest shelters and provides for numerous wild animals?
ANSWERS:

Fiction: Paid in Full

1. A
2. C
3. B
4. B
5. C
6. D
7. A
8. C

Poetry: maggie and milly and molly and may by E. E. Cummings

9. C
10. B
11. D
12. D
13. C
14. A

Evaluation: About forests

15. D
16. A
17. B
18. C
19. B
20. A
1 1
1 Gandalf spent 4
of his gold to buy a healing potion and spent 3
of what he had left to
buy a spell book.

He then had only 10 golds left.

How many golds did Gandalf originally have?

A 20 golds D 30 golds
B 24 golds E 40 golds
C 25 golds

2 A rectangular courtyard is 18 metres long and 15 metres wide.

A walkway 3 metres wide will be added around the courtyard.

What is the total area of the walkway?

A 108 square metres D 270 square metres


B 192 square metres E 378 square metres
C 234 square metres

3 A fruit vendor makes a profit of $1.50 for every watermelon she sells.

How many watermelons must the fruit vendor sell to make a total profit of $27?

A 16 watermelons D 20 watermelons
B 18 watermelons E 24 watermelons
C 19 watermelons

4 Steve Jobs is paid $300 a month plus a commission of $20 for each iPhone he sells.

How many iPhones must Steve Jobs sell to make at least $540 a month?

A 20 iPhones D 14 iPhones

B 18 iPhones E 12 iPhones
C 16 iPhones
5 The length of a rectangular garden is thrice its width.

If the area of the garden is 48 square metres, what is the length of the garden?

A 3 metres D 12 metres
B 4 metres E 15 metres
C 10 metres

1
6 One-sixth of the passengers in an aeroplane were seated in business class, 3
were
seated in the tourist class, and the rest were seated in economy class.

If there are 150 passengers in the aeroplane, how many passengers were seated in
economy class?

A 65 passengers D 80 passengers
B 70 passengers E 85 passengers
C 75 passengers

7 In a recent excavation in Rome, Indiana Jones found a coin marked by the Roman
numeral XXIV.

What does this Roman numeral represent?

A 24 D 45
B 26 E 46
C 44

8 Jane is 24 years younger than Mike.

If Jane is 16 years old, how old is Mike?

A 8 years old D 40 years old


B 16 years old E 44 years old
C 24 years old
9 Rocky wants to make a square boxing ring such that 10 trips around the boxing ring will
equal exactly 1 kilometre.

How many metres should the length of the side of the boxing ring be?

A 20 metres D 50 metres
B 25 metres E 100 metres
C 40 metres

10 Frank works on his History paper for 2 hours and 15 minutes.

If he started at 6:50 PM, what time did he finish his History paper?

A 9:15 PM D 8:35 PM
B 9:05 PM E 8:05 PM
C 8:55 PM

1
11 Batman rode his Batmobile to travel a distance of 75 kilometres in 2 2 hours.

How far can the Batmobile travel in 6 hours?

A 150 kilometres D 180 kilometres


B 160 kilometres E 195 kilometres
C 175 kilometres

12 The length of a rectangle window is four times its width.

If the perimeter of the window is 20 metres, what is the area of the window?

A 12 square metres D 25 square metres


B 16 square metres E 36 square metres
C 20 square metres
13 The aeroplane from Sydney was scheduled to arrive at 2:30 PM but arrived 45 minutes
early.

At what time did the aeroplane arrive?

A 1:45 PM D 12:45 PM
B 1:30 PM E 12:15 PM
C 1:15 PM

14 Spiderman bought 50 capsules of web slingers and packed them equally into 6 boxes.

How many full boxes did Spiderman have?

A 2 full boxes D 8 full boxes


B 4 full boxes E 9 full boxes
C 6 full boxes

15 Tony Stark has a garden that is 12 metres long and 8 metres wide.

The posts for the fence around the garden need to be 4 metres apart.

How many posts does the fence need?

A 10 posts D 20 posts
B 12 posts E 40 posts
C 15 posts

16 The scale on a map shows that 1 centimetre is equal to an actual distance of 25


kilometres.

If the distance between two cities is 9.5 centimetres on the map, what is the actual
distance between the two cities?

A 195.5 kilometres D 244 kilometres


B 212 kilometres E 250 kilometres
C 237.5 kilometres
5
17 A recipe for cake requires 6
of a cup of sugar.

1
If Michelle only has only 4
of a cup of sugar, how much more sugar does she need?

1 1
A 4
of a cup D 2
of a cup
1 7
B 3
of a cup E 12
of a cup
5
C 12
of a cup

1 2
18 Mrs. Thompson went to the supermarket and bought 2 6 kilograms of pork, 3 3
1
kilograms beef and 2 2 kilograms of lamb.

How many kilograms of meat did she buy altogether?

1 1
A 7 3 kilograms D 8 2 kilograms
1 3
B 7 2 kilograms E 8 4 kilograms
1
C 8 3 kilograms

19 The Hulk has a piece of wire that is 44 centimetres long.

He bent the wire to form a square.

If there were no leftover wires, what is the area of the square that The Hulk made?

A 121 square centimetres D 144 square centimetres


B 130 square centimetres E 160 square centimetres
C 136 square centimetres

20 Last week, Fred Flintstone conducted a lecture that started at 9:30 AM and ended at
2:15 PM.

How long did his lecture last?

A 4 hours 15 minutes D 5 hours 25 minutes


B 4 hours 45 minutes E 5 hours 45 minutes
C 5 hours 15 minutes
ANSWER KEY
QUESTION CORRECT ANSWER
A
Given:
¼ G = Healing Potion
⅓ Remaining = Spell book
10 golds left

1 Solution:
G - ¼ G = ¾ G ← Left after buying healing potion
1
¾ G - (¾ G) (⅓) = ( 2 𝐺)
1
2
𝐺 = 10
G = 20 golds

C
Given:
18 m x 15 m
walkway = 3m

2 Solution:
A of whole = (18 + 6) (15 + 6) = 504 m^2
A of courtyard = 18 x 15 = 270 m^2

504 - 270 = 234 m^2

B
Given:
Profit = $1.5 for each water melon
$26 = ?
3
Solution:
27/1.5 = 18 watermelons

E
Given:
$300 per month plus $20 commission per iphone
$540 = ?
4
Solution:
Let X = number of iphone
300 + 20(X) = 540
20X = 240
X = 12 iPhones
D
Given:
L = 3W
A = 48

Solution:
5 48 = LW ← L = 3W
48 = 3W (W)
16 = W (W)
W=4

L = 3W = 3(4) = 12 m

C
Given:
⅙ P = Business class
⅓ P = Tourist class
Remaining = economy class

150 passenger

6 Solution:
P - ⅙ P = ⅚ P ← Remove the business class
⅚ P - ⅓ P = ½ P ← Remove the Tourist class

Economy class = ½ P
P = 150

Economy = ½ (150) = 75 passengers

A
Given:
XXIV

Solution:
X = 10
7 I=1
V=5
It should be arrange from highest to lowest
If Low value is at first it will be negative

XXIV = 24
D
Given:
J = M - 24
J = 16 years old
8
Solution:
J = M - 24
16 = M - 24
M = 40 years old

B
Given:
1 km = 10 trips around the boxing ring

Solution:
9
1 km = 1000 m

1000/10 = 100 m ← 1 trip around the boxing ring


S = 100/4 = 25 m

B
Given:
History = 2 hours 15 mins
Start = 6:50 Pm
10
Solution:
6:50 PM + 2 hours + 15 mins = 9:05 PM

D
Given:
D = 75 km for 2 ½ hours

Solution:
11 V = d/t
V = 75/(2 ½)
V = 30 km per hour

6(30) = 180 km
B
Given:
L = 4W
P = 20 m

Solution:
P = 2(L + W)
12
20 = 2(4W + W)
10 = 5W
W=2m
L = 4W = 4(2) = 8 m

A = LW = 8 (2) = 16 m^2

A
Given:
Arrive = 2:30 PM
45 mins early
13
Solution:
Arrival = 2:30 - 45 mins = 1:45 PM

D
Given:
50 capsules
pack into 6 boxes
14
Solution:
50/6 = 8.3333 boxes
Therefore he have 8 full boxes

A
Given:
Garden = 12m x 8m
4 meters apart = Fence

15 Solution:
P = 2( 12 + 8)
P = 40 meters

40/4 = 10 posts
C
Given:
1 cm = 25 km

16 distance = 9.5 cm

Solution:
25 𝑘𝑚
9.5 cm x 1 𝑐𝑚
= 237.5 km

E
Given:
⅚ sugar = cake
¼ cup
17
Solution:
7
⅚ - ¼ = 12 of a cup

C
Given:
1 2 1
2 6 kilograms of pork, 3 3 kilograms beef and 2 2 kilograms of lamb.
18
Solution:
2 ⅙ + 3 ⅔ + 2 ½ = 8 ⅓ kg

A
Given:
44 cm = Wire

Solution:
19 P = 44 cm
44 = 4S
S = 11 cm

A = 11 x 11 = 121 cm^2

B
Given:
9:30 AM to 2:15 PM
20
Solution:
2:15 PM - 9:30 AM = 4 hours and 45 mins
NO. QUESTION

1 Chadwick is looking for a hotel with a Wifi connection. He is choosing between the four major hotels in the city. The data
below shows the rates per day and the rates for wifi in dollars.

Which of the following hotels should he choose if he wants to spend as little as possible?

A. Hotel A
B. Hotel B
C. Hotel C
D. Hotel D

2 Alisa, Brian, and Jerome are college students. One is taking a degree program in Tourism; another is in Nursing, and another
one is in Architecture. Each of them has one specific hobby that is different from everyone else’s. The hobbies in question are
cooking, crocheting, and painting.

- Alisa is a nursing student.


- The tourism student likes painting.
- Brian loves to cook.
- The nursing student likes crocheting.

What is Jerome’s hobby?

A. Crocheting
B. Cooking
C. Painting
D. Cannot be determined due to lack of information.
3 Road freight is a more sensible choice when it comes to localised freight, even if air freight is excellent for shipments to other
countries (sea freight is a possibility if you're ready to wait longer). A method like this has a low cost. Your expenditures will
go up if you rely more on air freight. This is due to the apparent reality that maintaining and operating an aeroplane is far
more expensive than keeping and using a freight vehicle.

Which of the following statements, if true, best strengthens the above argument?

A. Road Transport uses less capital expense.


B. Road Transport does not require buying or hiring a crate for travel. Cages are built into the vehicles and floats.
C. Road Transport alongside other animals can be a positive social experience for your dog.
D. Road Transport delays the delivery of your package.

4 Miguel is a director and he wants to create four new movies but can’t decide what genre. The scriptwriter gave him four lists
of genres, and he needed to choose only one from each list.

LIST 1 LIST 2 LIST 3 LIST 4

horror historical drama musical

scifi animated action historical

comedy action silent scifi

Which of the following can he not choose if he already chose horror, action, and musical?

A. silent
B. historical
C. drama
D. scifi

5 Cats and dogs are the most popular pets in the world. Cats are more independent and are generally cheaper and less
demanding pets. Dogs are loyal and obedient but require more attention and exercise, including regular walks. Therefore,
some people prefer dogs because they are more affectionate than cats.

Which of the following statements, if true, best strengthens the above argument?

A. Dogs are warmhearted and loving.


B. Dogs always get excited when seeing their owners.
C. Cats can be left alone in a house.
D. Cats are easily trained to use a litter box.
6 Jake wants to buy 9 boxes of pizza. There are four shops that sell the products he wants. The shops, the prices, and special
offers are shown in the table below.

Shop Standard price Special offer


Pop’s Pizzeria $12 Buy one get one at 75% off
A Pizza Paradise $13 20% off the final price
Dough-re-mis $14 Buy 2 get 1 free
Pan’s $10 Half off the first three boxes

Jake wants to buy all his pizzas from the same shop and to spend as little as possible.

Which shop should Jake buy all his pizzas from?

A. Pop’s PIzzeria
B. A Pizza Paradise
C. Dough-re-mis
D. Pan’s

7 Nisha wants to buy new school supplies for the next school term but doesn’t know where to buy them from. Therefore, she
asked her friends to rate the malls on her list on a scale of 1 to 100.

MALL Cyrus Kitty Jae Willa


Pitt Street 56 62 45 61
Stockland 32 94 53 59
Westfield 49 12 38 42
Chadstone 12 63 81 13

Whose bar chart is being represented?

A. Cyrus
B. Kitty
C. Jae
D. Willa
8 Many individuals have said that living on a farm is the finest experience one can have. While some argue that it is affordable
and healthful, others assert that individuals raised on farms have strong family values. But then, some individuals feel that
city living is superior because of the way of life there.

Which of the following statements, if true, best strengthens the above argument?

A. Those who live on farms usually prioritise their needs over their wants.
B. Living with the family on the farm helps create a strong bond between the family members, which is much needed
in creating a strong society.
C. People living on farms have a simple lifestyle and unlike their city flamboyant lifestyle peers.
D. It is on a farm where you will enjoy most of the beautiful sceneries, fresh air, animals, and fresh food.

9 Each of the following questions contains two statements labelled I, II, and III. There may be a causal connection between the
two statements. These two statements may have the exact cause or have different reasons. These statements might be of
various causes with no connection. Read both statements in each question and choose the correct answer from the given
answer options.

I. Tanya has five cavities.


II. Dental services are expensive.
III. Tanya’s family is poor.

A. Statement I is the effect, and statements II and III are its causes.
B. Statement II is the effect, and statements I and III are its causes.
C. Statement III is the effect, and statements I and II are its causes.
D. The statements do not relate to each other.

10 Both iOS 16 and Android 13 are superb operating systems at their most recent iterations, but in significantly different ways.
Apart from the fundamental touchscreen-focused layout, they seem considerably different from a design perspective while
sharing many of the same capabilities. However, in terms of freedom, android is the best choice.

Which of the following statements, if true, best strengthens the above argument?

A. Google Assistant is more than an excellent voice interface for Google search.
B. When Apple releases a new update or patch, all phones — those that are still supported, anyway — get it.
C. Android phones don’t need to be recharged as often as iPhones.
D. Android is open-source software. It's also far more accepting of alternative applications.
NO. ANSWER

1
HOTEL RATE PER DAY WIFI TOTAL
Hotel A 200 45 245
Hotel B 175 55 230
Hotel C 185 75 260
Hotel D 165 65 225

Since he wants to spend as little as possible, he should choose Hotel D with $225. Option D is the correct answer.

NAME HOBBY COLLEGE PROGRAM

Alisa Crocheting Nursing

Brian Cooking Architecture

Jerome Painting Tourism

Based on the table, it can be concluded that Jerome’s hobby is painting. Option C is the correct answer.

3 The main argument of the passage is that road transport is better than air transport because it is cheaper since
maintenance and operation of air freight are expensive. Options A, B, and C support the argument, and option D opposes
it. Option A can be the answer, but option B clearly states and supports the idea.

Option B is the correct answer.

LIST 1 LIST 2 LIST 3 LIST 4

horror historical drama musical

sci-fi animated action historical

comedy action silent sci-fi

Sci-fi can not be chosen because it appears in lists 1 and 4 where we already chose horror and musical, respectively. The
other choices may be chosen since they can be chosen in list 2, and action can be chosen in list 3 and vice versa. Option D
is the correct answer.

5 The main argument of the passage is that some people prefer dogs because of their affection. Options C and D oppose the
main argument, while options A and B strengthen it. Among options A and B, option A uses the words warm-hearted and
loving, which are synonymous with the argument word affectionate. Therefore, it best strengthens the argument.

Option A is the correct answer.


6 Pop’s Pizzeria:
The special offer means that for every pizza Jake buys, he may get a second one at 75% less than the standard price of $12.
To find what the second pizza will cost, multiply the standard price by 0.75 and subtract the product from $12.
$12 * 0. 75 = $9
$12 − $9 = $3 (𝑇ℎ𝑒 𝑐𝑜𝑠𝑡 𝑜𝑓 𝑡ℎ𝑒 𝑠𝑒𝑐𝑜𝑛𝑑 𝑝𝑖𝑧𝑧𝑎)
$12 (𝑓𝑖𝑟𝑠𝑡 𝑝𝑖𝑧𝑧𝑎) + $3 (𝑠𝑒𝑐𝑜𝑛𝑑 𝑝𝑖𝑧𝑧𝑎) = $15

By repeating the transaction 4 times, Jake can get 8 boxes of pizza for $60:

$15 * 4 = $60

Since he needs 9 boxes, Jake will need to buy the last one at full price, as the special offer cannot be availed by itself. To
get the price of exactly 9 boxes, add $12 to the $60. Thus, it will cost Jake 72 dollars to buy 9 boxes of pizza from Pop’s
Pizzeria.

A Pizza Paradise:
The special offer means that the full price of 9 boxes of pizza bought at the standard price of $13 will be deducted by 20%.
To find the price of 9 boxes, multiply it by $13

9 𝑏𝑜𝑥𝑒𝑠 * $13 = $117

Since 117 dollars is the full price (100%) and it will be subtracted by 20% as stated in the special offer, then Jake will only
have to pay 80% of the full price. To find the actual cost, multiply $117 by .80.

$117 * 0. 80 = $93. 6.

Thus, it will cost Jake 93.60 dollars to buy 9 boxes of pizza from A Pizza Paradise.

Dough-re-mis:
The special offer means that Jake can get 3 boxes of pizza for the standard price of 2. Since the standard price at
Dough-re-mis is $14, multiply it by 2 to get the cost of 3 pizzas:

2 * $14 = $28

Jake must repeat this transaction thrice to get all 9 boxes of pizza.

$28 * 3 = $84.

Thus, it will cost Jake 84 dollars to buy 9 boxes of pizza from Dough-re-mis.

Pan’s:
The special offer means that Jake will only have to pay 50% of the standard price for the first three boxes, and the full price
for the other 6. Since the standard price is $10, the first three boxes will only cost 5 dollars each.
$5 * 3 = $15 𝑓𝑜𝑟 3 𝑏𝑜𝑥𝑒𝑠
$10 * 6 = $60 𝑓𝑜𝑟 𝑡ℎ𝑒 𝑟𝑒𝑠𝑡 𝑜𝑓 𝑡ℎ𝑒 𝑝𝑖𝑧𝑧𝑎
$60 + $15 = $75

Thus, Jake will have to pay 75 dollars for 9 boxes of Pizza at Pan’s. This means that Pop’s Pizzeria will cost Jake the least
amount of money.

Therefore, option A is the correct answer.


7 Based on the chart, Chadstone has a low value. Cyrus and Willa gave low values on Chadstone. Then, the value for
Stockland is near 30. Cyrus has these values.

Option A is the correct answer.

8 All of the statements strengthen the main argument. However, option C is the statement that best strengthens the
argument since it talks about the lifestyle on the farm.

Option C is the correct answer.

9 Due to the expensive dental services and Tanya’s poverty, she had five cavities.

Option A is the correct answer.

10 The main argument of the passage is that by using android, you will have more freedom. Option B opposes the idea, while
options A, C, and D support the argument. However, only option D talks about the freedom of using android.

Option D is the correct answer.


Read the extracts below then answer the questions that follow.

Extract A: How Overthinking Can Affect Mental And Physical Health

Overthinking is not a medical term in and of itself, but evidence indicates that the habit can have actual
effects on our health. Overthinking typically involves focusing on the negative, such as replaying the
past, obsessing on negative events, or worrying about the future.

According to Ashley Carroll, a psychologist at Parkland Memorial Hospital, ruminating on certain


thoughts can snowball into larger, more intense negative thoughts. Carroll argues that overthinking is
problematic because it interferes with daily existence.

"When it becomes harmful to our lives or really hinders our everyday functioning," Carroll said, "for
instance, if you have difficulties sleeping at night because you can't turn these ideas off, that is affecting
your daily functioning." "If it's affecting your appetite or isolating you from others because you're so
immersed in your thoughts..."

Carroll argues that dwelling on the worst-case scenarios and results can be a flawed method of
self-defence.

"For some people, it might be a form of defence," she explained. "For instance, I'll automatically
presume that everyone is untrustworthy, so I won't get close to anyone, thereby safeguarding myself."

Carroll adds that much thinking can also impair physical health. She stated that some of her patients
who struggle with negative thoughts and anxiety have also had headaches, physical aches, and
stomach issues. Overthinking is also frequently linked to mental health conditions.

To overcome the tendency of overthinking, Carroll suggests that a good first step is to identify the
triggers. It may be the result of a past traumatic event or a current source of stress in your life. Carroll
adds that once you have identified these triggers, you may begin to find solutions to overcome them.

She explains, "Whenever patients fall into a ruminating loop, I always recommend controlled breathing
exercises." "It assists them in refocusing their attention on their breathing and soothing their central
nervous system. Then, activities such as journaling assist individuals in expressing and processing their
inner ideas. Therefore, any form of mindfulness activity that requires intense focus on the now might
keep you from dwelling on the past or the future."

Extract B: How Overthinking Can Be Used To Your Advantage

If you have ever experienced the annoyance of overthinking, you understand how aggravating the
situation can be. It is easy to become paralysed by overwhelming anxiety and terror when caught in the
midst of unrestrained thought processes.

However, not all overthinking is negative. In actuality, the process of overthinking is not at fault. Rather,
the difficulty is in the types of thoughts that pass through your mind. If you could regulate your thoughts,
overthinking would no longer be a cause for concern. With this awareness comes immense power, and
you will no longer be trapped in the vicious cycle of overthinking.
We should replace the term "overthinking" with something like "inappropriate negative thinking" or
"poorly directed thinking." Those, however, do not slide off the tongue as readily, which is why the word
"overthinking" has stayed. However, as we have shown, it is not necessarily your thought processes
that are negatively affecting your life and performance. What's genuinely at fault is focusing on the
wrong things!

Fear is the reason it's crucial to realise that overthinking is not the issue, but focusing on the incorrect
areas is. Believing that ideas are the problem makes us scared of our own cognitive processes.
However, you cannot escape your ideas, so there is no point in being afraid of them. By recognising
that poor concentration and negative thoughts are the actual problem, you can minimise your dread.
You should not fear your own thoughts, but instead recognise how amazing it will be if you obtain the
ability to choose your thoughts. Overthinking can and will be used to your advantage in just this way.

Awareness is only as valuable as the actions that follow. Understanding the thoughts occupying your
mind serves little value if you do not use this knowledge to effect good change. Overthinking is only a
problem when you are overthinking the wrong things. So, what if you could learn to control your own
thoughts? That would indicate that excessive thought would no longer be a concern.

Even if you were to overthink again, the control you've developed will ensure that your ideas are the
ones you select. But this requires effort. Though mastering your thoughts is not difficult, it requires
consistency.

Overthinking is not generally a negative trait. The source of trouble is too focusing on the wrong things.
There, insecurity, worry, perfectionism, and indecision flourish. To transform the process of overthinking
from a detriment to an asset, you must learn mental control. The initial step is to master your own mind.
Such mastery cannot be achieved, though, if you continue to run and hide from your thoughts. Face
them head-on to increase your self-awareness and comprehension.

What information can you find on extract A that cannot be found on extract B

Answer the following questions by choosing the letter of the best answer.

1 How are the two articles different from each other?

A. Extract A presents overthinking as a negative trait that can cause negative feelings and hamper
decision making abilities. Extract B says that overthinking is not inherently negative.
B. Extract A states that one can control overthinking tendencies and their thoughts by focusing on
the right things, whereas Extract B argues that there is no way to control overthinking.
C. Extract A focuses on how to turn the process of overthinking into an asset, while Extract B
emphasises the negative aspects of overthinking.
D. Extract A asserts that overthinking is caused by focusing on the wrong things, while Extract B
claims that overthinking is caused by one's surroundings and life events.
2 What similarities do the two extract share?

A. They focus on why overthinking is bad. Both extracts discourage the act of putting too much
thought and effort on things that bothers a person.
B. They emphasise the positive side of overthinking. Both extracts promote the idea that
overthinking is a positive trait that should be encouraged.
C. They address the topic of overthinking. Both extracts highlight the importance of regulating our
thoughts and the role of self-awareness and mental control in achieving this.
D. They focus on the ways to stop overthinking. Both extracts encourage its readers to seek
professional help whenever they feel like they are thinking too much.

3 What information is in extract A that is not found in extract B?

A. Information based on previously conducted research.


B. Information from life experiences of multiple people.
C. Information based on old surveys and outdated data.
D. Information from an expert in the field of medicine.

4 What do the two articles tell us about our thoughts?

A. The two articles tell us that our thoughts can have a significant impact on our emotions,
decisions, and well-being.
B. The two articles suggest that we have no control over our thoughts and that they are entirely
determined by external factors.
C. The two articles imply that overthinking is completely natural and should be embraced in all
circumstances.
D. The two articles suggest that our thoughts have no influence on our emotions, decision-making
abilities or life outcomes.

5 What is the relationship between the two articles?

A. Complementary
B. Contradictory
C. Problem and solution
D. Cause and effect

6 How are the two extracts different in terms of how they were written?

A. Extract A is written in the first person while Extract B is written in the second.
B. Extract A is written informally while Extract B is written in a formal manner.
C. Extract A is written in a sequential form while Extract B is written in a cause and effect structure.
D. Extract A is written in an objective tone while Extract B is written in a conversational and
motivational tone.

Read the poem below then answer the questions that follow.

Service by Edgar A. Guest


To the cause one man gave gold,
Then withdrew into his den
From the battle line, and told
How he served his fellowmen.

When they came and begged for aid, 5


Gladly from his purse he gave,
And he hoped that those he paid
Would continue being brave.

"To the cause,'' another said,


"I've no gold that I can give, 10
But I'll fight for it instead.
Just so long as I shall live."

Day by day and night by night,


He that hath no gold to spare,
In the thickest of the fight 15
Fought and cheered his fellows there.

There are hundreds here to share.


For the principle, their pelf.
But he better serves who'll dare
To a cause to give himself. 20

This world does not need your coin


Half so much as it needs you,
What it wants is men to join
In the work it has to do.

Answer the following questions by choosing the letter of the best answer

7 What is the significance of the poem's title to the message it conveys?

A. It suggests that the poem is about how important it is to make monetary contributions to assist
charitable causes as a form of service.
B. It implies that the poem is about the personal satisfaction and fulfilment that can be gained from
offering service to a cause.
C. It indicates that the poem is about the difficulties of offering one’s service for a cause as well as
the reasons to avoid doing so.
D. It shows the poem's main idea, which is that it is important to fully commit to a cause and take
action on it as a form of service.

8 What is the message that the speaker is trying to convey through the poem?
A. Giving material resources or financial aid is not as important as actively participating in the
cause, to fight for it, and to make a personal sacrifice for it.
B. It is preferable to give money to a cause while sitting comfortably in your home rather than
getting involved in the cause itself.
C. When it relates to supporting a cause, intangible items are not required; the only contribution
that is accepted are the money and necessary goods.
D. It is considerably more difficult to provide financial support to a cause than it is to take part in the
cause itself because you are giving up your own money.

9 Which words can you use to describe the second man?

A. Selfless, kind, and, proactive


B. Giving, hardworking, and famous
C. Rich, powerful, and passionate
D. Intelligent, cunning, and uptight

10 What does lines 7-8 tell us about the first man?

A. He is simply concerned with offering financial assistance to the cause; he does not care whether
or not the recipients of the monetary assistance are courageous or not.
B. He does not have faith in the capabilities of the people he gave money to and is hesitant to give
funds but he is hoping that his gift will help them continue to support the cause.
C. He is completely unconcerned with the outcome of the situation and offered the money so that
he could simply demonstrate how affluent he is.
D. He hoped that the people he gave money to would remain courageous throughout the ordeal;
he believes that the cause is important and requires a continued effort from these people.

11 What is the difference can we observe between the two men just by reading the poem?

A. The two men in the poem have different approaches to serving the cause.
B. The two men in the poem have different educational backgrounds.
C. The two men in the poem have different hobbies and aspirations in life.
D. They are not different, the two men serve the same cause in the same way.

12 What is the main difference between the first man mentioned in the poem and the second
man?

A. The first man has more money to give


B. The first man is more generous than the second man
C. The second man is willing to fight for the cause
D. The second man has more experience with serving others

Read the article below then answer the questions.


There are sentences that have been removed from the article. Choose the correct letter below
that contains the sentence that best fits in the paragraph. Take note that there is an extra
sentence which you do not need to use.

War and its repercussions

War is a catastrophic event that has devastating repercussions on both a societal and personal level. It
is a conflict between nations, states or groups that involves the use of weapons and violence.
13……………… In this essay, we will discuss the repercussions of war, including physical and
psychological trauma, economic devastation, and loss of life.

One of the most significant repercussions of war is the physical and psychological trauma it inflicts on
individuals. War results in injuries, disabilities, and deaths, and those who survive often carry physical
and emotional scars for the rest of their lives. 14……………. These injuries can affect the ability of the
individual to work, support their family, and lead a normal life.

War also causes economic devastation, with countries spending billions of dollars on weapons, troops,
and war-related expenses. 15……………. The economic repercussions of war can be felt for
generations, with countries struggling to rebuild their economies long after the conflict has ended.

Another devastating repercussion of war is the loss of life. War leads to the deaths of soldiers, civilians,
and innocent people caught in the crossfire. The loss of life is tragic and has a significant impact on
families and communities. The families of those killed in war are left with the burden of grief, with many
struggling to cope with the loss of a loved one. War also results in the displacement of people, with
refugees fleeing their homes and seeking shelter in other countries. The displacement of people has a
profound impact on communities, with families torn apart and individuals forced to live in challenging
conditions. 16…………….

In addition to the physical and emotional toll of war, it also has long-lasting political and social
repercussions. War often leads to political instability, with governments collapsing, and new ones taking
their place. 17…………… These divisions can be challenging to heal, and they can lead to ongoing
conflicts and tensions.

In conclusion, war has profound and devastating repercussions on individuals, families, and societies. It
causes physical and psychological trauma, economic devastation, loss of life, displacement, and
long-lasting political and social repercussions. 18…………….. We must work together as a global
community to find ways to resolve our differences peacefully, and to ensure that the horrors of war are
never experienced by future generations.

Choose the letter of the correct sentence that best fits the missing lines in each paragraph.

A. The impact of war on children is especially devastating, with many children orphaned,
separated from their families, or forced to grow up in refugee camps.
B. Physical injuries include loss of limbs, burns, and shrapnel wounds, while psychological
injuries include PTSD, anxiety, depression, and other mental health conditions.
C. World War II began in the 1930’s and is one of the most notable wars that left devastating
repercussions all over the world.
D. The cost of war is immense, with resources diverted from essential services such as
healthcare, education, and infrastructure.
E. War has a profound impact on individuals, families, and societies, and its repercussions can
be felt for generations to come.
F. The social fabric of societies is also impacted, with divisions created between different ethnic,
religious, and social groups.
G. It is essential to work towards preventing war and promoting peaceful solutions to conflicts, as
the cost of war is too high to bear.

Read the extracts below then answer the questions that follow.

A. The most common kind of volcano in the C. Shield volcanoes are the biggest on Earth,
world is a cone made of ash. They may but unlike composite volcanoes, they don't
look like an idealised picture of a volcano form tall mountains with conical peaks.
because they are steep, cone-shaped hills They are instead wide volcanoes with
with a big crater on top. gentle slopes that look like a warrior's
shield laying flat on the Earth. Shield
Cinder cones are called scoria cones when volcanoes have a convex shape because
they are made of scoria. Scoria are pieces they get flatter as you get closer to the top.
of lava that are thrown into the air and are
usually solid when they land. They have Shield volcanoes are so big that they dwarf
irregular shapes and a lot of bubble-shaped other types of volcanoes, even big ones
cavities. that are made up of several smaller ones.

Some cinder cones, like Sunset Crater Shield volcanoes have slopes that are only
Volcano, are almost perfect cones, but a few percent steep and get even less
most of the time they aren't. Cinder cones steep as you get closer to the top. The low
that form over a linear fissure vent are long, viscosity of the lava that comes out of
and ones that form in areas with strong these volcanoes causes thin, wide-spread
prevailing winds may be much taller on the lava flows, eruptions from both the summit
side that is not facing the wind. When the and fissure vents on the sides of the
vent moves during an eruption, it can also volcano, and widening and sinking along
cause long cinder cones to form. They can the summit and rift zones. Lava flows can
also be broken when lava flows coming move through channels or lava tubes for a
from the base of the cone carry away a part long way from where they came out of the
of the flank. ground.

B. Composite volcanoes can be the most D. In some parts of the ocean floor, you can
beautiful ones of all. A typical composite often find underwater volcanoes and
volcano has a cone shape with a concave volcanic vents. Some of them are still
bottom and a steeper top. Most of the time, active and make their presence known by
the highest points of these mountains are shooting steam and rocks high above the
covered in snow and stand high above the surface of the water when they are in
other mountains nearby. shallow water. Many others are at such
great depths that the huge weight of the
Composite volcanoes are probably the water above them creates a high,
most complicated type of volcanic structure squeezing pressure that stops steam and
because they have many different types of gases from forming and exploding. Even
eruptions and erupt lava with a wide range very large eruptions in deep water may not
of chemical compositions. Most composite shake the surface of the ocean.
cones have one main vent at the top, even
if they have more than one vent. Because there is always water around
submarine volcanoes, they can act
Like other mountains, composite volcanoes differently than volcanoes on land. When
will eventually be worn away by the forces sea water flows into active shallow
of erosion. Landslides, rock avalanches, submarine vents, it causes violent
lahars, and debris flows are more likely to steam-blast eruptions. If lava flows into the
happen in places where rocks have been sea from land or erupts onto a shallow sea
changed by heat and water and where floor, it may cool so quickly that it breaks up
pyroclastic deposits are not well formed. into sand and rocks. As a result, a lot of
small pieces of broken volcanic rock are
made.

Answer the following questions by choosing the correct letter that matches the description.
Take note that the letters can be repeated accordingly.

Which extract mentions about…

19 a type of volcano that has many various types of explosions and is presumably the
most complex form of volcanic formation?

20 a type of volcano that could be formed of fragments of lava which are tossed into the air
and typically become hard when they reach the ground?

21 a type of volcano, that a few of them are active, and that when located in shoal waters,
they make their presence felt by striking stones and vapours up into the air?

22 a type of volcano that is so big that it makes other forms of volcanoes appear small,
even those which are composed of many smaller ones?

23 a type of volcano which just like some other landforms, these kinds of volcanoes will be
eroded away over time?

24 a type of volcano that sometimes breaks down whenever it erupts and lava flows down
from its sides?

Read the article below then answer the questions that follow.
Florence Nightingale

​Florence Nightingale was a British nurse, social reformer, and statistician who lived from 1820 to 1910
and was known as "The Lady With the Lamp." She is best known as the person who started modern
nursing. When she was a nurse during the Crimean War, she learned a lot about how to keep things
clean. In 1860, she opened St. Thomas's Hospital and the Nightingale School for Training Nurses. Her
work to improve healthcare in the 19th and 20th centuries had a big impact on the quality of care at that
time.

Florence Nightingale was born in Florence, Italy, on May 12, 1820. Her parents were Frances and
William Shore Nightingale. She was the youngest of her parents' two kids. The wealthy British family of
Florence Nightingale moved in high-class social circles. Her mother, Frances, came from a family of
merchants and was proud of the fact that she knew a lot of important people. Even though her mother
liked to move up the social ladder, Florence was said to be awkward in social situations. She tried not
to be the centre of attention as much as she could. Florence was a strong-willed girl who often got into
fights with her mother, who she thought was too controlling. Still, she was eager to make her mother
happy, like most daughters. In defence of the mother-daughter relationship, Florence wrote, "I think I've
got something more good-natured and agreeable."

William Shore Nightingale was Florence's father. He was a wealthy landowner who had inherited two
estates when Florence was five years old. One was in Lea Hurst, Derbyshire, and the other was in
Embley Park, Hampshire. Florence grew up on the family estate at Lea Hurst. Her father gave her a
classical education, which included learning French, German, and Italian.

Florence Nightingale started helping sick and poor people in the village near her family's estate when
she was still very young. By the time she was 16, it was clear that she was meant to be a nurse. She
thought it was her divine calling. When Nightingale told her parents that she wanted to be a nurse, they
were not happy about it. In fact, her parents told her she couldn't become a nurse. During the Victorian
era, a young woman with Nightingale's social standing was expected to marry a wealthy man, not take
a job that the upper social classes saw as lowly menial labour.

Richard Monckton Milnes, a "suitable" man, asked Nightingale to marry him when she was 17. She said
no. Nightingale told him why she didn't want to be with him. She said that even though he was
intellectually and romantically interesting to her, her "morally active nature" needed to be satisfied, and
that wouldn't happen in this life. Even though her parents didn't want her to, Nightingale went to school
to become a nurse at the Lutheran Hospital of Pastor Fliedner in Kaiserwerth, Germany, in 1844.

Early in the 1850s, Nightingale went back to London and got a job as a nurse in a Middlesex hospital
for sick governesses. Nightingale's performance there was so good that her boss promoted her to
superintendent after only a year on the job. Nightingale's job was hard because there was an outbreak
of cholera and the conditions were dirty, which helped the disease spread quickly. Nightingale made it
her goal to improve hygiene practices, which led to a big drop in the number of deaths at the hospital.
Her health got worse because she had to work so hard. She had just started to feel better when the
hardest thing she had to do in her nursing career came up.

In 1860, she paid for St. Thomas' Hospital and the Nightingale Training School for Nurses to be built
there. Nightingale became someone that people looked up to. Poems, songs, and plays were written
about the hero and dedicated to her. Girls wanted to be like her. Women from the upper classes, eager
to follow her lead, began to sign up for the training school. The upper classes no longer looked down on
nurses because of Nightingale. In fact, they came to see it as a respectable job.

Nightingale got "Crimean fever" while she was at Scutari, and she would never fully get better. By the
time she was 38, she was stuck at home and couldn't get out of bed. This would be the case for the rest
of her life. Nightingale kept working from her bed, as determined and committed as ever to improving
health care and making patients feel better.
Answer the following questions by choosing the letter of the best answer.

25 What was Florence Nightingale known for?

A. Florence Nightingale was known for her adventurous travels, as she was one of the first people
to explore many remote parts of the world.
B. Florence Nightingale is best known as the founder of modern nursing and she was also
commended for her tireless work during a notable war.
C. Florence Nightingale was known for her advancements in the field of medical statistics and data
analysis, pioneering the use of graphs and charts to illustrate healthcare trends.
D. Florence Nightingale was known for her work as a midwife, delivering babies and providing
prenatal care to expectant mothers in her community.

26 What pushed Nightingale to become an advocate for better healthcare and hygiene
practices?

A. her personal desire for fame and recognition in society


B. influence of her friends who were also in the medical field
C. her experiences as a nurse during the Crimean War
D. realisation that the healthcare system in her country needed reform

27 What was the impact of Nightingale's work on nursing and healthcare?

A. She is credited with starting the modern nursing profession; her contributions in the war helped
to reduce the death rate and gain recognition for the importance of nursing.
B. Nightingale's work had no impact on nursing and healthcare, as the advancements in medicine
were solely due to the work of male physicians.
C. Her work in the Crimean War led to the widespread belief that women were unfit for any kind of
professional work, and her legacy was forgotten for many years.
D. Nightingale's work in improving education and teaching, as well as her advocacy for physical
education, helped to revolutionise the educational system.

28 What was the reason for Nightingale's decision to reject Milnes' proposal?

A. Because she was afraid that getting married would distract her from her professional pursuits
and cause her to fall behind her colleagues.
B. Because she did not want to uproot herself from her comfortable existence in England. She felt
that marrying him would drastically change her life for the worse.
C. Because she felt that her moral nature needed to be satisfied and that would not happen in a
marriage with him. She believed that her calling was to be a nurse.
D. Because for her, marrying would stop her from doing all the things she wants to do when she
finally decides to leave the medical field.

29 What did Nightingale do after she got bedridden?

A. She kept working from her bed, she was driven to enhance health care and help people.
B. She ultimately reached the age of retirement and started living a carefree life after that.
C. She abandoned her goal of bringing about positive change in the medical system.
D. She decided to start her own family and settle down somewhere peaceful and quiet.

30 What impact did Florence Nightingale have on the perception of nursing as a profession?

A. It remained a lowly, menial labour job.


B. It became a respectable job for women of the upper classes.
C. It became a job only for men.
D. It was abolished as a profession.
Answer key:

Comparative

1. A
2. C
3. D
4. A
5. B
6. D

Poetry - Service by Edgar A. Guest

7. D
8. A
9. A
10. D
11. A
12. C

Cloze Test - Is waking up early good or bad?

13. E
14. B
15. D
16. A
17. F
18. G

Evaluation - Types of Volcanoes

19. B
20. A
21. D
22. C
23. B
24. A

Non-fiction - Florence Nightingale

25. B
26. C
27. A
28. C
29. A
30. B
1 The game between the Red Lions and the Blue Eagles began at 8:30 PM.

There were four quarters in the game and each quarter lasted for 12 minutes.

If there was a 10-minute break after each quarter, what time did the game end?

A 9:18 PM D 9:48 PM
B 9:28 PM E 9:58 PM
C 9:36 PM

2 Dora the Explorer walked one kilometre in 20 minutes.

How long would it take her to walk 10 kilometres at the same speed?

A 3 hours D 3 hours 30 minutes


B 3 hours 10 minutes E 3 hours 33 minutes
C 3 hours 20 minutes

3 A florist has 72 red roses and 48 pink roses.

If she’s to make 8 bouquets out of the roses, using the same number of each type of
roses in each bouquet, how many roses will be in each bouquet?

A 15 roses D 18 roses
B 16 roses E 19 roses
C 17 roses

4 Last month, the shop owner reduced the price of a dress by 15 percent.

Two weeks later, the shop owner increased the reduced price of the dress by 20
percent.

What is the current price of the dress if it cost $230 before the first price reduction?

A $184.00 D $241.50

B $195.50 E $276.00
C $234.60
5 J & A Transport Services shipped three boxes of widgets yesterday.

The first box had 453 widgets, the second box had three times as many widgets as the
first box had, and the third box had twice as many widgets as the second box had.

What was the total number of widgets J & A Transport Services shipped yesterday?

A 4 500 widgets D 4 560 widgets


B 4 530 widgets E 4 580 widgets
C 4 540 widgets

6 Edward Cullen spent 40 percent of his money on his school project.

If Edward Cullen spent $35, how much money does he have now?

A $87.50 D $52.50
B $72.50 E $48.50
C $66.00

7 The office photocopier can print 20 pages of the document in 5 seconds.

At this rate, how many pages of the document can the office photocopier print in 10
minutes?

A 2 400 pages D 1 600 pages


B 2 000 pages E 1 500 pages
C 1 800 pages

8 Miss Henderson started marking test papers at 11:00 AM.

By 2:00 PM, she had marked 15 test papers.

If she marks the same number of test papers each hour, how many test papers had she
marked by 4:00 PM?

A 5 test papers D 20 test papers


B 10 test papers E 25 test papers
C 15 test papers
9 The numbers below follow a pattern.
9 ? 12 17 15 20
What is the missing number in the pattern?

A 10 D 14
B 11 E 16
C 13

10 Edna can read 50 pages of the book in 45 minutes.

At this rate, how long will it take her to read 195 pages?

A 180 minutes D 144.5 minutes


B 175.5 minutes E 128 minutes
C 160 minutes

11 Bert and Elmo agreed to share the cost of buying a radio which cost $135.50.

3 2
If Bert’s share was 5
of the cost and Elmo’s share was 5
of the cost, how much was
Bert’s share?

A $81.30 D $54.20
B $73.50 E $48.40
C $66.20

12 Steve Jobs, the man behind Apple products, died at the age of 56 in 2011.

In what year was he born?

A 1953 D 1956
B 1954 E 1957
C 1955
13 Rosalie bought a pack of sharpeners at $2.34 and 2 erasers priced at $1.35 each.

The sales tax on the entire purchase was $1.

If Rosalie was left with $4.96, how much money did she have at the start?

A $8 D $11
B $9 E $12
C $10

14 Jack has six times as many candies as Jill.

If Jack has 30 candies, how many candies do the two of them have together?

A 35 candies D 180 candies


B 70 candies E 210 candies
C 90 candies

15 Barbie went to the clothing store to buy a new blouse.

The blouse she selected was discounted by 40 percent off the original price.

What was the original price of the blouse if the discounted price was $34.44?

A $52.20 D $74.44
B $57.40 E $86.10
C $68.60

16 Jacob had only one week left to train for the marathon.

He ran 1 kilometre on Monday, 3 kilometres on Tuesday, 4 kilometres on Wednesday, 7


kilometres on Thursday, and 11 kilometres on Friday.

If Jacob continued this running pattern, how many kilometres would he run on Sunday?

A 15 kilometres D 22 kilometres
B 18 kilometres E 29 kilometres
C 19 kilometres
17 Bart is currently 14 years old, and in 12 years, his father Homer will be twice as old as
he will be.

How old is Homer today?

A 32 years old D 48 years old


B 38 years old E 52 years old
C 40 years old

18 Last month, the Math Club had 25 members.

If the Math Club had 40 members this month, by how many percent did the number of
members increase?

A 15 percent D 37.5 percent


B 24.5 percent E 60 percent
C 30 percent

19 Agnes takes 80 steps per minute and she moves 40 centimetres with each step.

At this rate, how far will Agnes move in two hours?

A 3.84 kilometres D 3 840 kilometres


B 38.4 kilometres E 38 400 kilometres
C 384 kilometres

20 The decimals below follow a pattern.


1.5 ? 6.8 6.95 12.1 12.25
What is the missing decimal in the pattern?

A 6.65 D 1.65
B 6.45 E 1.45
C 1.85
21 Ed is 7 years older than Edd and Eddy is half as old as Ed.

If the average (arithmetic mean) of their ages is 16, how old is the oldest of the three?

A 24 years old D 12 years old


B 22 years old E 11 years old
C 15 years old

22 Batman drove from Gotham City to National City at an average speed of 50 kilometres
per hour, and it took him 1 hour 30 minutes to reach National City

If Batman increases his average speed by 20 percent, he will arrive at National City how
many minutes earlier?

A 25 minutes D 15 minutes
B 20 minutes E 10 minutes
C 18 minutes

23 Ivan left his house at 1:20 PM and reached his grandfather’s house at 1:35 PM.

If he travelled at an average speed of 95 metres per minute, what is the distance


between Ivan’s house and his grandfather’s house?

A 1.425 metres D 142.5 metres


B 14.25 metres E 1 425 metres
C 28.5 metres

24 Amethyst paid $25.50 for a book when it was discounted 15 percent off the regular
price.

If Ruby bought the same book at a discounted price of $24.60, how much discount did
Ruby get?

A 18 percent D 62 percent
B 20 percent E 82 percent
C 22 percent
25 By mass, 60 percent of the healthy snacks are almonds, 30 percent are hazelnuts, and
the rest are pistachios.

If 420 grams of the healthy snack are hazelnuts, how many grams of almonds are in the
healthy snack?

A 210 grams D 960 grams


B 480 grams E 1 680 grams
C 840 grams

26 Dexter was able to answer 90 percent of all the questions in the IQ test and got 75
percent correct on all the questions he answered.

What percentage of all the questions in the test did Dexter answer correctly?

A 67.5 percent D 90 percent


B 75 percent E It cannot be determined.
C 88 percent

27 Three-fourths of the books at the library were paperbacks and the rest were
hard-bound.

If there were 2 848 more paperbacks than hard-bound books at the library, what was
the total number of books at the library?

A 2 136 books D 5 696 books


B 3 642 books E 8 544 books
C 4 272 books

2
28 Seven-ninths of the houses in Arnold’s neighbourhood are painted red, 3
of the
remaining houses are painted blue, and the rest are painted white.

If 6 houses in Arnold’s neighbourhood are painted white, what is the total number of
houses in Arnold’s neighbourhood?

A 60 houses D 78 houses
B 69 houses E 81 houses
C 75 houses
29 Jack and Jill have $985 together.

If Jack has $90 more than Jill, how much money does Jack have?

A $586.50 D $447.50
B $537.50 E $402.50
C $492.50

30 It cost $36 to fence a rectangular garden.

If the length of the garden is 2 metres longer than its width and fencing materials cost
$1.50 per metre, what is the area of the garden?

A 30 square metres D 36 square metres


B 32 square metres E 40 square metres
C 35 square metres

31 Princess Jasmine had $234.

2 1
She donated 5
of the money to a charity organisation and donated 6
of the remaining
amount to the local pet shelter.

How much money did she donate to the local pet shelter?

A $23.40 D $32.20
B $25.60 E $39.00
C $27.00

3
32 Barbie spent 7
of her money on perfume and spent the rest of the money on 14 pairs of
earrings.

If each pair of earrings cost $8, how much did she spend on the perfume?

A $72 D $84
B $76 E $96
C $82
2
33 If 5
of Big Bird’s money is enough to buy 40 sharpeners, how many sharpeners can he
3
buy with 4
of his money?

A 50 sharpeners D 70 sharpeners
B 60 sharpeners E 75 sharpeners
C 65 sharpeners

2
34 Of the 471 current members of the Book Club, 3
are new members.

If half of the new members are males, how many new members of the Book Club are
females?

A 133 D 160
B 141 E 172
C 157

35 There are 13 animals on Old MacDonald’s farm, some are geese and some are goats.

If Old MacDonald counted a total of 40 legs, how many goats are there?

A 5 goats D 8 goats
B 6 goats E 9 goats
C 7 goats
ANSWER KEY
QUESTION CORRECT ANSWER
D
Given:
Start = 8:30 PM
Each quarter = 12 minutes
10 minutes break after each quarter
1
Solution:
Total time = 4(12) + 3(10) = 78 minutes or 1 hour and 18 minutes
8:30 PM + 1 hour 18 minutes = 9:48 PM

C
Given:
1 km = 20 minutes
10 km = ?

2 Solution:
10(20) = 200 minutes

1 ℎ𝑜𝑢𝑟
200 minutes x 60 𝑚𝑖𝑛𝑢𝑡𝑒𝑠
= 3 hours and 20 minutes

A
Given:
72 red roses and 48 pink roses
8 bouquets

3 Solution:
Total roses = 72 + 48 = 120 roses

120/8 = Numbers of flowers per bouquets


Numbers of flowers per bouquets = 15 roses

C
Given:
Last month 15% off
2 weeks later 20% increase
First price = $230
4
Solution:
230 - (230(0.15)) = $195.5

195.5 + (195.5(0.2)) = $234.60


B
Given:
Yesterday = 3 boxes
1st box = 453 widgets
2nd box = 3(453) = 1359 widgets
5 3rd box = 2(1359) = 2718 widgets

Solution:
Total = 453 + 1359 + 2718
Total = 4530 widgets

D
Given:
School project = 40% = $35

Solution:
Let M = Money
6
M(0.4) = 35
M = $87.5

Left = 87.5 - (87.5(0.4))


Left = $52.5

A
Given:
20 pages = 5 seconds

Solution:
60 𝑠𝑒𝑐𝑜𝑛𝑑𝑠
7 10 minutes x 1 𝑚𝑖𝑛𝑢𝑡𝑒
= 600 seconds

600/5 = 120 sets

120(20) = 2400 pages

E
Given:
Start = 11:00 AM
at 2:00 PM = 15 test paper

Solution:
2:00 Pm - 11:00 AM = 3 hours
8
3 hours = 15 test paper
1 hour = 5 test paper

4:00 Pm - 11:00 AM = 5 hours

5(5) = 25 test paper


D
Given:

9 ? 12 17 15 20
9
Solution:
Pattern is add 5 then subtract 2
9 + 5 = 14

B
Given:
50 pages = 45 minutes
195 pages

Solution:
10
50 pages = 45 minutes
9
1 page = 10 minute

9
195 ( 10 ) = 175.5 minutes

A
Given:
Radio = $135.5
Bert = ⅗ (Radio)
Elmo = ⅖ (Radio)
11
Solution:
Bert = ⅗ (135.5)
Bert = $81.30

C
Given:
Died = 56 years old at year 2011
12
Solution:
Born = 2011 - 56
Born = 1955
D
Given:
Pack of sharpeners = $2.34
2 Erasers = $1.35 each
Tax = $1
13 Left = $4.96

Solution:
Money = 4.96 + 2.34 + 1.35 + 1.35 + 1
Money = $11

A
Given:
Jack = 6(Jill)
Jack = 30 candies
14
Solution:
Total = 30 + ⅙ (30)
Total = 35 candies

B
Given:
40% off
Discounted = $34.44

Solution:
15
Let X= original price

X - X(0.4) = 34.44
0.6X = 34.44
X = $57.40

E
Given:
1 week left
Monday = 1km
Tuesday = 3 km
Wednesday = 4 km
Thursday = 7 km
16 Friday = 11 km

Solution:
Pattern is adding the previous 2 numbers
Saturday = 7 + 11 = 18 km
Sunday = 11 + 18
Sunday = 29 km
C
Given:
Bart = 14 years old
Homer + 12 = 2(Bart + 12)

17 Solution:
Homer + 12 = 2(Bart + 12)
Homer + 12 = 2(14 + 12)
Homer = 52 - 12
Homer = 40 years old

E
Given:
25 members = Last month
This month = 40 members

18 Solution:
40 - 25 = 15 members

15
Percentage = 25
x 100
Percentage = 60 %

A
Given:
80 steps per minute
40 cm each step
2 hours

Solution:
1 hour = 60 minutes
2 hours = 120 minutes
19
Number of Steps = 120(80)
Number of Steps = 9 600 steps

Distance = 9 600 (40)


Distance = 384 000 cm

1 𝑘𝑚
384 000 cm x 100 000 𝑐𝑚
= 3.84 km
D
Given:
1.5 ? 6.8 6.95 12.1 12.25
20
Solution:
Pattern is adding 0.15 then adding 5.15
1.5 + 0.15 = 1.65

B
Given:
Ed = Edd + 7
Eddy = ½ Ed
Average = 16

Solution:
21 𝐸𝑑 + 𝐸𝑑𝑑 + 𝐸𝑑𝑑𝑦
16 = 3
1
𝐸𝑑 + 𝐸𝑑 − 7 + ( 2 )(𝐸𝑑)
16 = 3
48 = Ed + Ed - 7 + ½ Ed
55 = 2 ½ (Ed)
Ed = 22 years old

D
Given:
Speed = 50 km per hour
Time = 1 hour and 30 minutes
If speed increases by 20%

Solution:
Increased speed = 50 + 50(0.2) = 60 km per hour
22
Distance = 50(1.5)
Distance = 75 km

New Time = 75/60


Time = 1.25 hours or 1 hour and 15 minutes

1 hour and 30 minutes - 1 hour and 15 minutes = 15 minutes


E
Given:
Left = 1:20 PM
Arrived = 1:35 PM
95 meters per minute
23
Solution:
Time = 1:35 - 1:20 = 15 minutes
Distance = 15(95)
Distance = 1425 meters

A
Given:
Amethyst = $25.5
15% off
Ruby = $24.6

Solution:
Let X = regular price

24 X - X(0.15) = 25.5
0.85X = 25.5
X = $30

Ruby’s Discount:
Let D = Discount
30 - 30(D) = 24.6
30D = 5.4
D = 0.18 or 18%

C
Given:
60% = Almonds
30% = Hazelnuts
Remaining = Pistachios
Hazelnuts = 420 grams

25 Solution:
Let T = Total mass of nuts
(0.3) T = 420 ← Almonds
T = 1400 grams

Hazelnuts = 1400(0.6)
Hazelnuts = 840 grams
A
Given:
Answered = 90%
Correct = 75% of he answered
26
Solution:
Percent = 0.9(0.75) x 100
Percent = 67.5 %

D
Given:
Paperback = ¾
Remaining = Hardbound
Paperback = Hardbound + 2 848
27
Solution:
Total = Hardbound + Paperback
Total = ( ¾ Total - 2848) + ¾ Total
½ (Total) = 2848
Total = 5696 books

E
Given:
7
9
= Red
2
Blue = 3
remaining
White = Left = 6 houses

28 Solution:
Let T = number of houses
7 2
T - T( 9 ) = 9 T ← Remove red
2 2 2 2
9
T- 9T (3) = 27
T ← Remove the Blue or White houses
2
27
T=6
T = 81 houses

B
Given:
Jack + Jill = $985
Jack = Jill + 90

29 Solution:
(Jill + 90) + Jill = 985
2(Jill) = 895
Jill = $447.5
Jack = 447.5 + 90
Jack = $537.50
C
Given:
Fence = $36
L=W+2
$1.5 per meter

Solution:
36/1.5 = Length of fence
Length of fence = 24 meters = Perimeter of garden

24 = 2(L + W)
30
24 = 2(W + 2 + W)
24 = 4W + 4
4W = 20
W = 5 meters

L=5+2
L = 7 meters

Area = 5(7)
Area = 35 m^2

A
Given:
$234
Charity = ⅖
Pet shelter = ⅙ Remaining

Solution:
Charity = ⅖ (234)
Charity = $93.6

Remaining = 234 - 93.6


31
Remaining = $140.4

Pet shelter = 140.4 ( ⅙)


Pet shelter = $23.40
D
Given:
3
Perfume = 7
Remaining = 14 pairs of earrings
Pair of earrings = $8

Solution:
3
Remaining = 1 - 7
4
Remaining = 7
of the money

32 Also;
Remaining = 14(8)
Remaining = $112

Let M = Money
4
112 = 7 M
M = $196

Perfume = 196 ( 7 )
3

Perfume = $84

E
Given:
⅖ of money = 40 sharpeners

Solution:
33 ⅖ of (Money) = 40 sharpeners
0.01 (Money) = 1 Sharpener

3
4
/ 0.01 = Number of sharpener
Number of sharpener = 75 sharpener
C
Given:
471 members
⅔ = New members
½ of New = Males

Solution:
New members = ⅔ (471)
34
New members = 314 members

Males = 314 (½)


Males = 157

Females = 314 - 157


Females = 157 females

C
Given:
13 animals
Total legs = 40 legs

Solution:
Goat = 4 legs
Geese = 2 legs
35
Let G = Number of Goats
Number of geese = 13-G

G(4) + (13 - G)(2) = 40


4G + 26 - 2G = 40
2G = 14
G = 7 goats
NO. QUESTION

1 The piece of cardboard shown below folds to form a house for use with a model railway. The other side of the cardboard is
black, which forms the inside of the house.

Which of the illustrations below correctly shows the assembled house?

A B C D

2 Steve and Gerard want to use a code to send secret messages to each other. In the code, each letter of the alphabet is
represented by a one-digit number as follows:

A B C D E F G H I J K L M N O P Q R S T U V W X Y Z

7 0 4 3 6 9 0 4 5 6 7 8 9 2 3 5 1 1 6 7 5 9 4 5 9 6

For example, the word TOKEN would be written in the code 7-3-7-6-2.

To test the system, Steve writes four words in the code and sends them to Gerard. Gerard tries to decode them and gets the
following answers:

ABOUT
LUCKS
ABORT
TRACE

Gerard is confident he has decoded 3 of these words correctly, but he is unsure about the other one.

Which word is Gerard unsure about?

A. ABOUT
B. LUCKS
C. ABORT
D. TRACE
3 People who struggle to learn a new language can benefit from listening to popular music. According to recent research,
connecting with people's interests and motivating them to learn a language can be accomplished by employing popular
songs, movies, and TV shows. Furthermore, music encourages higher-order thinking and benefits students who are
right-brained and have good hearing. It has been demonstrated that music can improve linguistic relationships. The ability to
continue learning a language while having fun might help learners gain more confidence in their abilities.

Which of the following statements, if true, best strengthens the above argument?

A. Another advantage relates to the mental health challenges that people face, particularly during the pandemic. There
is evidence that popular songs can help reduce anxiety.
B. The youth like popular songs because of their authenticity in the real world, the personalised options available in
streaming libraries, and the relatable topics embedded within them.
C. Pop songs have numerous features that support learning, inspiration and relaxation. Its lyrics are repetitive, easy to
understand, conversational, and they elicit emotional responses.
D. Traditional approaches to adult language teaching are more effective because they frequently employ resources
such as textbooks and generic learning materials.

4 Only rich students will pass the qualifying examination.

If the above statement is true, which of the following statements must also be true?

A. If Adrian fails the qualifying examination, it is because he is not rich.


B. Passing the qualifying exam does not depend on being rich or poor.
C. It is difficult to pass the qualifying examination.
D. If Felix wants to pass the qualifying exam, he may do so even if he is poor.

At the age of 37, LeBron James became the first active NBA player to become a billionaire. Despite earning nearly $400
million in salary during his NBA career with the Los Angeles Lakers, Cleveland Cavaliers, and Miami Heat, James' off-court
earnings far outweigh that. According to Forbes, James has earned more than $900 million in business investments and
endorsements.

If the information in the box is true, whose reasoning is correct?

May: “Aside from being a great basketball player, Lebron James sure knows how to invest his salary in businesses.”
Finn: “Being an NBA superstar like Lebron James surely gives him lots of opportunities outside of the basketball world.”

A. May only
B. Finn only
C. Both May and Finn
D. Neither May nor Finn
6

To pass the first level qualifying exam, a student needs to achieve the following criteria required by the department below:

● Every subject taken by the students must have at least a 70% score. Anything below will be considered a fail.
● The general weighted average of all the subjects taken must at least reach 75% to pass the examination.
● If the general average is between 70% - 75%, then the student will receive a status of conditional and will retake
the examination.

Erica: “I got my results yesterday and I was sad to know that I only got a general weighted average of 73%. However, I know I
can pass since my father is the mayor and the principal owes us big time.”

Which of the following sentences shows the mistake Erica has made?

A. Erica assumes that two events happened because one causes the other to happen.
B. Erica tries to make others accept her ways because it has been done or believed for a very long time.
C. Erica assumed that his father’s relationship with the principal can help her pass the examination.
D. Erica seems to believe that she only has two options where, in fact, there could be more options to choose from.

7 Anton wants to buy 20 jogging pants for his elementary class. There are four available tailoring shops that can make jogging
pants. The shops, the price of the varsity jackets, and the special offers are shown in the table below:

Standard Price of a Varsity Jacket Special Offer

Shop A $45 Buy 8 get 1 free

Shop B $55 40% off for the first 10 jackets

Shop C $65 20% off final price

Shop D $50 No Special Offer

Anton wants to buy all the jogging pants from the same shop and spend as little as possible.

What is the difference in price between the cheapest and most expensive options?

A. $230
B. $190
C. $260
D. $40
8 Most successful basketball players in the hall of fame began playing the sport at an early age, though a significant number of
hoopers played the sport later in life. However, no professional basketball player, who ignored the advice of his coaches, has
become a part of the hall of fame.

If the above statement is true, which of the following statements must also be true?

A. The most successful basketball players play the sport at a young age.
B. Not all professional basketball players who played the sport at a young age ignored their coaches’ advice.
C. No professional basketball player who learned basketball later in life will ignore the advice of his coaches.
D. The more times a basketball player follows his coaches’ advice, the more likely he is to be successful.

9 Margarette invited Joshua to travel south and go to their favourite resting place. They both left their respective houses at 9
AM. Margarette's car travels 80 km/hr while Joshua rides a motorcycle that travels at 50 km/hr. Margarette’s car travelled 115
km to arrive at the restaurant and she waited for her friend to arrive.

How long will it take Joshua to catch up to the restaurant after Margarette arrives?

A. 52 minutes
B. 130 minutes
C. 105 minutes
D. 51 minutes

10 A sculpture is placed in a glass box and displayed in a museum as shown in the diagram below. The sculpture can be viewed
from all sides.

Which one of the following is a possible view from one of the sides of the glass box?

A. B. C. D.
11 Three lab scientists - Alex, Brylle, and Kyle were suspected of eating inside the chemical laboratory. The team supervisor
knows that two of them are guilty. When faced with a suspension of a week, they made the following statements:

Alex: I am guilty.
Brylle: Alex is not guilty.
Kyle: Alex is guilty.

However, only two of the three are telling the truth. The other one is lying.

Which of the following statements must be true?

A. Brylle is telling the truth.


B. Alex is lying.
C. Kyle is lying.
D. Alex is guilty and is telling the truth..

12 The bar graph below depicts book sales (in thousands) from six branches of a publishing company over two consecutive
years, 2015 and 2016.

What percentage is the total sales of branch B6 for both years of the total sales of branch B3 for both years?

A. 88.64%
B. 81.40%
C. 74.42%
D. 90.70%
13 A toy for children may appear to be an unlikely candidate for the classical concert hall. Every year, however, thousands of
musicians gather around the world for toy piano festivals, conferences, and concerts. These composers and performers
gather to discuss the latest developments in toy piano music and perform new pieces while exploring its sound, range, and
playing technique. Italy and Korea have recently held their first toy piano festivals. Despite being designed and marketed for
children and families, toy pianos have been used for decades to compose everything from concertos to pop songs.

Which of the following statements, if true, best strengthens the above argument?

A. Unlike many other instruments used for composition, the toy piano is not globally standardised and there are a lot of
ways to make it.
B. Toy piano specialists are becoming more common as in-demand performers and composers.
C. With its history and connection to childhood, the toy piano is used to convey innocence and nostalgia.
D. Toy pianos offer composers an exciting escape from the relatively closed world of classical music.

14 There are 1,564 books in Fiona’s personal library that she plans to donate. The fiction books must be arranged in 54 boxes.
Every book must be placed in the box before she can donate it to the public library.

How many books did Fiona already place in the donation boxes if 136 books are borrowed by her friends and relatives and
932 books are kept as her personal collection?

A. 992
B. 632
C. 1,428
D. 496

15 Aaron, Draymond, Max, John, and Klay are competing in a local marathon. The competing marathoners are at the final lap of
the competition. The results are shown below:

● Klay finished ahead of Draymond in the final level but came right after Aaron.
● Max is faster than Aaron.
● John finished ahead of Max in a matter of split seconds.
● Draymond stumbled while running and finished the marathon in last place.

Who among the triathletes achieved 4th place?

A. Max
B. John
C. Aaron
D. Klay
16

Stigma associated with COVID-19 diagnosis has the potential to devalue, reject, and exclude school students. This is
synonymous with bullying, and it could indicate that students are looking for someone to blame for the effects of
COVID-19 on their lives.

Emanuel: “Hi, I am Emmauel. I am John’s brother. I hope we can be friends.”


Rose: “Oh, I am sorry. I cannot trust you and make you my friend. You are John’s brother and he tested positive for COVID-19
last year. You might be sick.”

Which of the following sentences shows the mistake Rose has made?

A. Rose insulted Emanuel by throwing personal shades at him, such as commenting on his personality, character, and
looks.
B. Rose assumes that what is true of the whole is true to all its parts.
C. Rose rejects Emanuel to become her friend on the basis of him being John’s brother.
D. Rose does not have any proof or evidence that Emanuel cannot be trusted and cannot be his friend.

17 Daniel is planning to learn sports in the coming summer break. He put up a list of sports he could choose. He plans to pick his
top four sports but he needs to choose one sport per list.

List 1 List 2 List 3 List 4

Hockey Cricket Football Basketball

Snowboarding Badminton Softball Water Polo

Golf Tennis Table Tennis Rugby

Basketball Volleyball Baseball Football

Daniel’s first three choices were: Basketball, Football, and Badminton.

What sport can Daniel not take?

A. Hockey
B. Rugby
C. Tennis
D. Baseball

18 A few statements have been made, and two conclusions have been drawn from them. Assume the statements are true, and
then determine which of the given conclusions follows the statement logically.
Statement 1: Only a few reds are greens.
Statement 2: Only a few greens are blues.
Statement 3: Some blues are oranges.

Conclusion 1: Some reds are not greens.


Conclusion 2: Some greens are not oranges.

Which one of the following statements is true?

A. Both conclusions I and II follow.


B. Only conclusion 1 follows.
C. Only conclusion 2 follows.
D. Neither conclusion 1 nor conclusion 2 follow.

19 Early-risers have better concentration and are more alert and energised throughout the day. Morning people are more
proactive and goal-oriented, have strong problem-solving skills, and do well in school, eventually going to good colleges and
landing high-paying jobs, according to a 2010 study. Adequate and consistent sleep improves blood pressure, immunity, basic
brain functions, and mood, and allows your body to repair itself. Waking up early makes it easier to go to bed early, and
maintaining consistent sleep and wake times regulates your circadian rhythm and leads to deeper sleep at night.

Which of the following best expresses the main conclusion of the argument?

A. Many productive and healthy people rise early. People who get things done recommend waking up early because it
gives you more time for errands or work.
B. If you have to rush out the door because you woke up late, you're more likely to grab a quick and unhealthy snack or
skip breakfast entirely.
C. Working out in the morning when you're most energised, not only gets your day started on the right foot, but it's
also beneficial to your physical and mental health.
D. If you get up early, you can reduce the amount of stress in your life and finish your work without feeling rushed or
under any additional pressure.
20 A net with different sides is shown below.

If the net was folded into a cube, which of the following would be a possible output?

21 Addison and Aurora want to use a code to send secret messages to each other. In the code, each letter of the alphabet is
represented by a one-digit number as follows:

A B C D E F G H I J K L M N O P Q R S T U V W X Y Z

5 3 2 8 7 0 4 7 9 5 6 7 8 7 4 6 4 8 1 7 6 9 5 6 1 2

For example, the word CLOWN would be written in the code 2-7-4-5-7.

To test the system, Addison writes four words in the code and sends them to Aurora. Aurora tries to decode them, and gets
the following answers:

FAITH
BUNCH
MAYBE
SIZES

Aurora is confident she has decoded 3 of these words correctly, but she is unsure about the other one.

Which word is Aurora unsure about?

A. FAITH
B. BUNCH
C. MAYBE
D. SIZES

22 Three pupils - Jona, Samantha, and Almira were suspected of not paying for their food in the canteen. The Prefect of
Discipline knows that two of them are guilty. When faced with a sanction, they made the following statements:

Jona: Samantha is guilty.


Samantha: I am not guilty.
Almira: Samantha is not guilty.

However, only 1 of the three is telling the truth. The other two students are lying.

Which of the following statements must be true?

A. Almira is guilty but not lying.


B. Samantha is guilty and lying.
C. Jona is lying.
D. Samantha is not guilty but lying.

23 In this world, every child should greatly appreciate their parents. Parents are the ones that go through the most hardships
when raising their children to live a healthy and happy life. Every parent wants to see their kids happy and content in life.
They should all have the right to let their children pursue better careers because children occasionally end up making poor
professional judgments. Parents can at least help their children make wise decisions even if they are not the ones working on
their own job paths. No parent wants to make a poor choice that would harm their child's future.

Which of the following statements, if true, best weakens the above argument?

A. Parents should not make career decisions for their children because the constant pressure to live up to the parents'
expectations has a negative impact on the child's mental health.
B. Parents who choose their child's career are more likely to be supportive of their child's future. Many parents who
have failed in the past relive their aspirations in their children.
C. Children should be free to pursue a career that makes them happy or that they are passionate about.
D. Parents must remember that their children's careers are not their own. Their children can make mature decisions
based on their skill set.

24 Harriet wants to buy 20 pairs of sandals for her online shop. Four manufacturers contacted Harriet and gave her special offers
as shown in the table below:

Standard price for a pair of shoes Special Offer


Shop A $125 No Special Offer
Shop B $120 Buy 10 pairs, get 10% off
Shop C $115 Buy 4 get 1 free
Shop D $130 20% off final price

Harriet wants to buy all pairs of sandals from the same manufacturer and she wants to spend as little as possible.

Which shop should Harriet buy all pairs of sandals?

A. Shop A
B. Shop B
C. Shop C
D. Shop D

25 The pie chart below shows Mike’s budget plan under various categories.

If the sum of the two categories is represented by 144 degrees of the whole pie chart, what portions of Mike’s budget are
allocated for it?

A. Medicine and Transportation


B. Bank and New Computer
C. Food Allowance and Personal Cash
D. New Computer and Transportation

26

On any given day in Australia, approximately 43,000 children have a parent in prison. According to researchers, roughly half
of all adults who end up in prison are parents. When an adult is arrested and imprisoned, there is a good chance they are
the parent of someone.

Elijah: “Let’s go to the new ice cream store! I heard it is owned by Sally’s mom.”
Alexis: “Don’t buy there, Elijah. Don’t you know that Sally’s mom just got out of prison? Her mom is not trustworthy.”

Which of the following sentences shows the mistake Alexis has made?

A. Alexis attempts to discourage Elijah from buying in the ice cream store because of the history of Sally’s mom.
B. Alexis tries to make Elijah believe in something because it has been done in the past until now.
C. Alexis gives Elijah two choices, where in fact, there are a lot of choices that Elijah can choose from.
D. Alexis gives personal attacks on Sally by discriminating against her personality, character, and looks.

27 The prices for the National Basketball League games are shown in the table below. One ticket per person is required to watch
the basketball games.

Pass for 1 day Pass for 3 days (consecutive) Pass for 5 days (consecutive)

$170 $250 $340

Kirby, Brylle, Aaron, and Benjamin are planning to watch the live basketball games on the following dates: 7, 8, 9, 17, 18, 19,
20, 21, 29.

How much is the cheapest valid combination that the four friends could use?

A. $4,040
B. $3,040
C. $760
D. $720

28 A few statements have been made, and two conclusions have been drawn from them. Assume the statements are true, and
then determine which of the given conclusions follows the statement logically.

Statement 1: All soft drinks are water.


Statement 2: All water is oil.
Statement 3: Only a few oils are fluid.

Conclusion 1: All soft drinks are oil.


Conclusion 2: No water is fluid.

Which one of the following statements is true?

A. Both conclusions 1 and 2 follow.


B. Only conclusion 1 follows.
C. Only conclusion 2 follows.
D. Neither conclusion 1 nor conclusion 2 follows.

29 Travelling is an important part of life because it is the best way to get away from a hectic schedule. Travelling is a great way to
relieve stress, anxiety, and depression. It also benefits both mental and physical health. We only have one life, and we should
be grateful as it makes us the most advanced creatures on the planet. People not only get to see the beauty of nature, but
also different geographies, topographies, and people. Travelling is about discovering new places, cultures, cuisines, rituals,
and ways of life.

Which of the following best expresses the main conclusion of the argument?

A. Travelling allows us to make friends and connections with people from all over the world. Making connections
abroad is one of the smartest things you can do today.
B. Travelling is unquestionably the best option for those who believe in living a healthy lifestyle. Travellers are less likely
to suffer from health problems because they are more active.
C. The ability to comprehend and develop an appreciation for nonverbal communication is another significant benefit
of travelling.
D. Travelling is essential for mental development and happiness. Travelling is an excellent way to maintain a consistent
pattern of learning and awareness throughout one's lifetime.

30 Please refer to the following object below.

As shown above, which of the nets below CANNOT be folded up to form the object?

31

Before the National Elections on May 9, the Commision on Elections decided to invite each presidential candidate to an
open forum for them to answer essential questions regarding the social issues that the country is facing. Each candidate
will have their time to speak in front of the audience.

Moderator: “Mr Timmy Jr., what is your final message to the people of this country?”
Timmy Jr.: “Winning the national elections is a priority. For the voters, vote for me or live through six more years of poverty.”

Which of the following sentences shows the mistake Timmy has made?
A. Timmy encouraged the people to vote for him because the majority of his supporters also supported him in the
campaign period.
B. Timmy provided two extreme options for the voters, misrepresenting the consequences of choices that are available
when making a decision.
C. Timmy personally attacks other candidates in the open forum to show the people how dominant he is in debates.
D. Timmy thought that what is true for the whole is also true for the parts.

32 The following bar chart depicts the composition of GDP of the two countries (Australia and New Zealand).

If both countries' total GDP is the same, by what percentage is Australia's agricultural income higher than New Zealand's
service income?

A. 100%
B. 200%
C. 133.33%
D. 66.67%

33 In many schools, using a cell phone in class is prohibited. Cell phones ought to be prohibited in schools since they might be
hazardous, distract kids, and enable cheating. On the other hand, students in school are distracted by cell phones. When
students text in class, other students notice, which makes it extremely difficult for them to study. Additionally, if it rings
during class, the teacher may have to pause what she is saying and focus everyone's attention on the cell phone.

Which of the following statements, if true, best weakens the above argument?

A. Cyberbullying can be harder to identify than other forms of bullying, making it tough for teachers to identify and
prevent it when it occurs.
B. Using social media in class can keep students interested and encourage them to participate in discussions. Some
teachers incorporate digital platforms into lessons to encourage participation.
C. While cell phones can encourage participation by providing multiple channels, they can also lead to less in-person
discussion and learning opportunities.
D. Mobile phones should be permitted in schools because they can be used for educational purposes. They can also be
put in silent mode to avoid disruption in classes.

34 There are 1,000 pupils who participated in the mock national exam. The grade school pupils can be classified into the
following categories, as shown in the pie chart below.

Which of the following matches the data in the pie chart?

A.
Variable Year 2 Year 3 Year 4 Year 5 Year 6

Value 200 150 250 350 50


B.
Variable Year 2 Year 3 Year 4 Year 5 Year 6

Value 200 350 150 200 100


C.
Variable Year 2 Year 3 Year 4 Year 5 Year 6

Value 350 75 75 300 200


D.
Variable Year 2 Year 3 Year 4 Year 5 Year 6

Value 250 250 200 150 150

35 In Australia, the number of officially registered homeschooled kids rose from 10,000 in 2011 to nearly 20,000 in 2018. The
ability of parents to choose their child's curriculum is the most significant advantage of homeschooling. Parents have control
over what their children study, when they study, and for how long. Nobody knows their child as they do. And no one cares
more about their child's development than they do. Thus, children are better off when their parents become their teachers.

Which of the following statements, if true, best weakens the above argument?

A. Schools prepare students to meet the fierce competition that they will face in real life. Homeschooling, on the other
hand, does not enable them to compete with other children.
B. Home-schooled children may feel lonely, socially inept, and isolated, especially if they do not have siblings.
C. A parent's educational credentials do not automatically qualify him or her as a good teacher. Parents may not have
been trained to teach, making homeschooling difficult for them.
D. If a child is suffering from social anxiety, depression, or brain health issues, one advantage of homeschooling is the
opportunity to focus on mental and emotional well-being.

36 Drunk driving is prohibited in Australia. Anyone caught by the police will have to pay a fine of $4,000 and will receive a licence
disqualification worth 6 months or more and possible prison time. One night, David was caught by Jasmine, a policewoman,
and he was caught guilty of being under the influence of alcohol.

Jasmine: “You are obviously drunk, Mr David. I need you to surrender your licence.”
David: “No, I am not drunk. I just had a few beers before driving. I am fine.”

Which of the following sentences shows the mistake David has made?

A. David blamed his friends for giving him a few beers, which resulted in him being apprehended by the police.
B. David made personal attacks on the arresting officer even if he was clearly guilty of drunk driving.
C. David disqualifies the claim because of the past history of Jasmine, the policewoman.
D. David used an ambiguous reason for drinking a few beers to escape from the penalty of drunk driving.

37 Eight chess prodigies are competing in the World Chess Championship. Each participant receives up to 21 points in each
round. The results for the first three rounds are shown below:

Round 1 Round 2 Round 3

Vernon 20 15 15

Nicole 21 17 16

Dan 19 19 17

Jester 18 18 17

Sane 17 18 17

Dexter 19 15 17

Simon 18 19 21

Eddy 16 15 18

One competitor will definitely win the competition outright if they score enough points in the fourth and final round.
What is the minimum score that this chess prodigy needs to guarantee the win?

A. 17
B. 18
C. 19
D. 20

38 The prices for the newly opened Ocean park are shown in the table below.

Pass for 1 day Pass for 3 days (consecutive) Pass for 5 days (consecutive)
$75 $145 $210

Alexander, Jordan, and Reynald will need a ticket for the following days of the month: 3, 4, 13, 14, 15, 16, 17, 22, 23, 24
How much is the cheapest valid combination the three of them could use?

A. $1,515
B. $590
C. $2,250
D. $505

39 Joseph is planning to take subjects ahead of his class. He put up a list of subjects he could choose. He plans to pick his top
four subjects but he needs to choose one subject per list.

List 1 List 2 List 3 List 4

English Health Economics Coding

Music Earth Science History Oral Communication

Mathematics Physics Effective Communication Financial Management

Biology Music Linguistics Mathematics

Joseph’s first three choices were: Music, Mathematics, and Effective Communication.

What subject can Joseph not take?

A. Biology
B. Economics
C. Coding
D. Physics
40 Denise wants to tile her kitchen floor and is trying to decide between her four favourite tiles. However, only three of them are
suitable for the job as one of them won’t cover the floor without leaving numerous gaps.

Which one is it?

A. B. C. D.
NO. ANSWER

1 The correct illustration of the assembly of the house can only be either from these two combinations:

Combination 1 Combination 2
Grey front roof White front roof
White front wall Grey front wall
White side wall White side wall
Grey side roof Grey side roof

Option A matches with Combination 2. Option A is the correct answer.

2 ABOUT - 70357, the possible letter combinations are as follows: A/K/T, B/G, D/O, I/P/U/X, A/K/T
● ABOUT

LUCKS - 85476, the possible letter combinations are as follows: L, I/P/U/X, C/H/W, A/K, E/J/S/Z
● LUCKS
● LICKS

ABORT - 70317, the possible letter combinations are as follows: A/K/T, B/G, D/O, Q/R, A/K/T
● ABORT

TRACE - 71746, the possible letter combinations are as follows: A/K/T, Q/R, A/K/T, C/H/W, E/J/S/Z
● TODAY

Therefore, Gerard is unsure of the word LUCKY. Option B is the correct answer.

3 The main argument talks about how pop songs can help alleviate the struggle that people usually experience when learning a
new language. The best statement that strengthens it is Option C because it points out the versatile features of pop songs that
help enhance learning while keeping listeners relaxed and inspired. Furthermore, it also ties back to the main argument by
talking about the characteristics of the songs and their lyrics, which help people evoke strong emotional responses. Option A is
a close answer but its justification focuses solely on how pop songs help people overcome their mental health challenges and
it does talk about language learning. Same with Option A, Option B is a close answer but it only points out the originality and
accessibility of pop songs to the audience, not talking about how it directly influences learning a new language. Option D gives
the opposite of what is being asked so it cannot be the answer. Option C is the correct answer.
4 Option A is the correct answer as passing the qualifying examination is conditional on being rich. Although you may be
confused with other answer options, be aware that the logical indicator “only” suggests that a portion of the students will pass
the qualifying examination if they are rich.

Option A is the correct answer.

5 Both May and Finn have correct reasoning. May is correct in pointing out that Lebron James surely knows how to use the
money he earned playing basketball. By investing in businesses, his money grew exponentially, which made him a billionaire.
Finn is also correct in saying that a lot of opportunities await NBA superstars. In the passage, Lebron James is suggested to
have attracted lots of endorsements and business investments, implying his popularity as an NBA superstar makes it possible
to gain many opportunities outside of basketball.

Option C is the correct answer.

6 Option C is the correct answer because Erica assumes that her father and his relationship with other people, such as the
principal, can save her from not failing the qualifying exam. It is a logical flaw because her father’s popularity does not
necessarily mean that he can do everything for the things Erica wanted. Option A is incorrect because the statement is not a
cause-and-effect situation. Option B is also incorrect because the statement does not discuss tradition. Lastly, Option D is also
incorrect because it does not centre on choosing between two options, which means that the type of flaw is clearly irrelevant
to the statement made by Erica.

Option C is the correct answer.

7
Standard Price of a Special Offer Total
Varsity Jacket

Shop A $45 Buy 8 get 1 free ($45 x 8) + ($45 x 8) + $45 +


$45
= $810

Shop B $55 40% off for the first 10 ($55 x 10 x 0.60) + ($55 x 10)
jackets = $880

Shop C $65 20% off final price $65 x 20 = $1,300 - $ 260 (20%
off)
= $1,040

Shop D $50 No Special Offer $50 x 20


= $1,000

Highest price offered comes from Shop C.


Lowest price offered comes from Shop A.

$1, 040 − 810 = $230

Option A is the correct answer.


8 The given statement states that no professional basketball player who ignored the advice of his coaches has become a part of
the hall of fame. However, it does not imply that all professional basketball players who played the sport at a young age
ignored their coaches’ advice. This means that some professional basketball players who played the sport at a young age may
have followed their coaches' advice and have become successful enough to be included in the hall of fame.

Option A is not necessarily true because the statement only mentions that a significant number of hoopers played the sport
later in life, but it does not imply that the most successful basketball players played the sport at a young age.

Option C is not necessarily true because the statement only mentions that no professional basketball player who learned
basketball later in life ignored the advice of his coaches, but it does not imply that all professional basketball players who
learned the sport later in life followed their coaches' advice.

Option D is not necessarily true because the statement does not mention any correlation between following coaches' advice
and becoming successful in basketball. It only states that no professional basketball player who ignored the advice of his
coaches has become a part of the hall of fame.

Option B is the correct answer.

9 𝑀𝑎𝑟𝑔𝑎𝑟𝑒𝑡𝑡𝑒'𝑠 𝑠𝑝𝑒𝑒𝑑: 80 𝑘𝑚/ℎ𝑟


𝐽𝑜𝑠ℎ𝑢𝑎'𝑠 𝑠𝑝𝑒𝑒𝑑: 50 𝑘𝑚/ℎ𝑟

𝑊ℎ𝑒𝑛 𝑀𝑎𝑟𝑔𝑎𝑟𝑟𝑒𝑡𝑡𝑒 𝑠𝑡𝑜𝑝𝑝𝑒𝑑 𝑎𝑡 115 𝑘𝑚, 𝐽𝑜𝑠ℎ𝑢𝑎 ℎ𝑎𝑠 𝑡𝑟𝑎𝑣𝑒𝑙𝑙𝑒𝑑 𝑓𝑜𝑟


115 / 80 = 1. 44 ℎ𝑜𝑢𝑟𝑠

𝐽𝑜𝑠ℎ𝑢𝑎 𝑤𝑖𝑙𝑙 𝑡𝑎𝑘𝑒 115 / 50 = 2. 3 ℎ𝑜𝑢𝑟𝑠 𝑡𝑜 𝑟𝑒𝑎𝑐ℎ 𝑡ℎ𝑒 𝑠𝑎𝑚𝑒 𝑑𝑖𝑠𝑡𝑎𝑛𝑐𝑒 𝑎𝑠 𝑀𝑎𝑟𝑔𝑎𝑟𝑒𝑡𝑡𝑒
= 2. 3 ℎ𝑜𝑢𝑟𝑠 − 1. 44 ℎ𝑜𝑢𝑟𝑠 = 0. 86 ✕ (60 𝑚𝑖𝑛 / ℎ𝑜𝑢𝑟) = 51. 6 𝑚𝑖𝑛𝑢𝑡𝑒𝑠
= 52 𝑚𝑖𝑛𝑢𝑡𝑒𝑠 (𝑟𝑜𝑢𝑛𝑑𝑒𝑑 𝑜𝑓𝑓)

It will take 52 minutes for Joshua to catch up at their favourite resting place.

Option A is the correct answer.

10 Option A is the Front View but has no Fillet Line.


Option C is the Top View but has an extra line.
Option D is the Left View but has no line emphasising an extruded part.
Option B is the Front View.

B. C. D.

A.

Option B is the correct answer.


11

Based on the table above, we can say that:

Alex is guilty and is telling the truth.


Brylle is not guilty but lying.
Kyle is guilty and is telling the truth.

Option D is the correct answer.

12 Getting the total sales of both B6 and B3 for both years,

𝐵6 = 110 + 85 = 195
𝐵3 = 120 + 95 = 215
𝐵6 / 𝐵3 = 195/215 = 0. 906976
0. 906976 ✕ 100% = 90. 70%

𝑇ℎ𝑒 𝑡𝑜𝑡𝑎𝑙 𝑠𝑎𝑙𝑒𝑠 𝑜𝑓 𝐵6 𝑟𝑒𝑝𝑟𝑒𝑠𝑒𝑛𝑡𝑠 90. 70% 𝑜𝑓 𝑡ℎ𝑒 𝑡𝑜𝑡𝑎𝑙 𝑠𝑎𝑙𝑒𝑠 𝑖𝑛 𝐵3.

Option D is the correct answer.

13 The argument tells us about the rising trend of using toy pianos in formal music, concerts, and festivals. The best statement
that strengthens it is Option D as it highlights how toy pianos become a new way for artists to challenge and surprise
audiences. What's accepted on the concert stage is always changing, and the rise of the toy piano suggests we're ready to
welcome new sounds and instruments into the relatively closed world of classical music. Although Option C is a close answer,
it is still incorrect because it only describes the purpose of using the toy piano and it does not mention the current trend of
using toy pianos among musicians. Same with Option C, Option B is incorrect because it only talks about toy piano specialists.
Option A attempts to weaken the argument by stating that toy pianos vary from country to country so it cannot be the answer.

Option D is the correct answer.

14 To know how many books are already in the donation boxes, subtract the total number of books borrowed and the total
number of books kept in the collection box from the total books in her home library:

1, 564 − 136 − 932 = 496

There are 496 books placed already in the donation boxes.

Option D is the correct answer.

15 The marathoners who finished the race from first to last are John > Max > Aaron > Klay > and Draymond. The last finisher was
Draymond even if he stumbled while running. Klay finished fourth in the race because he cannot outrun Aaron. Aaron was
faster than Klay but trailed behind Max. John is the fastest among the runners. Therefore, Klay is in fourth place.
Option D is the correct answer.

16 Option C is the correct answer because Rose believes that Emanuel is not worthy to be her friend since he is John’s brother,
implying that she prefers not to be friends with someone who got sick from COVID-19. Rose’s statement is an example of guilt
by association where she rejects Emanuel because he is connected to a person that got sick from COVID-19. Option A is not
the correct answer because Rose was not giving any personal attacks on James so it is not ad hominem. Option B is incorrect
because Rose’s statement is not a fallacy of division. Lastly, Option D is also incorrect because Rose already had proof of
disliking Emanuel, which is having the information that Emanuel is John’s brother.

Option C is the correct answer.

17 As Badminton is only in list two, we know that the sports that Daniel cannot choose are under list two as well. Basketball and
Football can be found in other lists however Badminton stands alone in list two. Thus, Daniel cannot take Tennis.

Option C is the correct answer.

18

Only conclusion 1 follows. Since there is not enough information provided about the relationship of greens and oranges, we
cannot ascertain if some greens are indeed oranges, making conclusion 2 faulty. Option B is the correct answer.

19 The most important points of the main argument are the characteristics of the morning people, their personality, how
successful they are, and the health benefits of waking up early. The statement in Option A was able to identify these points
and was also able to determine and state the connections among the main points. It states that most productive and healthy
people wake up early to start doing their tasks and have an efficient way to use their time throughout the day. Option B is
incorrect because it only describes having an unhealthy snack if you wake up late. Option C is also incorrect because it only
mentions the benefit of having good exercise when you wake up early. Lastly, Option D is also incorrect because it only states
about not having stress at the start of the day.

Option A is the correct answer.

20 Option B is not applicable since the “Line” should be pointing towards the “Diamond Box”.
Option C is not applicable since the “Line” should be pointing towards the “Small Box”.
Option D is not applicable since the Circle, Small Box, and Diamond Box are not Adjacent faces.
Option A satisfies the condition when the cube is being formed.

Option A is the correct answer.

21 FAITH - 05977, the possible letter combinations are as follows: F, A/J/W, I/V, E/H/L/N/T, E/H/L/N/T
● FAITH
● FAINT
BUNCH - 36727, the possible letter combinations are as follows: B, K/P/U/X, E/H/L/N/T, C/Z, E/H/L/N/T
● BUNCH

MAYBE - 85137, the possible letter combinations are as follows: M/R, A/J/W, S/Y, B, E/H/L/N/T
● MAYBE

SIZES - 19271, the possible letter combinations are as follows: S/Y, I/V, C/Z, E/H/L/N/T, S/Y
● SIZES

Therefore, Aurora is unsure of the word FAITH.

Option A is the correct answer.

22

Based on the table above, we can say that:


Jona is guilty but is telling the truth.
Samantha is guilty and lying.
Almira is guilty and is not telling the truth.

Option B is the correct answer.

23 The main argument discusses the role of parents in choosing their children’s careers. The best statement that weakens it is
Option A because it points out how mentally draining it is on the part of children who constantly follow all their parents’
decisions. Furthermore, living up to expectations is too much of a burden for a child and it makes them demotivated at an
early age. Although Option C is a close answer, it is still incorrect because it only talks about children needing the freedom to
choose their careers without proving why parents should not be the ones making the career decisions. Same with Option C,
Option D is also incorrect because its idea is only about assuming that children can make mature decisions, which is not
entirely a contradiction to the main idea of the argument. Option B strengthens the argument so it cannot be the answer.

Option A is the correct answer.


24

Standard price for a pair of Special Offer Total


sandals

Shop A $125 No special offer $125 x 20


= $2,500

Shop B $120 Buy 10 pairs and get 10% off ($120 x 10 x 0.90) + (120 x
10 + 0.90)
= $2,160

Shop C $115 Buy 4 get 1 free ($115 x 4) + ($115 x 4) +


($115 x 4) + ($115 x 4)
= $1,840

Shop D $130 20% off final price $130 x 20 = $2,600 - $520


(20% discount)
= $2,080

Shop C offers the cheapest among all the stores. Option C is the correct answer.

25 Calculating each answer option to know which combination results in 144 degrees,

𝑂𝑛𝑒 𝑓𝑢𝑙𝑙 𝑐𝑖𝑟𝑐𝑙𝑒 𝑖𝑠 𝑒𝑞𝑢𝑖𝑣𝑎𝑙𝑒𝑛𝑡 𝑡𝑜 360°


𝑀𝑒𝑑𝑖𝑐𝑖𝑛𝑒 𝑎𝑛𝑑 𝑇𝑟𝑎𝑛𝑠𝑝𝑜𝑟𝑡𝑎𝑡𝑖𝑜𝑛 = 5% + 15% = 20% ✕ 360° = 72°
𝐵𝑎𝑛𝑘 𝑎𝑛𝑑 𝑁𝑒𝑤 𝐶𝑜𝑚𝑝𝑢𝑡𝑒𝑟 = 10% + 25% = 35% ✕ 360° = 126°
𝐹𝑜𝑜𝑑 𝑎𝑙𝑙𝑜𝑤𝑎𝑛𝑐𝑒 𝑎𝑛𝑑 𝑝𝑒𝑟𝑠𝑜𝑛𝑎𝑙 𝑐𝑎𝑠ℎ = 30% + 15% = 45% ✕ 360° = 162°
𝑁𝑒𝑤 𝐶𝑜𝑚𝑝𝑢𝑡𝑒𝑟 𝑎𝑛𝑑 𝑇𝑟𝑎𝑛𝑠𝑝𝑜𝑟𝑡𝑎𝑡𝑖𝑜𝑛 = 25% + 15% = 45% ✕ 360° = 144°
𝑇ℎ𝑒 𝑐𝑜𝑚𝑏𝑖𝑛𝑎𝑡𝑖𝑜𝑛 𝑜𝑓 𝑁𝑒𝑤 𝐶𝑜𝑚𝑝𝑢𝑡𝑒𝑟 𝑎𝑛𝑑 𝑇𝑟𝑎𝑛𝑠𝑝𝑜𝑟𝑡𝑎𝑡𝑖𝑜𝑛 𝑤𝑖𝑙𝑙 𝑟𝑒𝑠𝑢𝑙𝑡 𝑡𝑜 144°.

Option D is the correct answer.

26 The best statement that shows the mistake that Alexis made is Option A because it points out how an irrelevant history of
Sally’s mom is enough to discourage Elijah from buying in the ice cream store. This is an example of a genetic fallacy where a
person discredits a claim by pointing out information that is irrelevant to the argument made. Option B is incorrect because
Alexis did not appeal to tradition. Option C is also incorrect because the logical flaw was not a false dilemma and Elijah was not
deciding on two options. Lastly, Option D is incorrect because Alexis did not give any personal attacks on Sally. She only
pointed out the history of her mom, who just got out of prison.

Option A is the correct answer.


27 Listing all the possible combinations to know which one is the cheapest,

Combinations Total Payment

Option 1 Pass for 3 days: 7, 8, 9 $250 + $250 + $170 + $170 + $170 =


Pass for 3 days: 17, 18, 19 $720
Pass for 1 day: 20
Pass for 1 day: 21 Since four of them are going: $1,010 x
Pass for 1 day: 29 4
= $4,040

Option 2 Pass for 3 days: 7, 8, 9 $250 + $340 + $170


Pass for 5 days: 17, 18, 19, 20, 21 = $760
Pass for 1 day: 29
Since four of them are going: $760 x 4
= $3,040

Option 3 Pass for 1 day: 7, 8, 9, 17, 18, 19, 20, $170 x 9 = $1,530
21, 29
Since four of them are going: $1,530 x
4
= $6,120

The cheapest valid combination is $3,040. Option B is the correct answer.

28

Only conclusion 1 follows. Option B is the correct answer.

29 The points of the passage are about the good things that happen when you travel such as getting away from a busy schedule,
relieving stress and anxiety, interacting with different people, and getting to see the various wonders of the world. Only Option
D summarises the passage as it indicates how travelling can help us achieve happiness and mental stability. Travelling gives us
constant knowledge and lessons through the experience we get when we travel. Option A is incorrect because it only talks
about the importance of building friends and networking throughout the world. Option B is also incorrect because it only talks
about how we achieve a healthy life by travelling. Lastly, Option C is also incorrect because it only talks about communication
among various cultural groups, which does not summarise what the passage is all about.

Option D is the correct answer.


30 Options A, B, and D can all be folded up to form the given object.
Option C will not form the given object as one face would be missing in the object as shown below:

Option C is the correct answer.

31 Option B best shows Timmy's mistake because he stated there were only two options for the voters: either people vote for him
or everyone will suffer six more years of poverty. A false dilemma involves a choice between two mutually exclusive options,
implying that no other options exist. The choice to vote for Timmy appears obvious because another option is clearly worse
than the other, which is poverty. Option A is incorrect because the logical flaw is about appeal to the majority. There was no
basis to say that the majority wanted to vote for him other than his supporters. Option C cannot be the answer because
Timmy did not give any personal attacks to other candidates. Lastly, Option D is not the answer because the logical flaw is not
a fallacy of division.

Option B is the correct answer.

32 Plotting the actual percentage of each GDP composition,

𝐴𝑢𝑠𝑡𝑟𝑎𝑙𝑖𝑎'𝑠 𝑎𝑔𝑟𝑖𝑐𝑢𝑙𝑡𝑢𝑟𝑎𝑙 𝑖𝑛𝑐𝑜𝑚𝑒 = 30


𝑁𝑒𝑤 𝑍𝑒𝑎𝑙𝑎𝑛𝑑'𝑠 𝑠𝑒𝑟𝑣𝑖𝑐𝑒 𝑖𝑛𝑐𝑜𝑚𝑒 = 15
𝐷𝑖𝑓𝑓𝑒𝑟𝑒𝑛𝑐𝑒 = 30 − 15 = 15
𝐷𝑖𝑓𝑓𝑒𝑟𝑒𝑛𝑐𝑒/𝑁𝑒𝑤 𝑍𝑒𝑎𝑙𝑎𝑛𝑑'𝑠 𝑠𝑒𝑟𝑣𝑖𝑐𝑒 𝑖𝑛𝑐𝑜𝑚𝑒 = 15/15 = 1
1 ✕ 100% = 100%

Option A is the correct answer.


33 The main argument tells us the reasons why cell phones must be prohibited in schools. Students who carry cell phones may
disrupt classes and also distract them from learning their lessons. The best statement that weakens the argument is Option D
because it points out that cell phones can be used for educational purposes to help students academically as well as put their
phones in silent mode to avoid distraction during class discussions. Option A, B, and C are irrelevant to the point of the
passage.

Option D is the correct answer.

34

The table that is shown in Option A, best matches the information in the pie chart. Option A is the correct answer.

35 The argument tells us that children are better if they are homeschooled, especially if their parents will be the ones teaching
them. It highlighted the rising trend of homeschooled children and the benefits it gives to both parents and children. The best
statement that weakens it is Option C because it points out that parents cannot be automatically recognized as good teachers
since they do not have the same training as a professional teacher and it may become a burden to the learning development
of the children. Option B is a close answer but is still incorrect because it only talks about the lack of social interaction of a
homeschooled only child but it does not apply to all instances of homeschooled children since some have siblings and can still
interact with their parents. Same with Option B, Option A is incorrect because it only talks about competition as a catalyst for
child development, which does not contradict the main idea of the argument. Option D strengthens the argument so it cannot
be the answer.

Option C is the correct answer.

36 Option D is the correct answer because the phrase “a few beers” is an ambiguous way of reasoning out to a police officer that
David was not yet drunk because he only had a few drinks. David’s statement is an example of a fallacy of equivocation where
ambiguous phrases have double meanings. It appears true but given that it has another meaning, it creates a fallacy. Option A
is incorrect because the fallacy relates to a cause and effect, but it was not stated in David’s statement that he blamed his
friends. Option B is also incorrect because David did not personally attack the arresting officer. Lastly, Option C is incorrect
because David did not know any history or personal information about the policewoman.

Option D is the correct answer.


37 Ranking the chess prodigies based on their total scores after the first three rounds,

Round 1 Round 2 Round 3 Total Ranking

Vernon 20 15 15 50 7th

Nicole 21 17 16 54 3rd

Dan 19 19 17 55 2nd

Jester 18 18 17 53 4th

Sane 17 18 17 52 5th

Dexter 19 15 17 51 6th

Simon 18 19 21 58 1st

Eddy 16 15 18 49 8th

Based on the table above, the top two chess prodigies after round 3 were Simon and Dan.
To ensure his win, Simon needs to have 19 points in the final round to secure the victory.
If Nate gets 19 points, his total score will be: 58 + 19 = 77 points.
Even if Dan gets 21 points in the final round, it will not be enough to win or tie Nate’s total score because 55 + 21 = 76 points.

Option C is the correct answer.

38 Listing all the possible combinations to know which one is the cheapest,

Combinations Total Payment

Option 1 Pass for 3 days: 13, 14, 15 $145 + $145 + $75 + $75 + $75 + $75 =
Pass for 3 days: 22, 23, 24 $590
Pass for 1 day: 3
Pass for 1 day: 4 Since three of them are going:
Pass for 1 day: 16 $590 x 3 = $1,770
Pass for 1 day: 17

Option 2 Pass for 3 days: 22, 23, 24 $145 + $210 + $75 + $75 = $505
Pass for 5 days: 13, 14, 15, 16, 17
Pass for 1 day: 3 Since three of them are going:
Pass for 1 day: 4 $505 x 3 = $1,515

Option 3 Pass for 1 day: 3, 4, 13, 14. 15, 16, 17, $75 x 10 = $750
22, 23, 24
Since three of them are going:
$750 x 3 = $2,250

The cheapest valid combination is $1,515. Option A is the correct answer.


39 The main rule we should put in mind is that we can only choose one subject per list. As Effective Communication is only in list
three, we know that the subjects that Joseph cannot choose are under list three as well. Music and Mathematics can be found
in other lists however Effective Communication stands alone in list three. Among the answer options, Economics is the only
subject found in list 3 together with Effective Communication. Thus, Joseph cannot take Economics.

Option B is the correct answer.

40

Only Tile A has numerous gaps when assembled. It is a polygon with a high number of sides which is comparable to a circle.

Option A is the correct answer.

You might also like